Sei sulla pagina 1di 175

TONBRIDGE SCHOOL

PHYSICS AQA A 1451/2451


Revision Easter 2011

Information
This document contains elements that should be used for revision in preparation for the mock exams when you return and the Physics exams in the summer: PHYA4/1&2 Tuesday 21st June am Fields and Further Mechanics PHYA5A/1 and PHYA5/2A Monday 27th June am Nuclear and Thermal Physics/ Astrophysics. Both exams will be 1 hr 45 mins long; remember to bring a calculator, ruler and BLACK pen. You should have the CGP Revision book. The following list are some examples of revision websites; they are no means comprehensive and are often aimed at B grade candidates, so Tonbridge students shouldnt get any questions wrong! http://ranelaghalevelphysics.wikispaces.com/A2Physics AWESOME SITE great for revision http://www.s-cool.co.uk/alevel/physics.html http://www.schoolphysics.co.uk/age16-19/ http://www.antonine-education.co.uk/Physics%20A%20level/welcome_to_a2_physics.htm

Index
Pages 2 Pages 3-12 Pages 13 175 Specification break down Specification topics list 6 past papers with mark schemes/model answers and examiners reports you may have seen some of these questions before.

Ralph Fleming Head of Physics

GCE Physics A Specification for AS exams 2009 onwards and A2 exams 2010 onwards (version 1.1)

2 Specification at a Glance

AS Examination Unit 1 PHYA1 Particles, quantum phenomena and electricity Written Examination (70 marks/120 UMS), 6 or 7 structured questions 1 hours 40% of the total AS marks 20% of the total A Level marks Available January and June Unit 2 PHYA2 Mechanics, materials and waves Written Examination (70 marks/120 UMS), 6 or 7 structured questions 1 hours 40% of the total AS marks 20% of the total A Level marks Available January and June Unit 3 Investigative and practical skills in AS Physics Either PHA3T, Centre Marked Route T 50 marks Practical Skills Assignment (PSA 9 raw marks) Investigative Skills Assignment (ISA 41 raw marks) Or PHA3X, Externally Marked Route X 55 marks Practical Skills Verification (PSV teacher verification) Externally Marked Practical Assignment (EMPA 55 raw marks) 20% of the total AS marks 10% of the total A Level marks Available June only A2 Examination Unit 4 PHYA4 Fields and further mechanics Written Examination (75 marks/120 UMS) 1 hours Section A is 25 multiple choice questions, each worth one mark. Section B is a written paper of 4/5 structured questions and consists of 50 marks. 20% of the total A Level marks Available January and June Unit 5 One of Units PHA5A, PHA5B, PHA5C, PHA5D Written Examination (75 marks/120 UMS) 1 hours Section A: Nuclear and Thermal Physics 40 marks Compulsory section 4/5 structured questions Section B: one of the following options. Each paper has 4/5 structured questions and 35 marks.

AS Award 1451

A Level Award 2451

Options: A Astrophysics B Medical Physics C Applied Physics D Turning Points in Physics 20% of the total A Level marks (Section A 10%, Section B 10%) Available June only Unit 6 Investigative and practical skills in A2 Physics Either PHA6T, Centre Marked Route T 50 marks Practical Skills Assessment (PSA 9 marks) Investigative Skills Assignment (ISA 41 marks) Or PHA6X, Externally Marked Route X 55 marks Practical Skills Verification (PSV teacher verification) Externally Marked Practical Assignment (EMPA 55 raw marks) 10% of the total A Level marks Available June only

AS

A2

A Level

GCE Physics A Specification for AS exams 2009 onwards and A2 exams 2010 onwards (version 1.1)

New GCE Physics AA specification for first teaching 2008: version 0.2, draft submitted to to QCA (July 2007) New GCE Physics A specification specification for first teaching 2008: version 0.2, draft submitted to QCA (July 2007) New GCE Physics for first teaching 2008: version 0.2, draft submitted QCA (July 2007) New GCE Physics A specification for teaching first teaching 2008: version 0.2, draft submitted to QCA (July 2007) New GCE Physics A specification for first 2008: version 0.2, draft submitted to QCA (July 2007) New GCE Physics A specification for first teaching 2008: version 0.2, draft submitted to QCA (July 2007) New GCE Physics A specification for first teaching 2008: version 0.2, draft submitted to QCA (July 2007) New GCE GCE Physics Physics A A specification specification for for first first teaching teaching 2008: 2008: version version 0.2, 0.2, draft draft submitted submitted to to QCA QCA (July (July 2007) 2007) New

3.4 Unit 44 Fields and Further Mechanics 3.43.4 Unit 4 Fields and Further Mechanics 3.4 Unit Fields and Further Mechanics Unit 4 PHYA4 Fields and Further Mechanics 3.4 Unit 4 Fields and Further Mechanics 3.4 Unit 4 Fields and Further Mechanics 3.4 Unit 4 Fields and Further Mechanics 3.4 Unit 4 Fields and Further Mechanics This isis the first A2 module, building on the key ideas and knowledge covered inin AS This is the first A2 module, building on the key ideas and knowledge covered in AS This the first A2 module, building on the key ideas and knowledge covered AS 3.4 Unit 4 Fields and Further Mechanics 3.4 Unit 4 Fields and Mechanics This is the first A2 unit, building onadvances the key ideas and knowledge covered in and AS Physics. The first section This is the first A2 Further module, building on the key ideas and knowledge covered in AS AS This is the first A2 module, building on the key ideas and knowledge covered in AS physics. The first section the study of momentum introduces circular physics. The first section advances the study of momentum and introduces circular physics. The section advances the study of momentum and introduces circular This is the first A2 module, building on the key ideas and knowledge covered This isstudy the first A2 module, building on the key ideas andmotion knowledge covered in in AS advances the of momentum and introduces circular and oscillatory and covers gravitation. physics. The first section advances the study of momentum and introduces circular This isThe the first A2and module, building on the key ideas and knowledge covered in AS AS and oscillatory motion covers gravitation. Electric and magnetic fields are physics. first section advances the study of momentum and introduces circular and oscillatory motion and covers gravitation. Electric and magnetic fields are and oscillatory motion and covers gravitation. Electric and magnetic fields are physics. The first section advances the study of momentum and introduces circular This is the A2 module, building on the key ideas and knowledge covered in physics. The first section advances the study of momentum and introduces circular Electric and magnetic fields are covered, together with basic electromagnetic induction. Electric fields leads and oscillatory motion and covers gravitation. Electric and magnetic fields are physics. The first section advances the study of momentum and introduces circular and oscillatory motion and covers gravitation. Electric and magnetic fields are covered, together with basic electromagnetic induction. Electric fields lead into covered, together with basic electromagnetic induction. Electric fields lead into covered, together with basic electromagnetic induction. Electric fields lead into and oscillatory motion and covers gravitation. Electric and magnetic fields are physics. The first section advances the study of momentum and introduces circular and oscillatory and covers gravitation. Electric and magnetic fields are into capacitors and how motion quickly they charge and discharge through a resistor. Magnetic fields leads into the covered, together with basic electromagnetic induction. Electric fields lead into and oscillatory motion and covers gravitation. Electric and magnetic fields are capacitors and how quickly they charge and discharge through a resistor. covered, together with basic electromagnetic induction. Electric fields lead into capacitors and how quickly they charge and discharge through a resistor. capacitors and how quickly they charge and discharge through a resistor. covered, together with basic electromagnetic induction. Electric fields lead into and oscillatory motion and covers gravitation. Electric and magnetic fields are generation and transmission of alternating current. covered, together with basic electromagnetic induction. Electric fields lead into capacitors and how quickly they charge and discharge through a resistor. covered, together with basic electromagnetic induction. Electric fields lead into into Magnetic fields lead into the generation and transmission ofof alternating current. capacitors and how quickly they charge and discharge through aa resistor. Magnetic fields lead into the generation and transmission of alternating current. Magnetic fields lead into the generation transmission alternating current. capacitors and how quickly they charge and discharge through a resistor. covered, together with basic electromagnetic induction. Electric fields lead capacitors and how quickly they charge and discharge through resistor. Magnetic fields lead into the generation transmission of alternating current. capacitors and how quickly they charge and discharge through a resistor. Magnetic fields lead into the generation and transmission of alternating current. Magnetic fields lead into the generation transmission of alternating current. capacitors and how quickly they charge and discharge through a resistor. 3.4.1 Further Mechanics Magnetic fields lead into the generation and transmission of alternating current. Further Mechanics 3.4.1 Further Mechanics 3.4.1 Further Mechanics 3.4.1 Magnetic fields lead into the generation and transmission of alternating current. Magnetic fields lead into the generation and transmission of alternating current. Further Mechanics 3.4.1 Further Mechanics 3.4.1 Further Mechanics 3.4.1 Further Mechanics 3.4.1 concepts 3.4.1 Momentum concepts Momentum concepts Momentum Momentum concepts Further Mechanics Further Mechanics 3.4.1 Momentum concepts Momentum concepts as the rate of change ofof momentum Force Force as the rate of change of momentum Momentum concepts Force as the of change momentum Momentum concepts Force as the rate rate of change of momentum Force as the rate of change of momentum momentum Momentum concepts ( ) mv Force as the rate of change of momentum mv ( ) mv Force as the rate of change of Momentum concepts Force as the rate of change of momentum FF == F = ( ) mv Force as the rate of change of momentum ( ) mv t (mv Force as t (tmv ) )the rate of change of momentum F = FF = F = = ( )tt = mv t t Impulse F mv )) ( )= mv Impulse F =( mv Impulse F (mv F= = t tt F Impulse F t = ( (mv mv) ) aa t Impulse F t = ( mv ) tFof Impulse F tarea =( Impulse Significance area under force-time graph. Impulse t = mv ) Significance of area under a force-time force-time graph. Significance of under graph. Impulse F ttarea = (under mv )under Significance of area under a force-time force-time graph. Impulse F = ( mv ) Significance of under a force-time graph. Significance of area a force-time graph. Significance of area a graph. Principle ofof conservation ofof linear momentum applied toto problems inin one Significance of area under a force-time graph. Principle of conservation of linear momentum applied to problems in one Principle conservation linear momentum applied problems one Significance of area area under under amomentum force-time graph. Principle of conservation conservation of linear momentum applied to problems in one one Significance of a force-time graph. Principle of conservation of linear applied to problems in one dimension. Principle of conservation of linear momentum applied to problems in one dimension. dimension. dimension. Principle of of linear momentum applied to problems Principle of conservation of linear momentum applied to problems in in one dimension. Principle of conservation of linear momentum applied applied to to problems problems in in one one Elastic and inelastic collisions; explosions. dimension. Elastic and inelastic collisions; explosions. Elastic and inelastic collisions; explosions. Elastic and inelastic collisions; explosions. dimension. Principle of conservation of linear momentum dimension. Elastic and inelastic collisions; explosions. dimension. Elastic and inelastic collisions; explosions. Elastic and inelastic collisions; explosions. dimension. Elastic and inelastic collisions; explosions. motion Circular motion Circular motion Circular Elastic and inelastic collisions; collisions; explosions. explosions. Circular motion Elastic and inelastic Circular motion Circular motion inin a circular path atat constant speed implies there isis an acceleration and Motion Motion in a circular path at constant speed implies there is an acceleration and Motion a circular path constant speed implies there an acceleration and Circular motion Circular motion Motion in a circular path at constant speed implies there is anthere acceleration and requires a centripetal Motion in a circular path at constant speed implies is an acceleration and Circular motion Motion in a circular path at constant speed implies there is an acceleration and requires a centripetal force. requires a centripetal force. a in centripetal force. Motion a circular path constant speed implies there an acceleration and Circular requires Motion in amotion circular path at at constant speed implies there is is an acceleration and force. requires a centripetal force. Motion in acentripetal circular path at constant constant speed speed implies implies there there is is an an acceleration acceleration and and vv requires aa centripetal v 2force. requires a force. Motion in a circular path at requires centripetal force. Angular speed == = f Angular speed =v =2 2 ff Angular speed = Angular speed v requires a centripetal force. r v= requires a centripetal force. rv= 2 r Angular speed 2f Angular speed = f Angular speed == = 2 Angular speed = f f v r2 2 2 2 r v r vff Angular speed speed = = = =2 2 r v= v 2 2 Angular 2 a= r2 Centripetal acceleration Centripetal acceleration a = = r 2 Centripetal acceleration a = = r Centripetal acceleration 2 2 r v2 v r r v 2 r r a = = 2 r r Centripetal acceleration v 2 2 a = = r Centripetal acceleration a = v= = Centripetal acceleration a= r2 Centripetal acceleration 2 r 2 2 2 r v r mv a2 = = 2 r r Centripetal acceleration r2 mv m mv a = Centripetal acceleration Centripetal force Centripetal force FF == r 2r Centripetal force F = mv 2= =2 m r = Centripetal force = m mv 2 2 2 r r mv 2 r =2 r Centripetal force F mv = =m m 2 r r 2 Centripetal force FF == mm r Centripetal force F = = 2 Centripetal force = be r 2examined. 2 2 mv r2 The derivation of aa = vF / r not r mv The derivation of a= = v /will r will not be examined. The derivation of v / r will not be r 2 r examined. Centripetal force = m 2= r 2 The derivation of aof =F v /r will not be examined. Centripetal force = =m r be examined. 2r The derivation a = v /r r will not The derivation of aof == v2 /2 r/v will not be examined. The derivation a = / will not be examined. r The derivation of a v r will not be examined. harmonic motion Simple harmonic motion Simple harmonic motion Simple The derivation of a= = v22//r r will will not not be be examined. examined. The derivation of a v Simple harmonic motion Simple harmonic motion features of simple harmonic motion. Characteristic Simple harmonic motion Characteristic features of simple harmonic motion. Characteristic features of simple harmonic motion. Simple harmonic motion Simple harmonic motion Characteristic features of simple harmonic motion. Simple harmonic motion 2 2 2 Characteristic features of simple harmonic motion. Characteristic features harmonic motion. Characteristic features of simple motion. Simple harmonic motion Condition for shm: a= ( 2 f )of x simple Condition Characteristic features of harmonic motion. Condition for shm: a = ( 2 xharmonic for shm: a = 2 ffsimple ) x 2 Characteristic features simple harmonic motion. motion. 2 2 x harmonic Condition for shm: a(= = 2 2 Characteristic features of simple Condition for aa == 2 fof x Condition forshm: shm: Condition for shm: a (()2 ff x ))2 x 2 2 Condition for shm: (A 2 f ) 2 2 2 2 x= A cos 2 ft v = 2 f x x= =Condition A cos cos 2 ft ftand and v = 2 f A x x A 2 and v = 2 f A x for shm: shm: a a= = ((2 2ff ))222 x 2 Condition for x= =A A2 cos 2 ft and and v2 = 2 2 f22 A x2 2x 2x x == A cos and == A x f x cos 2 ft v = A Graphical representations linking and t .tt .. x A cos 2ft ft and v v 2f f A x Graphical representations linking xv va a and Graphical representations linking ,, , v ,, a and 2 x, x 2 2 x2 x, v, a and t . x= =A Acos cos 2 ft ft and v v= = 2 2 f A A Graphical representations linking x 2 and f x Graphical representations linking x , v , a and t. Graphical representations linking x , v , a and t . Velocity as gradient of displacement-time graph. Graphical representations linking x , v , a and Velocity as representations gradient of of displacement-time displacement-time graph. Velocity as gradient Graphical linking x, v, a graph. and t .t . Graphical representations linking x , v , a and Velocity as gradient of displacement-time graph. Velocity as gradient of displacement-time Velocity as gradient of displacement-time Graphical representations linking xgraph. , v, graph. agraph. and tt .. Velocity as gradient of displacement-time graph. Velocity as gradient displacement-time 2 fA. Maximum speed == 2 of fA. Maximum speed = 2 fA. Maximum speed Velocity as gradient of displacement-time graph. 2 2 fA. Maximum speed = 2 Maximum speed =2 2 fA Velocity as gradient of displacement-time graph. fA. Maximum speed = Maximum acceleration = (2 f )ff A. 2 fA. Maximum speed = Maximum acceleration = (2 )2 A. 2 Maximum acceleration = (2 ) A. 2 fA. Maximum speed = 2 2(2 2 Maximum acceleration = f ) A. 2 fA. Maximum speed = 2 Maximum acceleration = f) A Maximum acceleration =(2 (2 f ) A. fA. Maximum speed =2 Maximum acceleration = (2 fA. )2A. Maximum acceleration = (2 f ) harmonic systems Simple harmonic systems Simple harmonic systems Simple Maximum acceleration = (2 f ) A. Maximum acceleration = (2f )2A. Simple harmonic systems Simple harmonic systems Study of mass-spring system. Study of mass-spring system. Study of mass-spring system. Simple harmonic systems Simple harmonic systems Simple harmonic systems Study of mass-spring system. Simple harmonic systems Study of mass-spring system. Study of mass-spring system. Simple harmonic systems m Study of mass-spring system. m m Study ofof mass-spring system. TT = 2 Study mass-spring system. T= = 2 m ofm 2 Study m mmass-spring system. k2 T=
k pendulum. m k T of =2 2simple k Study k Study of simple pendulum. Study of simple pendulum. T = k Study of simple pendulum. Study of simple pendulum. k Study simple pendulum. l of Study simple pendulum. ll of TT = 2 T= = 2 lof Study simple pendulum. 2 Study of pendulum. l simple Study g2g g T= lofl simple pendulum. TT == 2 T = 2 2 lg k k TT == 2 T = 2 2 m

lgk T= =2 2g gof E ,k E and total energy with displacement, and with time. Variation of E ,p E and total energy with displacement, and with time. Variation of E E total energy with displacement, and with time. Variation k, p and p T g of E , E and total energy with displacement, and with time. Variation k p g E and total energy with displacement, and with time. Variation of k,kE p E total energy with displacement, and with time. Variation of k,p p and E ,E and total energy with displacement, and with time. Variation of E , E and total energy with displacement, and with time. Variation of k Variation of of Ek,E E totaltotal energy with displacement, and with time. , and Epp and energy with displacement, and with time. Variation kp

14

GCE Physics A Specification for AS exams 2009 onwards and A2 exams 2010 onwards (version 1.1)

A specification for first teaching 2008: version 0.2, and draft submitted to QCA (July 2007) Forced vibrations resonance New GCE Physics AA specification for first teaching 2008: version 0.2, draft submitted toto QCA (July 2007) New GCE Physics specification for first teaching 2008: version 0.2, draft submitted QCA (July 2007)

vibrations and resonance Forced nce and the effects of damping on the sharpness of resonance. Qualitative treatment free and forced vibrations. Qualitative treatment of free and forced vibrations. difference between driver and driven of displacements. 3.4.2 ification for first teaching 2008: Gravitation versionand 0.2, draft submitted to QCA (July 2007) Resonance the effects of damping on the sharpness resonance. Resonance and the effects of damping on the sharpness of resonance. es of these effects in mechanical systems and stationary wave situationsof Phase difference between driver and driven displacements. Phase difference between driver and driven displacements. fication for first teaching 2008: version 0.2, submitted to QCA (July 2007) New GCE Physics A draft specification for first teaching 2008: version 0.2, draft submitted to QCA (July 2007) tion Newton's law Examples of these effects in mechanical systems and stationary wave situations Examples of these effects in mechanical systems and stationary wave situations tions and resonance Gravity as a universal attractive force acting between all matter. ns lawof atment free and forced vibrations. Gravitation 3.4.2 Gravitation 3.4.2 point masses ions resonance asthe a and universal attractive acting between all matter. Forced vibrations resonance nd effects of damping on force thebetween sharpness ofand resonance. Force atment of free and forced vibrations. Newtons law nce between driver and driven displacements. Qualitative treatment of free and forced vibrations. Newtons law Gm m 1 submitted 2 , tion for effects firstpoint teaching 2008: version 0.2,sharpness draft to QCA (July 2007) nd the ofmechanical damping on the of resonance. F = where G is the gravitational constant. etween masses hese effects in systems stationary wave situations Resonance and the effects of damping on the sharpness of resonance. Gravity as aa universal attractive force acting between all matter. Gravity as universal attractive force acting between all matter. 2 and r nce between driver and driven displacements. Phase difference between driver and driven displacements. Gm Gm m g 2008: versionin 0.2, draft submitted to QCA and (July stationary 2007) 1m 2 hese effects mechanical systems wave F =situations where G isis the gravitational constant. Force between masses Examples of these effects in mechanical and stationary wave situations where G ispoint the gravitational constant. F = 2 12 2systems where G the gravitational constant. Force between point masses tional field strength ns and resonance rr w Gravitation 3.4.2 t of of a force field as a region in which a body experiences a force. ment free and forced vibrations. niversal attractive force acting between all field matter. Gravitational strength Gravitational field strength ce effectsby he of damping on the sharpness of resonance. Gravitational field strength entation gravitational field lines. law Gm 1 mNewtons 2 F = where G is the gravitational constant. n point masses forced vibrations. of aF force field as a region in which a body experiences between driver and driven displacements. niversal attractive force acting between all matter. Concept of aa force field as a region in which a body experiences aa force. Concept of force field as a region in which a body experiences force. 2 Concept Gravity as a universal attractive force acting between all matter.a force. r mping on the sharpness of resonance. e effects in mechanical systems and stationary wave situations g ce per unit mass defined by = Gm1 m 2 Representation by gravitational field lines. Representation by gravitational field lines. Gm m Representation by gravitational field lines. 1 2 F= where G is the gravitational constant. n point masses m = where G is the gravitational constant. Force between point masses F and driven displacements. field strength 2 F r2 r F hanical systems and stationary wave situations GM force field as a region in which a body experiences a force. g gg as force per unit mass defined by = as force per unit mass defined ggiven as force per mass definedby by g = ude of g unit g in a radial field by = field strength mm Gravitational field strength on by gravitational field 2 lines. r orce field as a region in which a body experiences a force. ersal attractive force acting between all matter. Concept of a force field as a region in which a body experiences a force. F GM g =Magnitude mass defined by Magnitude of gg gg in a radial field given by == GM of g in agravitational radial field given by nunit by gravitational field lines. Gm m Magnitude of in a radial field given by Representation by field lines. 1 2 tional potential m F= where G is the gravitational constant. oint r2 r2 orcemasses acting between all matter. F F zero r2 GM g unit mass defined by = tanding of the definition of gravitational potential, including value at g g as force per unit mass defined by = g = 2 potential gGm in a radial field given by Gravitational 1m 2 m potential m Gravitational where G is the gravitational constant. Gravitational potential and of gravitational potential difference. ld strength r GM of the r2 GM of the definition of gravitational potential, including zero zero value atvalue infinity, and of Understanding of gravitational including value at Understanding of the definition of gravitational potential, including zero at ein field as a field region in which a experiences a ggiven g ain radial given by = body W =definition m Vforce. one moving mass mUnderstanding Magnitude g in a radial field given by g = 2potential, 2 by of potential gravitational potential difference. r infinity, of gravitational difference. y gravitational field lines. and r and of gravitational potential difference. GM potential g of the definition ofinfinity, gravitational potential, including zero value at on inof which a body experiences a force. Work done in moving mass m given by V V in a radial field given by = ude F W = mm VV Work done in moving mass m given by potential done in moving mass m given by W = Gravitational potential potential g =difference. tgravitational mass defined by Work r eld g of lines. the definition of gravitational potential, including zero value at m Understanding of the definition of gravitational potential, including zero value at GM W = m V moving mass m given by cal representations of variations of and V with r. F VV== VV in ag radial field given by GM Magnitude gravitational potential difference. in a radial field given by Magnitude of GM of infinity, and of gravitational potential difference. g by = GM g ain radial field given by by = Gravitational potential V in a radial field given by r r V V a radial field given by = m V W = m V in moving mass m given Work done moving mass m given by W = m V r2 r g = Graphical d to g by GM representations of variations of gg and V with r. Graphical representations of variations of and GM GM V with r. r by Magnitude resentations of variations of g and V with r. g = field ven V V in by a radial given = tential V V in a radial field given by = of 2 V r Vr of gravitational g including V related to gr by == Vzero value at the potential, g V related to g by gdefinition by = of planets and satellites esentations of variations of g and V with r. Graphical representations of variations of ggand VVwith r Graphical representations of variations of and withr. r. r r avitational potential difference. period and speed related to radius of circular orbit. V V gravitational potential, including zero value at W = m V ving mass m given by g= by satellites g= related by V to g and of planets satellites nets and Orbits of planets and satellites considerations for an orbiting satellite. r Orbits tial difference. r GM and speed related to radius of circular orbit. Orbital and speed related to radius of circular orbit. a radial field by V =period period and speed related to radius of circular orbit. ance of a=geosynchronous orbit. W mgiven V Orbital en by nets and satellites r of derations for an orbiting Orbits planets and satellites satellite. Orbits of planets and satellites Energy considerations for an orbiting satellite. Energy considerations for an orbiting satellite. GM to orbit. and speed related radius of circular orbit. entations of variations of gOrbital and V with r. of a geosynchronous period and speed related to radius of circular orbit. c Fields V= ven by Significance aa geosynchronous Orbital of period and speed related orbit. to radius of circular orbit. Significance of geosynchronous orbit. erations satellite. V for anrorbiting Energy considerations for an orbiting satellite. ds g = Energy considerations for an orbiting satellite. f a geosynchronous orbit. mbs ationslaw of g and V with r. Significance of a geosynchronous orbit. Fields r 3.4.3 Electric Fields 3.4.3 Electric aw Significance of a geosynchronous 1 Q1Q 2 orbit. s Electric Fields 3.4.3 s and satellites where 0 is the etween point in a vacuum F = Coulombs law charges 1 Q1Q Coulombs law 2 4 0 0 is where n speed point charges in radius a vacuum F = orbit. r 2the d related to of circular w 2 Coulombs law 1 QQ 4 r 1Q 22 3.4.3 Electric Fields 0 1Q ations for an orbiting satellite. Q1Q 2 1 point where 0is the Force between charges in aa vacuum FF == 1 1 Q1Q vity of free space. where Force between point charges in vacuum 0 is the 2 22 where is the n point charges in a vacuum F = 0 free space. where 0 is the Force between point charges in a vacuum F = 4 to radius of circular orbit. geosynchronous orbit. r 2 4 0 r 2 4 0 r 4 0 0 r satellite. cting field strength Coulomb's law strength permittivity of free space. permittivity of free space. ree space. permittivity of free space. s orbit. F betweenF Force point charges in a vacuum unit charge by E = field ce per unit defined charge defined bystrength E = strength strength Electric field Electric field strength Electric Q F 1 Q1Q 2 Q F F ,where 0 is the F oint charges in a vacuum F as = per unit charge defined by force EE = on by electric field lines. force per unit charge defined by E E as unit defined by EE == entation by electric field lines. E as force per unit charge defined by 4 =Q r 2 charge Q 0 0 Q Q 1 1 Q1Q 2 Q 1 Q field Evacuum in a radial field given bywhere E = 0 is e space. a F= n by electric lines. is the the permittivity of free space. 2 Representation by electric lines. 2 E in a 4 radial field given by ude of E = 4 r 0 Representation by electric field lines. r 0 Representation lines. 2 1 Q by electric 0 r field 1 Q ength E in a radial field given by Magnitude E = V 2of E in 4 a radial field given by E = 1 1 Q 2Q 4 E E in a uniform field given by = r F 0 4 E in a radial field given by Magnitude of E = E in a radial Magnitude of V field given by E = 0 r 2 2 it charge defined by E= d 4 ude of E in a uniform field by E = 4 V 0 0r r Q given V E= E in a uniform field given by d Magnitude of E in a uniform field given by E = F d V y by electric d E = field lines. Magnitude ofQ EE in aa uniform field given by ==dV Q Magnitude in uniform field given byEE 1 of dd a radial field given by E =
1 Q 4 0 r 2

Qualitative treatment of free and forced vibrations. Resonance and the effects of damping on the sharpness of resonance. vibrations and resonance Phase difference between driver and driven displacements. ive treatment of free and forced vibrations. Examples of these effects in mechanical systems and stationary wave situations. vibrations and resonance Forced

nes.

15

by g = of variations of g and V with r. sentations r Orbits of planets and satellites V g = Orbital periodfor and speed related toand radius of circular orbit. GCE Physics A Specification AS exams 2009 onwards A2 exams 2010 onwards (version 1.1) nets and satellites r Energy considerations for an orbiting satellite. and speed related to radius of circular orbit. s and satellites Significance of a geosynchronous orbit. erations for an orbiting satellite. related to radius of circular orbit. fd aspeed geosynchronous orbit. 3.4.3 Electric Fields ations for an orbiting satellite. sgeosynchronous orbit. Coulombs law 1 Q1Q 2 w where 0 is the Force between point charges in a vacuum F = 1 Q1Q 2 4 0 r 2 where 0 is the n point charges in a vacuum F = permittivity 4 0of free r 2 space. 1 Q 1Q 2 where 0 is the point charges in a vacuumElectric F= ree space. 4field r 2strength Electric strength 0 field F strength e space. E Eas as force force per per unit unit charge charge defined by E = F Q unit charge defined by E = Representation by electric field lines. rength Representation by electric field lines. Q F 1 Q nit charge defined by E = Magnitude of E in a radial field given by n by electric field lines. Magnitude of E in a radial field given by E = Q 1 Q 4 0 r 2 E in a radial field given by E = by electric field lines. 4 0 r 2 V Q of E in a uniform field given by E = 1 0 Magnitude n a radial field given by E Magnitude = d V 2 of E in a uniform field given by E E in a uniform field given by 4 =0 r d V n a uniform field given by E = d 0.2, draft submitted to QCA (July 2007) on for first teaching 2008: version

on for first teaching 0.2, draft submitted to QCA (July 2007) 2008: version Electric potential

definition of absolute electric potential, including zero value at ng 3 charge Q given by W = Q V ctric potential difference. potential Electric Q 1 potential Electric Magnitude of radial field given by by given by = W= Q V V in aof ing charge Understanding definition of absolute electric potential, including zero value at a radial fieldQ given V Understanding of definition of absolute electric potential, including zero value at 4 r 0 and Q 1 infinity, of electric potential difference. on of absolute electric potential, including zero value at a radial field given by V infinity, and of electric potential difference. E=and V with r. ntations of variations of Q given by W = Q V Work done in moving charge 4 r otential difference. Work done 0 0 in moving charge Q given by W = Q 1V Q Graphical representations of variations of E and ectric gravitational fields Qand given by W =of Q V arge E and V with r.in a radial ntations of variations field given by V Vwith =1 r.Q Magnitude of V 4 0 r V in a radial field given by V = Magnitude ofcharacteristics e square law fields having many in common. 1 Q 0 lectric given and gravitational fields al by Vbut = charges 4 0 r Comparison of electric and gravitational fields esfield always attract may attract or repel. Graphical representations of variations of E and V with r. 4 e square law fields having 0 r many characteristics in common. V with r. in common. Graphical representations offields variations of E and Similarities; inverse square law having many characteristics esof always attract but charges or repel. and V with may r. attract ns variations of E Comparison of electric and gravitational fields Differences; masses alwaysand attract but charges may attract or repel. of electric fields Comparison Similarities; inverse squaregravitational law fields having many characteristics in common. and gravitational fields Similarities; inverse square law attract fields having many may characteristics in common. Differences; masses always but charges attract or repel. itance; are law fields having many characteristics in common. 3.4.4 Capacitance Differences; masses always attract but charges may attract or repel. ays attract but charges may attract or repel. 3.4.4 Capacitance citance; Capacitance 3.4.4 Capacitance Capacitance Capacitance Definition of capacitance; Definition of capacitance; a capacitor Definition of capacitance; Q under a graph of charge against Q V and interpretation of area e; CQ = y a capacitor V under a graph of charge against C = of area interpretation 2 Q V and V = Q2/ C Energy stored by a capacitor 2 Energy stored by aQ capacitor stored a capacitor Energy rge = Q2/ C Derivation of by E1 = V and interpretation of area under a graph of charge against pacitor Derivation of and interpretation under a graph of charge pd Derivation E = interpretation of area under a graph of against charge against 2 Q V and ntation of charging and discharging of capacitors through of area p.d. rge interpretation of p.d. and area under a graph of charge against 2 2 1 1 1 2 2 2 2 E= C V = 2 Q V = 2 Q / C 2 C ntation of charging and capacitors E = discharging Q V = Cof V2 = Q2/ C through C, 2 Q/C Capacitor discharge their constants including determination from graphical data, Capacitor discharge discharge Capacitor C, - t/RC Graphical representation of charging and discharging of capacitors through Q = Q e ment of capacitor discharge, o Graphical representation of charging and discharging of capacitors through resistors Graphical representation of charging and discharging of capacitors through e constants including their determination from graphical data, resistors, experience ofresistors, the use of voltage and datalogger n have of charging and discharging ofacapacitors through - t/RC sensor Time constant = RC Q = Q e ment of capacitor discharge, o Time constant = RC, urves for a capacitor. Time constant = RC , sensor Calculation of voltage time constants including their determination from graphical data. d have experience of the use of a and datalogger Calculation of time constants including their determination from graphical data, Calculation of time constants including their determination from graphical data, - t/RC t/RC curves for a capacitor. Quantitative treatment of capacitor discharge Q = Q e Quantitative treatment of capacitor discharge, o t/RC tants including their Quantitative determination from graphical data, discharge, Q = Qo e- o treatment of capacitor Candidates should have experience of the use of a voltage sensor and datalogger nsity Q = Qo e- t/RC f capacitor discharge, Candidates should have experience of the use of a voltage sensor and datalogger to discharge curves for a capacitor. -carrying wire a magnetic field. experience ofin the use of plot adischarge voltage sensor should have and experience of the use of a voltage sensor and datalogger to plot discharge to Candidates plot curves fordatalogger a capacitor. nsity dfor is perpendicular to current. a capacitor. 3.4.5 curves for a capacitor. Magnetic Fields t-carrying wire in a magnetic field. Magnetic Fields 3.4.5 d rule. d is perpendicular to current. Magnetic flux density of the teslaflux ity B and definition Magnetic density d rule. Force on a current-carrying wire in a magnetic field. Force on a current-carrying wire in a magnetic field. naB magnetic field of the and definition tesla sity F = B I l, when field is perpendicular to current. ing wire in a magnetic field. F = B I l, when field is perpendicular to current. particles moving in a magnetic field. Flemings left hand rule. in a magnetic field Flemings rpendicular to current. left hand rule. B and definition of the tesla field is perpendicular to velocity. Magnetic flux density particles moving in a magnetic field. B and definition of the tesla Magnetic flux density rticles; application in devices such as the cyclotron. e field is perpendicular to velocity. and definition of thetesla Moving charges in a magnetic field 16 Moving a magnetic field in a magnetic field. in d flux linkage articles; application devices such asin the cyclotron. Forcecharges on charged particles moving agnetic field Force on charged particles moving in a magnetic field.

New GCE Physics A specification for first teaching 2008: version 0.2, draft submitted to QCA (July 2007) potential difference. definition of absolute potential, including zero value New GCEelectric Physics A specification for first teaching 2008:at version 0.2, draft submitted to QCA (July 2007) Work done in moving charge Q given rst teaching 2008: version 0.2, draft submitted to QCA (July 2007)by tric potential difference.

Understanding of definition of absolute electric potential, including zero value at infinity, and of electric

GCE Physics A Specification for AS exams 2009 onwards and A2 exams 2010 onwards (version 1.1)

3.4.5 Magnetic Fields Magnetic flux density


Force on a current-carrying wire in a magnetic field. F = B I l, when field is perpendicular to current. Fleming's left hand rule. Magnetic flux density B and definition of the tesla.

Moving charges in a magnetic field


Force on charged particles moving in a magnetic field. F = B Q v, when the field is perpendicular to velocity. Circular path of particles; application in devices such as the cyclotron.

Magnetic flux and flux linkage


Magnetic flux defined by F = BA, where B is normal to A. Flux linkage as NF, where N is the number of turns cutting the flux. Flux and flux linkage passing through a rectangular coil rotated in a magnetic field: flux linkage NF = BAN cos where is the angle between the normal to the plane of the coil and the magnetic field.

Electromagnetic induction
Simple experimental phenomena. Faraday's and Lenz's laws. Magnitude of induced emf = rate of change of flux linkage = Applications such as a moving straight conductor. Emf induced in a coil rotating uniformly in a magnetic field:

New GCE Physics specification for first teaching 2008: version 0.2, draftsubmitted submittedto toQCA QCA(July (July2007) 2007) New GCE Physics AA specification for first teaching 2008: version 0.2, draft New GCE Physics A specification for first teaching 2008: version 0.2, draft submitted to QCA (July 2007)

= BAN sin = BAN sin tt = BAN sin tt E The operation of a transformer; The operation transformer; The operation ofof aa transformer; The operation of a transformer; N s VVs The transformer equation =s s V s The transformer equation =N s = N The transformer equation V The transformer equation =p p p NN p NV pp Vp p Transformer efficiency /p I VV Transformer efficiency = Transformer efficiency == IsIs VV /I p s s pp pp Transformer efficiency =transformer. Is Vs / Ip Vp Causes of a Causes ofof inefficiency of atransformer. transformer. Causes ofinefficiency inefficiency of a Causes of inefficiency of a transformer. Transmission of electrical power at high voltage. Transmission electrical power at highvoltage. voltage. Transmission ofof electrical power at high Transmission of electrical power at high voltage.

17

GCE Physics A Specification for AS exams 2009 onwards and A2 exams 2010 onwards (version 1.1)

New GCE Physics A specification for first teaching 2008: version 0.2, draft submitted to to QCA (July 2007) New New GCE GCE Physics Physics AA specification specification for for first first teaching teaching 2008: 2008: version version 0.2, 0.2, draft draft submitted submitted to QCA QCA (July (July 2007) 2007) New GCE Physics A specification for first teaching 2008: version 0.2, draft submitted to QCA (July 2007) New GCE Physics A specification for first teaching 2008: version 0.2, draft submitted to QCA (July 2007)

Unit 55 Nuclear Physics ,Thermal Physics and an Optional Topic Unit Unit 5Nuclear Nuclear Physics Physics ,Thermal ,Thermal Physics Physics and and an an Optional Optional Topic Topic Unit 5 Nuclear Physics ,Thermal Physics and an Optional Topic Optional Topic Unit3.5 53.5 Nuclear Physics ,Thermal Physics and an Optional Topic This module consists ofof two sections. The first part ofof Section AA Nuclear and 3.5 This This module module consists consists oftwo two sections. sections. The The first first part part ofSection Section ANuclear Nuclear and and 3.5 This module consists of two sections. The first part of Section A Nuclear and Thermal Physics looks at the characteristics of the nucleus, the properties of Thermal Thermal Physics Physics looks looks at at the the characteristics characteristics of of the the nucleus, nucleus, the the properties properties of ofat the This unit consists of two sections. The first part of Section A 'Nuclear and Thermal Physics' looks 3.5 This module consists of two sections. The first part Section A Nuclear and of Thermal Physics looks at the characteristics ofof the nucleus, theisproperties characteristics of the nucleus, the properties of unstable nuclei and how energy obtained from the nucleus. unstable nuclei and how energy is obtained from the nucleus. In the second part of unstable unstable nuclei nuclei and and how how energy energy is is obtained obtained from from the the nucleus. nucleus. In In the the second second part part of of Thermal Physics looks athow the characteristics of of the nucleus, the properties of nature unstable nuclei and energy is obtained from the and nucleus. In the and second part of are In the second part of Section A, the thermal properties materials the properties of gases section A, the thermal properties of materials and the properties and nature of section section A, A, the the thermal thermal properties properties of of materials materials and and the the properties properties and and nature nature of of unstable andthermal how energy is obtained from the nucleus. In the second part of of section A, the properties of materials and the properties and nature studied innuclei depth. gases are studied inin depth. gases gases are are studied studied indepth. depth. section A, the thermal properties of materials and the properties and nature of gases are studied in depth. Section B offers an opportunity to study one of the one following optional topicsoptional to gain deeper understanding Section BB offers opportunity toto study ofof the following topics toto gain Section Section B offers offers an an opportunity opportunity to study study one one of the the following following optional optional topics topics togain gain and gases are studied in an depth. Section Bselected offers an opportunity to study one of the following optional topics to gain awareness of a branch of physics: deeper understanding and awareness ofof a selected branch ofof physics; deeper deeper understanding understanding and and awareness awareness ofthe a aselected selected branch branch ofphysics; physics; Section B offers an opportunity study one of following optional topics to gain deeper understanding andto awareness of a selected branch of physics; A A Astronomy and cosmology Astronomy and cosmology, A A Astronomy Astronomy and and cosmology, cosmology, deeper understanding and awareness of a selected branch of physics; A Astronomy and cosmology, Medical Physics, Medical Physics B. B. Astronomy Medical Medical Physics, Physics, AB B. and cosmology, B. Medical Physics, C Applied Physics, C C Applied Applied Physics, Physics, C Applied Physics B. Medical Physics, C Applied Physics, Turning Points inin Physics. D D Applied Turning Turning Points Points inPhysics. Physics. CD D D Turning Points in Physics. Turning Points in Physics, Physics. D Turning Points in Physics. Nuclear and Thermal Physics Nuclear Nuclear and and Thermal Thermal Physics Physics Nuclear and Thermal Physics 3 Nuclear and Thermal Physics Nuclear and Radioactivity Thermal Physics 3.5.1 Radioactivity Radioactivity 3.5.1 3.5.1 3.5.1 3.5.1 Radioactivity Radioactivity 3.5.1 Radioactivity for the nucleus Evidence Evidence for for the the nucleus nucleus Evidence Evidence for the nucleus Qualitative study of Rutherford scattering. Qualitative Qualitative study study of of Rutherford Rutherford scattering. scattering. Evidence for the nucleus Evidence for the nucleus Qualitative study of Rutherford scattering. Qualitative of Rutherford scattering. study of Rutherford scattering. Qualitative and radiation , ,, and and study radiation radiation , and radiation properties and experimental identification using simple absorption Their Their properties properties and and experimental experimental identification identification using using simple simple absorption absorption , Their and Their properties and experimental identification using simple absorption , radiation and applications radiation experiments; e.g. to relative hazards of exposure to humans. experiments; experiments; applications applications e.g. e.g. to to relative relative hazards hazards of of exposure exposure to tohumans. humans. Theirexperiments; properties and experimental identification using simple absorption applications e.g. to relative using hazards of absorption exposure to humans. Their properties and experimental identification simple experiments; applications e.g. k k k experiments; applications e.g. to relative hazards to humans. k of The inverse square for to radiation, I= , including its experimental The The inverse inverse square square law law for for radiation, radiation, I I==2 ,exposure ,including including its its experimental experimental to relative hazards of law exposure humans. 2 2 The inverse square law for radiation,kI = x x x , including its experimental x2 The inverse square law for radiation, its experimental The inverse square law for radiation, I = 2 , including verification; applications, e.g. toto safe handling ofof radioactive sources. verification; verification; applications, applications, e.g. e.g. tosafe safe handling handling ofradioactive radioactive sources. sources. x verification; applications, e.g. to safe handling of radioactive sources. Background radiation; examples of its origins and experimental elimination from including its experimental verification; applications, e.g. to safe handling of radioactive sources. Background Background radiation; radiation; examples examples of of its its origins origins and and experimental experimental elimination elimination from from verification; applications, e.g. to safe handling of radioactive sources. Background radiation; examples of its origins and experimental elimination from calculations. calculations. calculations. Background radiation; examples ofits its origins origins and experimental elimination from calculations. Background radiation; examples of and experimental elimination from calculations. decay Radioactive Radioactive decay decay calculations. Radioactive Radioactive decay Random Radioactive decay nature ofof radioactive decay; constant decay probability ofof a given Random Random nature nature ofradioactive radioactive decay; decay; constant constant decay decay probability probability ofa agiven given Radioactive decay Random nature of radioactive decay; constant decay probability of a given Random nature of radioactive decay; constant decay probability of a given nucleus; nucleus; nucleus; nucleus; Random nature of radioactive decay; constant decay probability of a given nucleus; N N N - t-- nucleus; N= = -= N ,N N = N e - , ,N N = = N N e et t t o 0 - oo = N , N = N e o t tt N t N, N = No e- t = Use Use of activity A = N Use of activity Use of of activity activity A A = = N N t Use of activity A = N ln 2 ln ln 22 Use Half of activity AT = ln N 2; determination T1/2 = from graphical decay data life, T ; ;determination determination from from graphical graphical decay decay data data Half Half life, life, 1/2 1/2== = determination from graphical decay data Half Half life, life, T1/2 from ln 2 ;; determination T1/2 =decay ; curves determination from graphical decay data Half including life, and log graphs; applications e.g. relevance toto storage ofof including including decay decay curves curves and and log log graphs; graphs; applications applications e.g. e.g. relevance relevance tostorage storage of including decay curves anddecay log graphs; applications e.g. relevance to storage of graphical decay data including curves and log graphs; applications e.g. relevance to storage of radioactive waste, radioactive dating. radioactive radioactive waste, waste, radioactive radioactive dating. dating. including decay waste, curves and log graphs; applications e.g. relevance to storage of radioactive radioactive dating. radioactive waste, radioactive dating. waste, radioactive dating. instability Nuclear Nuclear instability instability radioactive Nuclear Nuclear instability Graph of N against ZZ for stable nuclei. Graph Graph of of N N against against Zfor for stable stable nuclei. nuclei. Nuclear instability Nuclear instability Graph of N against Z for stable nuclei. + ++ - -Possible modes ofof unstable including , ,+ and electron capture. Possible Possible decay decay modes modes of unstable unstable nuclei nuclei including including ,, ,, and and electron electron capture. capture. Graph ofdecay N against Z for stable nuclei. nuclei Graph of N against Z for stable nuclei. Possible decay modes of unstable nuclei including , , and electron capture. + and Changes of N and Z caused by radioactive decay in simple + -representation Changes Changes of of N N and and Z Z caused caused by by radioactive radioactive decay decay and and representation representation in in simple simple Possible decay modes of unstable nuclei including , , and electron capture. Possible decayof modes unstable nuclei including , and , and electron capture. Changes N andof Z caused by radioactive decay representation in simple decay equations. decay decay equations. equations. Changes of N and Z caused by radioactive decay and representation in simple decay equations. Changes of N and Z caused by radioactive decay and representation in simple decay equations. of nuclear excited states; ray emission; application e.g. use of technetium-99m as a decay Existence equations. Existence ofof nuclear excited states; ray emission; application e.g. use ofof Existence Existence ofnuclear nuclear excited excited states; states; ray ray emission; emission; application application e.g. e.g. use use of source in medical diagnosis. Existence of nuclear excited states; ray emission; application e.g. use of technetium-99m as a source in medical diagnosis. technetium-99m technetium-99m as as a a source source in in medical medical diagnosis. diagnosis. Existence of nuclear excited states; in ray emission; application e.g. use of technetium-99m as a source medical diagnosis. technetium-99m as a source in medical diagnosis.

3.5 Unit 5 PHA5A-5D Nuclear Physics,Thermal Physics and an

18

GCE Physics A Specification for AS exams 2009 onwards and A2 exams 2010 onwards (version 1.1)

Physics A specification for first teaching 2008: version 0.2, draft submitted to QCA (July 2007)

Nuclear radius Estimate of radius from closest approach of alpha particles and determination of radius from electron Estimate of radius from closest approach of alpha particles and determination of diffraction; knowledge of typical values. radius from electron diffraction; knowledge of typical values. Dependence of radius on nucleon number Dependence of radius on nucleon number 1/3 rom closest approach of alpha particles and determination of R= ro 0 A nderived diffraction; knowledge of typical values. from experimental data. derived from experimental data. us on nucleon number Calculation of nuclear density. Calculation of nuclear density. R = ro A1/3 Nuclear Energy mental data. 3.5.2 Nuclear Energy ar density. Mass and energy 2 Appreciation that applies to all energy changes. E = mc Mass and energy Simple calculations on mass difference binding Appreciation that E =and mc2 applies toenergy. all energy changes. Atomic mass unit, u; Conversion of units; 1 u = 931.3 MeV. Simple calculations on mass difference and binding energy. = mc2 applies to allbinding energy energy changes. Graph of average per nucleon against nucleon number. Atomic massenergy. unit, u; conversion of units; on mass difference and binding Fission and fusion processes. ; Conversion of units; 1 u nuclear = 931.3 MeV. Simple calculations from masses of energy released in fission and fusion inding energy per nucleon against nucleon reactions. Graph of average bindingnumber. energy per nucleon against nucleon number. processes. Fission and fusion processes. Induced fission from nuclear masses Simple of energy released in fission and fusion calculations from nuclear ofreaction; energy released in mass. fission and fusion reactions. Induced fission by thermal neutrons; possibility of masses a chain critical The functions of the moderator, the control rods and the coolant in a thermal factors Induced fission nuclear reactor; affecting the choice of materials for the moderator, the hermal possibility of a chain reaction; critical mass. Induced fission by thermal neutrons; possibility a chain reaction; critical mass. control neutrons; rods and the coolant and examples of materials used; of details of particular moderator, rods and the coolant in a the thermal The functions of the moderator, control rods and the coolant in a thermal nuclear reactor; factors reactors are the not control required. affecting the choice of materials for the moderator, the control rods and the coolant and examples of tors affecting the choice of materials for the moderator, the aspects materials used; details particular are not required. eSafety coolant and examples of materials used;of details of reactors particular Fuel used, shielding, emergency shut-down. uired. Production, handling and storage of radioactive waste materials. Safety aspects

version Nuclear radius or first teaching 2008: 0.2, draft submitted to QCA (July 2007)

Fuel used, shielding, emergency shut-down. Thermal Physics g, emergency shut-down. Production, handling and storage of radioactive waste materials. g and storage of radioactive waste materials. Thermal energy Calculations involving change of energy. 3.5.3 Thermal Physics For a change of temperature; Q = m c where c is specific heat capacity. For a change of Q = m l where l is specific latent heat. state;Thermal energy ng change of energy. Ideal gases involving change of energy. mperature; Q = m c Calculations where c is specific heat capacity. Gas laws as experimental relationships between p,mc V, T mass. For a change of temperature; Q = T,and where c is specific heat capacity. te; Q = m l where l is specific latent heat. Concept of absolute zero of temperature. For a change of state; Q = m l, where l is specific latent heat. Ideal gas equation as pV = nRT for n moles and as pV = NkT mental relationships p, V, T and mass. for N molecules. between Ideal gases e zero of temperature. Avogadro constant NAGas , molar constant R , Boltzmann constant . and mass. lawsgas as experimental relationships between p, V,kT as pV = nRT for n molecular moles andmass. as pV = NkT Molar mass and Concept of absolute zero of temperature.

Molecular kinetic theory model Ideal gas equation as pV = nRT for n moles and as pV = NkT for N molecules. N A, molar gas constant R, Boltzmann constant k. Explanation of relationships between ,A V and T in constant terms ofRa simple molecular Avogadro constantpN , molar gas , Boltzmann constant k. lecular mass. model. Molar mass and molecular mass. heory model 1 = molecular N m c2rms Assumptions leading and ionships between p, Vto and T derivation in terms ofof a pV simple 3

2 1 kT = 2 m c rms = . Average molecular of kinetic pV = energy N m c2 g to and derivation rms 2 2 NA 3

3RT

kinetic energy

1 3 3RT 2 = kT = m c rms . 2 2 2 NA

19

GCE Physics A Specification for AS exams 2009 onwards and A2 exams 2010 onwards (version 1.1)

Molecular kinetic theory model


Explanation of relationships between p, V and T in terms of a simple molecular model. Assumptions leading to and derivation of

Average molecular kinetic energy

20

GCE Physics A Specification for AS exams 2009 onwards and A2 exams 2010 onwards (version 1.1)

New GCE Physics A specification for first teaching 2008: version 0.2, draft submitted to QCA (July 2007) New GCE Physics Physics A specification specification for first first teaching 2008: 2008: version 0.2, 0.2, draft submitted submitted to QCA QCA (July 2007) 2007) New New GCE GCE Physics A A specification for for first teaching teaching 2008: version version 0.2, draft draft submitted to to QCA (July (July 2007)

3.5 3.5 3.5 3.5 3.5

Options Options Options Unit 5A Astrophysics Unit 5A 5A Astrophysics Astrophysics Unit Unit 5A Astrophysics Unit 5A InAstrophysics this option, fundamental physical principles are applied to the study and In this this option, option, fundamental fundamental physical principles aredeeper applied to the study study and interpretation of the Universe. Students will gain insight behaviour In physical principles are applied to and In this option, fundamental physical principles are applied to the study and interpretation ofthe the Universe. In this option, fundamental physical principles are applied to the the into study and interpretation of the Universe. Students will gain deeper insight into the behaviour of objects at great distances from Earth and discover the ways in which information interpretation of the Universe. Students will gain deeper insight into the behaviour Candidates will gain deeper insight into the behaviour of objects at great distances from Earth and discover the interpretation of the Universe. Students will gain deeper insight into the behaviour of objects at great distances from Earth and discover the ways ways in which which information ways in which information from these objects can be gathered. The underlying physical principles of the optical from these objects can be gathered. The underlying physical principles of the of objects at great distances from Earth and discover the in information of objects at great distances from Earth and discover the ways in which information from these objects can be gathered. The underlying physical principles of the and other devices used are covered and some indication is given of the new information gained by the use of optical andobjects other devices are covered and some indication given of from these can gathered. The physical principles of the from these objects can be beused gathered. The underlying underlying physical principles ofthe thenew radio astronomy. Details of particular sources and their mechanisms are not required. optical and other devices used are covered and some indication given of the new information gained by theused use of radio astronomy. Details of particular sources optical and devices are covered and indication given the optical and other other devices used are covered and some some indication given of of the new new information gained by by the the use of radio astronomy. astronomy. Details Details of of particular particular sources sources and their mechanisms are not required. information gained use of radio information gained by the use of radio astronomy. Details of particular sources A.1.1 and Lenses and Optical Telescopes and their their mechanisms are not required. required. mechanisms are theirand mechanisms are not not required. Lenses Optical Telescopes A.1.1 and Lenses and and Optical Optical Telescopes Telescopes A.1.1 Lenses A.1.1 Lenses A.1.1 Lenses and Optical Telescopes Lenses Principal focus, focal length of converging lens. Principal focus, focal length of converging lens. Lenses Lenses Principal focus, focal length of converging lens. Formation of images by a converging lens. Formation of images by a converging lens. Principal Principal focus, focus, focal focal length length of of converging converging lens. lens. Formation of images by a converging lens. Ray diagrams. Ray diagrams. Formation of Formation of images images by by a a converging converging lens. lens. Ray diagrams. Ray diagrams. 1 1 1 Ray diagrams. 1 +1 1 =1 1 1 1+ u v= f +1 =1 u +v v = ff u u v f telescope consisting Astronomical telescope Astronomical consisting of of two two converging converging lenses lenses telescope consisting of two converging lenses Astronomical Ray diagram to show the image formation in normal adjustment. telescope consisting of two converging lenses telescope consisting of two converging lenses Astronomical Astronomical Ray diagram to show the image formation in normal adjustment. Ray diagram to show the image formation in normal adjustment. Angular magnification in normal adjustment. Ray diagram to the image formation in Ray diagram to show show the image formation in normal normal adjustment. adjustment. Angular magnification in normal adjustment. Angular magnification in normal adjustment. Angular magnification in normal angle subtended by image adjustment. at eye Angular magnification in normal adjustment. M= anglesubtended subtendedby by imageat at eye angle angle subtended by image image at eye eye eye subtended by object at unaided M= = angle M M = angle subtended by object at unaided eye angle subtended by object at unaided angle subtended object at unaided eye eye Focal lengths of theby lenses. Focal lengths of the lenses. Focal lengths of the lenses. Focal lengths of the lenses. f Focal lengths of the lenses. M = fo o f f o f M = oe M M= = fe f fe e telescopes Reflecting Reflecting telescopes Focal point of concave mirror. Reflecting telescopes telescopes Reflecting telescopes Focal point of concave mirror. mirror. Cassegrain arrangement using a parabolic concave primary mirror and convex Focal point of concave Focal point concave mirror. Focal point of of concave mirror. Cassegrain arrangement usingto a show parabolic concave primary the mirror and convex secondary mirror , ray diagram path of rays through telescope as far as Cassegrain arrangement using a concave primary mirror and Cassegrain arrangement using a parabolic parabolic concave primary mirror and convex convex Cassegrain arrangement using a parabolic concave primary mirror and convex secondary mirror ray secondary mirror , ray diagram to show path of rays through the telescope as far ,as the eyepiece. secondary mirror , diagram to show path of rays through the secondary mirror , ray ray diagram to show path as of far rays through the telescope telescope as as far far as as diagram to show path of rays through the telescope as the eyepiece. the eyepiece. Relative merits of reflectors and refractors including a qualitative treatment of the eyepiece. the eyepiece. Relative merits of reflectors and refractors including including a qualitative of spherical andof chromatic Relative merits of reflectors and refractors atreatment qualitative treatment spherical and chromatic aberration. Relative merits of and Relative merits of reflectors reflectors and refractors refractors including including a a qualitative qualitative treatment treatment of of aberration. spherical and and chromatic chromatic aberration. spherical and chromatic aberration. aberration. Resolving power spherical Resolving power Resolving Resolving power Appreciation of diffraction pattern produced by circular aperture. power power Resolving Appreciation of diffraction pattern produced by circular circular aperture. Resolving power of telescope, Rayleigh criterion, Appreciation of of diffraction pattern produced by circular Appreciation pattern produced by Appreciation of diffraction diffraction pattern produced byaperture. circular aperture. aperture. Resolving power of telescope, Rayleigh criterion, Resolving power of telescope, Rayleigh criterion, Resolving power of telescope, Rayleigh criterion, Resolving power of telescope, Rayleigh criterion, D D D D coupled device Charge Charge coupled device Use of CCD to capture images. coupled device Charge Charge coupled device Use Charge coupled device Use of CCD to capture images. Structure and operation of the charge coupled device: of CCD to capture images. Use of CCD to capture images. Structure and operation of the charge coupled device: Use of CCD to capture images. A CCD is silicon chip divided picture elements (pixels). Structure and operation of charge coupled device: Structure and operation of the theinto charge coupled device: A CCD is silicon chip divided into picture elements (pixels). Structure and operation of the charge coupled device: Incident photons cause electrons to be released. A CCD is silicon chip divided into picture elements (pixels). A CCD is silicon chip divided into picture elements (pixels). Incident photons cause electrons to be released. A CCD is a silicon divided into picture (pixels). The number of chip electrons liberated iselements proportional to the intensity of the light. Incident photons cause electrons to be released. Incident photons cause electrons to be released. The number of electrons liberated is proportional to the intensity of the the light. These electrons areelectrons trapped in potential wells into the CCD. The number of electrons liberated is the intensity Incident photons to be released. The number ofcause electrons liberated is proportional proportional to the intensity of of the light. light. These electrons are trapped in potential wells in the CCD. An number electron built up which is identical tothe theCCD. image formed on the CCD. These electrons are trapped in potential wells in The of pattern electrons liberated is proportional to the intensity of the light. These electrons areis trapped in potential wells in the CCD. An electron pattern is built up up which which is identical to the image image formed on the CCD. CCD. When exposure complete, charge processed to giveformed an image. An electron pattern is is to on An electron pattern is built built up the which is identical identical to the the image formed on the the CCD. These electrons are is trapped in 'potential wells' inis the CCD. When exposure is complete, the charge is processed to give an image. Quantum efficiency of up pixel >the 70%. When exposure is complete, charge is processed to an image. When exposure is built complete, the chargeto isthe processed to give give anCCD. image. An electron pattern is which is identical image formed on the Quantum efficiency of pixel pixel > 70%. 70%. Quantum efficiency of > Quantum efficiency of pixel > 70%.
21

Options Options

GCE Physics A Specification for AS exams 2009 onwards and A2 exams 2010 onwards (version 1.1)

When exposure is complete, the charge is processed to give an image. Quantum efficiency of pixel > 70%.
New GCE Physics A specification for first teaching 2008: version 0.2, draft submitted to QCA (July 2007) A.1.2 Telescopes New GCENon-optical Physics A specification for first teaching 2008: version 0.2, draft submitted to QCA (July 2007) New GCE Physics A specification for first teaching 2008: version 0.2, draft submitted to QCA (July 2007)

Non-optical Telescopes Similarities and differences compared to optical telescopes including structure, positioning and use, Non-optical Telescopes including comparisons of resolving and collecting powers. Non-optical Telescopes Single dish radio telescopes, I-R, U-V and X-ray telescopes Single dish radio telescopes, I-R, and X-ray telescopes Single dishand radio telescopes, I-R,U-V U-V X-ray telescopes Similarities differences compared toand optical telescopes including structure, A.1.3 Similarities Classification of Stars compared to optical telescopes including structure, and differences Similarities compared to optical telescopes including structure, positioning and and differences use, including comparisons of resolving and collecting powers. positioning positioningand anduse, use,including includingcomparisons comparisonsof ofresolving resolvingand andcollecting collectingpowers. powers. Classificationby of luminosity Stars A.1.3 Classification Classification of Stars A.1.3 Classification of Stars A.1.3 Relation between brightness and apparent magnitude. by luminosity Classification by luminosity Classification Classification by brightness luminosityand apparent magnitude. Relation between Relation brightness Apparent magnitude, m and Relationbetween between brightness andapparent apparentmagnitude. magnitude. Apparent magnitude, m apparent magnitude. Relation between intensity and magnitude, m Apparent Apparent magnitude, m and apparent Relation between intensity magnitude. Measurement of m from photographic plates andmagnitude. distinction between photographic and visual Relation between intensity and apparent Relation between intensity and apparent magnitude. Measurement of m from photographic plates and distinction between photographic magnitude not required. Measurement of photographic plates and distinction between photographic Measurement ofm mfrom from photographic and visual magnitude not required. plates and distinction between photographic and visual magnitude not required. and visualmagnitude, magnitude not required. Absolute Absolute magnitude, M M Absolute magnitude, Parsec and light year.year. M Absolute magnitude, M Parsec and light Parsec and light year. Definition of M ,light relation to m to m M, relation Definition of Parsec and year. M , relation to m Definition of Definition of M, drelation to m m M = 5 log dd m 10 m M M= =5 5log log 10 10 Classificationby bytemperature, temperature, black black body Classification Classification body radiation radiation by temperature, black Classification byWiens temperature, blackbody bodyradiation radiation Stefans law and displacement law. Stefan's law and Wien's displacement law. law. Stefans law and Wiens displacement Stefans and Wiens displacement law. General law shape of black body curves, experimental verification is not required. General shape of black body curves, experimental verification is not required. General shape of black body curves, experimental verification is General shape displacement of black body law curves, experimental verification isnot notrequired. required. Use of Wiens to estimate black-body temperature of sources Use of Wiens displacement law to estimate black-body temperature of Use of Wien's displacement law to estimate black-body temperature of sources -3 Use of displacement law to estimate black-body temperature ofsources sources maxT =Wiens constant = 2.9 10 -3 mK. T = constant = 2.9 10 mK. max = 2.9 10-3 mK. in its application. Inverse square law, assumptions maxT = constant Inverse square law, assumptions in application. Inverse square law, inits its application. Use of Stefans law assumptions to estimate area needed for sources to have same power Inverse square law,law assumptions in itsarea application. Use of Stefans to estimate needed for sources to Use of Stefans law to estimate area needed for sources tohave havesame samepower power output as the sun. Use of Stefan's law to estimate area needed for sources to have same power output as the Sun. output as the sun. 4 the sun. output as P = AT P = AT 44 P = AT Assumption that a star is a black body. Assumption that a Assumption that a star is a is black body. body. Assumption that astar star isa ablack black body. Principles of the use of stellar spectral classes of the use of stellar spectral Principles Principles of the use of stellar spectralclasses classes Description ofthe the use main classes: Principles of of stellar spectral classes Description of the main classes: Description of the main classes: Spectral of the Intrinsic Prominent Description main classes: Temperature (K) Spectral Intrinsic Temperature Prominent Spectral Intrinsic Temperature(K) (K) Prominent Class Colour Absorption Lines Class Colour Absorption Lines Class Colour Intrinsic Colour Absorption Lines Spectral Class Temperature (K) Prominent Absorption Lines O blue 25 000 50 000 He+, He, H O blue 25 He+, He, O blue 25000 000 50 50000 000 He+, He,H H blue 25 000 50 000 He+, He, H B O blue 11 000 25 000 He, H B blue 11 000 25 000 He, H B blue 11 000 25 000 He, H B blue 11 000 25 000 He, H A blue-white 7 500 11 000 H (strongest) A blue-white 7 500 11 000 H (strongest) A blue-white blue-white 7 500 11 H (strongest) ionised metals A 7 500 11 000 H 000 (strongest) ionized metals ionized metals F White 6 000 7 500 ionized metals ionised metals F White 6 000 7 500 ionized metals F White 6 000 7 500 ionized G yellow-white 5 000 6 000 metals ionised & neutral metals F White 6 000 7 500 ionized metals G K yellow-white orange 5 000 6 0003 500 ionized & neutral 5 000 & neutral neutral metals G yellow-white 5 ionized G yellow-white 5000 000 6 6 000 000 ionized & neutral metals M red < 3 500 metals neutral atoms, TiO metals K orange 3 500 5 000 neutral metals Temperature related to absorption spectra limited to Hydrogen Balmer absorption lines: need for atoms K 3 neutral in n = K2 state. orange orange 3500 500 5 5000 000 neutralmetals metals M red < 3 500 neutral atoms, TiO M red < neutral M red <3 3500 500 neutralatoms, atoms,TiO TiO Temperature related to absorption spectra limited to Hydrogen Balmer absorption 22 Temperature related to absorption spectra Temperature related spectralimited limitedto toHydrogen HydrogenBalmer Balmerabsorption absorption n = 2 state. lines: need for atomsto inabsorption n = 2 state. lines: need for atoms in lines: need for atoms in n = 2 state. A.1.2 A.1.2 A.1.2

Single dish radio telescopes, I-R, U-V and X-ray telescopes

GCE Physics A Specification for AS exams 2009 onwards and A2 exams 2010 onwards (version 1.1)

GCE Physics New GCE Physics A specification for first teaching 2008: version 0.2, draft submitted New to QCA (July 2007)A specification for first teach New GCE Physics A specification for first teaching 2008: version 0.2, draft submitted to QCA (July 2007)

The Hertzsprung-Russell diag General shape: main sequence General shape: main sequence, dwarfs and giants. shape: main sequence, dwarfs and giants. The Hertzsprung-Russell diagram General Axis scales range from 15 to 1 Axis scales range from 15 to 10 (absolute magnitude) and Axis scales shape: range from 15sequence, to 10 (absolute magnitude) and 50 000 K to 2 500 K (temperature) or General main dwarfs and giants. 50 000 K to 2 500 K (temperatu 50 000 K to 2 500 K (temperature) or OBAFGKM (spectral class). OBAFGKM (spectral class). Axis scales range from to 10 (absolute Stellar evolution: path of a star Stellar evolution: path of15 a star similar to ourmagnitude) Sun on the and Hertzsprung-Russell Stellar evolution: star similar to our on the Hertzsprung-Russell diagram from formation 50 000 K to 2 path 500 of Ka (temperature) orSun OBAFGKM (spectral class). diagram from formation to white diagram from formation to white dwarf. to white dwarf. Stellar evolution: path of a star similar to our Sun on the Hertzsprung-Russell stars anddwarf. black holes Supernovae, neutron stars an Supernovae, diagram from neutron formation to white Supernovae, neutron stars and black holesmagnitude of supernovae; Defining properties: rapid increa Defining properties: rapid increase in absolute Supernovae, neutron stars and black holes Defining composition and density of neu composition and density of neutron stars; escape > ccomposition for black holes. properties: rapid increase in absolute magnitude of velocity supernovae; and density of Defining properties: rapid in absolute magnitude of supernovae; neutron escape velocity > increase c for black holes. Use of supernovae as standard Use ofstars; supernovae as standard candles to determine distances. Controversy composition and of neutron stars; velocity > c forconcerning black holes. concerning accelerating Univer Use of supernovae asdensity standard candles to determine distances. Controversy accelerating concerning accelerating Universe and darkescape energy. Use of supernovae as standard candles to determine distances. Controversy Universe and dark energy. Supermassive black holes at th Supermassive black holes at the centre of galaxies. concerning accelerating Universe and dark energy. Supermassive black holes at the of galaxies. Calculation of the radius of the Calculation of the radius of centre the event horizon for a black hole Schwarzschild radius black holes at the centre of galaxies. (Supermassive Rs ) Calculation of the radius of the event horizon for a black hole Schwarzschild radius ( Rs ) Calculation of the radius of the event horizon for a black hole Schwarzschild radius 2GM ( R= ) 2GM Rs 2 R s s 2 c 3 2c GM Rs 2 A.1.4 Cosmology A.1.4 Cosmology c The Hertzsprung-Russell Hertzsprung-Russell diagram The diagram

A.1.4 Cosmology

effect Doppler effect Doppler Cosmology A.1.4 f v v f effect v v Doppler z= = = and = and = = effect zDoppler c f c cv f f cv z = and = = v c applied to optical and For to For v f c applied optical c and radio frequencies. c on binary stars vie Calculations on binary stars viewed in the plane of orbit, galaxies and Calculations quasars. For v << c applied to optical and radio v c applied to optical and frequencies. radio frequencies. For Hubbles law Calculations on binary stars viewed in the plane of orbit, galaxies andgalaxies quasars. Calculations on binary stars viewed in the plane of orbit, andHubbles quasars. law Red shift Red shift Hubbles law v = Hd v = Hd Hubble's law Red shift Simple interpretation as expans Simple interpretation as expansion of universe; estimation of age of universe,
Red shift v = Hd

assuming H is constant. assuming H is constant. v = Hd interpretation Simple as expansion of universe; estimation of age of universe, Qualitative treatment of Big Ban Qualitative treatment of Big Bang theory including fromassuming cosmological Simple interpretation as expansion of universe; estimation of evidence age of universe, H is constant. assuming H is constant.
Qualitative treatment of Bigof Bang evidence from cosmological background Qualitative treatment Bigtheory Bangincluding theory including evidence from microwave cosmological radiation, and relative abundance of H and He. Quasars Quasars microwave background radiation, and relative abundance of H and He.

background radiatio . microwave background radiation, and relative abundance of H and Hemicrowave

Quasars as bright radio sources. Quasars Discovery

Quasars as the most distant measurable objects.

Quasars as the most distant measurable objects.of distance. Quasars asshow the most distant measurable objects. Quasars large optical red shifts; estimation Discovery as bright radio sources. Discovery as bright radio sources. Quasars show large optical red shifts; estimation of distance. Quasars show large optical red shifts; estimation of distance.

Quasars as the most distant me Discovery as bright radio sourc Quasars show large optical red

23

Centre Number Surname Other Names Candidate Signature

Candidate Number

General Certificate of Education Advanced Level Examination Sample paper 1 2010

Physics A
Unit 4 Section A Fields and Further Mechanics Sample paper 1
For this paper you must have: An objective test answer sheet a calculator a Black ink or black-point pen a Data and Formulae Booklet

PHYA4/1

Time allowed The total time for both sections of this paper is 1 hour 45 minutes You are advised to spend approximately 45 minutes on this section. Instructions Use black ink or black ball-point pen. Do not use pencil Fill in the boxes at the top of this page. Answer all questions in this section For each question there are four responses. When you have selected the response which you think is the most appropriate answer to a question, mark the response on your answer sheet. Mark all responses as instructed on your answer sheet. If you wish to change your answer to a question, follow the instructions on your answer sheet. Do all rough work in this book not on the answer sheet Information The maximum mark for this section is 25 Section A and Section B of this paper together carry 20% of the total marks for Physics Advanced. You are expected to use a calculator where appropriate. A Data and Formulae Booklet is provided as a loose insert. - The question paper/answer book for Section B is enclosed within this question paper.

PHYA4/1

Teacher Resource Bank / GCE Physics A / PHYA4 Section A / Sample Question Paper / Version 1.0

AQA Physics Specification A PHYA4, Section A, Sample Question Paper


Multiple choice questions Each of Questions 1 to 30 is followed by four responses, A, B, C, and D. For each question select the best response and mark its letter on the answer sheet.

When a tennis player hits a returning ball of known mass with a racket, the force, F, exerted on it at time t can be represented by the following graph. F

0 0

Which one of the following cannot be obtained from the graph? A B C D the change in momentum of the ball the impulse given to the ball the speed of the ball as it leaves the racket the acceleration of the ball at any given time

In a vehicle impact test, a car of mass 1200 kg travelling at a velocity of 18 m s-1 is stopped by a large concrete block. A forcemeter attached to the block is used to measure the average force of the impact.

The forcemeter measured an average force of 240 kN. What was the duration of the impact? A B C D 0.090 s 0.18 s 0.90 s 1.8 s

Copyright 2009 AQA and its licensors. All rights reserved.

$$

%&'

Teacher Resource Bank / GCE Physics A / PHYA4 Section A / Sample Question Paper / Version 1.0

The graph shows how the force acting on a body changes with time.
12 10 8

force/N

6 4 2 0 0 10 20 30 40 50

time/s The body has a mass of 0.25 kg and is initially at rest. What is the average acceleration of the body assuming no other forces are acting? A B C D 0.25 m s-2 1.0 m s-2 20 m s-2 40 m s-2

A cricket ball of mass 0.20 kg moving at 6.0 m s-1 is caught and brought to rest in 0.30 s. What is the average force exerted on the ball? A B C D 0.40 N 4.0 N 6.0 N 9.0 N

Which one of the following statements is correct? In an inelastic collision A B C D momentum is conserved but kinetic energy is not conserved. momentum is not conserved but kinetic energy is conserved. both momentum and kinetic energy are conserved. neither momentum nor kinetic energy is conserved.

%&'$

Copyright 2009 AQA and its licensors. All rights reserved.

$$

$$

Teacher Resource Bank / GCE Physics A / PHYA4 Section A / Sample Question Paper / Version 1.0

An object of mass 200 g, moving with a velocity u, collides with a stationary object of mass 300 g. If they stick together in the collision, what is the value of the ratio

& kinetic energy after the collision # $ $ kinetic energy before the collision ! !? % "
A
1 1 2 5

1 16 1 25

A tennis ball of mass 5.0 102 kg moves at 12 m s1 perpendicularly towards a tennis racket. After being hit by the racket the ball rebounds along the same line at 18 m s1. What is the change in the momentum of the ball?
A B C D

0.30 N s directed towards the racket 0.30 N s directed away from the racket 1.5 N s directed towards the racket 1.5 N s directed away from the racket

Copyright 2009 AQA and its licensors. All rights reserved.

$$

%&'

Teacher Resource Bank / GCE Physics A / PHYA4 Section A / Sample Question Paper / Version 1.0

A car moves round a roundabout at a steady speed.

Which one of the following statements about the centripetal force F is incorrect?
A B C D F is perpendicular to the momentum of the car. F is in the same direction as the acceleration of the car. F is equal to the product of the momentum and the angular speed of the car. F is equal to the product of the mass and the angular speed of the car.

A body is in simple harmonic motion of amplitude 0.50m and period 4 seconds. What is the speed of the body when the displacement of the body is 0.30 m?
A B C D

0.10 m s 1 0.15 m s 1 0.20 m s 1 0.40 m s 1

10

To find a value for the acceleration of free fall, g, a student measured the time of oscillation, T, of a simple pendulum whose length, l, is changed. The student used the results to plot a graph of T 2 (y axis) against l (x axis) and found the slope of the line to be S. It follows that g is
A B C D

4!2 . S 42S. 2! . S 2S.

%&'$

Copyright 2009 AQA and its licensors. All rights reserved.

$$

$$

Teacher Resource Bank / GCE Physics A / PHYA4 Section A / Sample Question Paper / Version 1.0

11

vibrations X

mass A mass-spring system carries a mass of 0.40 kg. When the point of suspension is made to vibrate vertically at a frequency of 15 Hz, resonance occurs. What mass should be added to the 0.40 kg in order to reduce the resonant frequency to 10 Hz?
A B C D

0.20 kg 0.40 kg 0.50 kg 0.60 kg

12

The mass of a particular planet is 10% of the mass of the Earth and its radius is 50% of the radius of the Earth. What is the gravitational field strength at the surface of this planet?
A B C D

0.98 N kg-1 2.0 N kg-1 3.9 N kg-1 5.9 N kg-1

13

Satellites R and S move in different stable circular orbits around the Earth. The orbital period of R is smaller than the orbital period of S. Which one of the following quantities must be greater for S than for R?
A B C D

mass momentum kinetic energy potential energy

Copyright 2009 AQA and its licensors. All rights reserved.

$$

%&'

Teacher Resource Bank / GCE Physics A / PHYA4 Section A / Sample Question Paper / Version 1.0

14

A satellite of mass m moves along a circular orbit of diameter D above the surface of a planet. If the time period of the satellite is T, what is the kinetic energy of the satellite?
A
! 2m D 2T ! 2m D 2 2T

! 2m D 2 2T 2 ! 2m D 2T 2

15

The force between two point charges when at a certain separation is F. When the F distance between them is changed to 200 mm, the force between them becomes . 9 What was the distance between the two charges originally?
A B C D

20 mm 25 mm 67 mm 100 mm

16

An electron, of charge e and mass m, starts from rest and is accelerated through a potential difference V. What is the final velocity of the electron?
A
eV m

2eV m

2eV m
V e m

%&'$

Copyright 2009 AQA and its licensors. All rights reserved.

$$

$$

Teacher Resource Bank / GCE Physics A / PHYA4 Section A / Sample Question Paper / Version 1.0

17

An electron travelling horizontally enters a uniform electric field which is directed vertically downwards.

Which one of the following statements is correct?


A B C D

The electron follows a circular path in the field. The force on the electron acts vertically downwards in the field. There is no change in the speed of the electron. The electron accelerates whilst in the field.

18

Which one of the following graphs shows how the electric potential V due to a negative point charge varies with distance x from the charge?

+ V

+ V

+ V

+ V

_
A

_
B

_
C

_
D

Copyright 2009 AQA and its licensors. All rights reserved.

$$

%&'

Teacher Resource Bank / GCE Physics A / PHYA4 Section A / Sample Question Paper / Version 1.0

19

A capacitor, initially uncharged, is charged by using a constant current. The four graphs, (i) to (iv), on which both scales are linear, show how quantities could vary with time, t, during this charging process.

00
(i)

t 00
(ii)

0
(iii)

t 00
(iv)

Which line, A to D, in the table shows correctly the graphs which would represent current against time, and potential difference across the capacitor against time?
current against time A B C D pd against time

graph (i) graph (i) graph (iv) graph (iv)

graph (ii) graph (iii) graph (i) graph (ii)

20

A capacitor charged to a pd of 6.0 V stores a charge of 30 C. What is the energy stored by the capacitor charged at this pd?
A B C D

5.0 10-6 J 9.0 10-5 J 1.8 10-4 J 5.4 10-4 J

%&'$

Copyright 2009 AQA and its licensors. All rights reserved.

$$

$$

Teacher Resource Bank / GCE Physics A / PHYA4 Section A / Sample Question Paper / Version 1.0

21

movement

uniform magnetic field (upwards out of page)

The diagram shows a conducting wire PQ moving to the right, perpendicularly across a uniform magnetic field that is directed upwards out of the page. A force acts on the free electrons in the wire, causing them to redistribute and produce an emf between the ends of PQ. Which line, A to D, in the table gives the correct direction for the force on the electrons, and the relationship between the electric potentials, VP and VQ, of the ends of PQ.
direction of force on electrons A B C D potentials of P and Q

from P to Q from P to Q from Q to P from Q to P

VP > VQ VP < VQ VP > VQ VP < VQ

22

A straight wire of length 0.30 m carries a current of 2.0 A perpendicular to a uniform magnetic field of flux density 5.0 10-2 T. Under these conditions, the magnetic force acting on the wire balances its weight, so that the wire is in equilibrium. What is the mass of the wire?
A B C D

8.0 10-4 kg 3.1 10-3 kg 3.0 10-2 kg 8.2 10-1 kg

10

Copyright 2009 AQA and its licensors. All rights reserved.

$$

%&'

Teacher Resource Bank / GCE Physics A / PHYA4 Section A / Sample Question Paper / Version 1.0

23

coil

rotation
30

uniform magnetic field

axle A 1200-turn coil, of area 3.0 103 m2, is rotated in a uniform magnetic field of flux density 80 mT. What is the flux linkage when the plane of the coil makes an angle of 30 with the direction of the field, as shown by the front view in the diagram?
A B C D 24

0.14 Wb turns 0.17 Wb turns 0.23 Wb turns 0.29 Wb turns

A coil of 400 turns, each of area 2.0 10-4 m2, is placed with its plane perpendicular to a magnetic field. The magnitude of the flux density of the field varies with time as shown in the graph.
0.05

flux 0.04 density/T


0.03 0.02 0.01 0 0 1 2 3 4

time/ms

What is the emf induced in the coil during the first 3.0 ms?
A B C D

zero 1.3 10-3 V 3.3 10-3 V 1.3 V

%&'$

Copyright 2009 AQA and its licensors. All rights reserved.

$$

$$

11

Teacher Resource Bank / GCE Physics A / PHYA4 Section A / Sample Question Paper / Version 1.0

25

An aluminium ring is placed with its plane perpendicular to a magnetic field. The field increases at a uniform rate of 3.0 Ts-1. If the effective cross-sectional area of the ring is 1.0 10-2 m2, what emf is induced in the ring?
A B C D

15 mV 30 mV 45 mV 60 mV

26

The diagram shows a rectangular coil in a uniform magnetic field. The axis of the coil is perpendicular to the direction of the magnetic field.

The graph shows how the induced emf changes with time when the coil spins at constant frequency. At which point on the graph is the flux linkage through the coil zero and decreasing?

emf

time

12

Copyright 2009 AQA and its licensors. All rights reserved.

$$

%&'

Teacher Resource Bank / GCE Physics A / PHYA4 Section A / Sample Question Paper / Version 1.0

27

A 500 turn coil of area 20 cm2 rotates uniformly at 30 revolutions per second about an axis perpendicular to a uniform magnetic field of flux density 0.50 T. What is the peak emf induced in the coil?
A B C D

94 V 1.5 103 V 1.5 105 V 9.4 105 V

28

A step-up transformer which has a 100 turn primary coil is to be used to transform an alternating pd to 120 V. Which line, A to D, in the table gives the primary pd and number of turns on the secondary coil for this task?
primary pd/V A B C D number of secondary turns

6.0 6.0 9.0 9.0

1000 2000 1000 2000

29

Which one of the following features would not improve the efficiency of a transformer?
A B C D

thermal lagging around the coils a core which is laminated a core made from a material which is magnetically soft thick, low resistivity wire for the high current winding

30

A high voltage power line consists of cables of total resistance 50 ". The power line delivers 3.6 MW at 120 kV. What is the power dissipated in the power line due to the heating effect of the electric current?
A B C D

45 kW 72 kW 192 kW 288 kW

END OF QUESTIONS

%&'$

Copyright 2009 AQA and its licensors. All rights reserved.

$$

$$

13

Teacher Resource Bank / GCE Physics A / PHYA4 Section A / Sample Question Paper / Version 1.0

Keys to Objective Test Questions

This component is an objective test for which the following list indicates the correct answers used in marking the candidates responses.
Keys to Objective Test Questions 1 2 3 4 5 6 7 8 9 10 11 12 13 14 15

16 17 18 19 20 21 22 23 24 25 26 27 28 29 30

14

Copyright 2009 AQA and its licensors. All rights reserved.

$$

%&'

Centre Number Surname Other Names Candidate Signature

Candidate Number

For Examiners Use

Examiners Initials

Question General Certificate of Education Advanced Level Examination Easter 2010

Mark

1 2 3

Physics A
Unit 4 Section B Fields and Further Mechanics Sample Paper 1
For this paper you must have: a calculator a ruler a Data and Formulae Booklet

PHYA4/2

4 5 6 TOTAL

Time allowed The total time for both sections of this paper is 1 hour 45 minutes You are advised to spend approximately one hour on this section. Instructions Use black ink or black ball-point pen. Fill in the boxes at the top of this page. Answer all questions. You must answer the questions in the spaces provided. Answers written in margins or on blank pages will not be marked. Do all rough work in this book. Cross through any work you do not want to be marked. Information The marks for questions are shown in brackets. The maximum mark for this section is 51 You are expected to use a calculator where appropriate. A Data and Formulae Booklet is provided as a loose insert. You will be marked for your ability to: - use good English - organise information clearly - use specialist vocabulary where appropriate.

Areas outside the box will not be scanned for marking

1.

The gravitational field strength at the surface of a planet, X, is 19 N kg1. (a) (i) Calculate the gravitational potential difference between the surface of X and a point 10 m above the surface, if the gravitational field can be considered to be uniform over such a small distance. ........................................................................................................................... ...........................................................................................................................

(ii)

Calculate the minimum amount of energy required to lift a 9.0 kg rock a vertical distance of 10m from the surface of X. ........................................................................................................................... ...........................................................................................................................

(iii)

State whether the minimum amount of energy you have found in part (a)(ii) would be different if the 9.0 kg mass were lifted a vertical distance of 10 m from a point near the top of the highest mountain of planet X. Explain your answer. ........................................................................................................................... ........................................................................................................................... ...........................................................................................................................
(3)

(b)

Calculate the gravitational field strength at the surface of another planet, Y, that has the same mass as planet X, but twice the diameter of X. ..................................................................................................................................... ..................................................................................................................................... ..................................................................................................................................... .....................................................................................................................................
(2) (Total 5 marks)

Turn over
0 1

Areas outside the box will not be scanned for marking

2.

A capacitor of capacitance 330 F is charged to a potential difference of 9.0 V. It is then discharged through a resistor of resistance 470 k. Calculate (a) the energy stored by the capacitor when it is fully charged, ................................................................................................................................. ................................................................................................................................. ................................................................................................................................. .................................................................................................................................
(2)

(b)

the time constant of the discharging circuit, ................................................................................................................................. .................................................................................................................................


(1)

(c)

the p.d. across the capacitor 60 s after the discharge has begun. ................................................................................................................................. ................................................................................................................................. ................................................................................................................................. ................................................................................................................................. ................................................................................................................................. .................................................................................................................................
(3) (Total 6 marks)

Turn over
0 2

Areas outside the box will not be scanned for marking

3.

An electric motor in a machine drives a rotating drum by means of a rubber belt attached to pulleys, one on the motor shaft and one on the drum shaft, as shown in the diagram below.
drum drum pulley electric motor motor pulley

machine

rubber belt

(a)

The pulley on the motor shaft has a diameter of 24 mm. When the motor is turning at 50 revolutions per second, calculate (i) the speed of the belt, ........................................................................................................................... ........................................................................................................................... ........................................................................................................................... ........................................................................................................................... (ii) the centripetal acceleration of the belt as it passes round the motor pulley. ........................................................................................................................... ........................................................................................................................... ........................................................................................................................... ........................................................................................................................... ...........................................................................................................................
(5)

(b)

When the motor rotates at a particular speed, it causes a flexible metal panel in the machine to vibrate loudly. Explain why this happens. ..................................................................................................................................... ..................................................................................................................................... ..................................................................................................................................... ..................................................................................................................................... .....................................................................................................................................
(2) (Total 7 marks)

Turn over
0 3

Areas outside the box will not be scanned for marking

4.

To celebrate the Millennium in the year 2000, a footbridge was constructed across the River Thames in London. After the bridge was opened to the public it was discovered that the structure could easily be set into oscillation when large numbers of pedestrians were walking across it. (a) What name is given to this kind of physical phenomenon, when caused by a periodic driving force? .....................................................................................................................................
(1)

(b)

Under what condition would this phenomenon become particularly hazardous? Explain your answer. ..................................................................................................................................... ..................................................................................................................................... ..................................................................................................................................... ..................................................................................................................................... ..................................................................................................................................... .....................................................................................................................................
(4)

(c)

Suggest two measures which engineers might adopt in order to reduce the size of the oscillations of a bridge measure 1 ........................................................................................................... ..................................................................................................................................... measure 2 ........................................................................................................... .....................................................................................................................................
(2) (Total 7 marks)

Turn over
0 4

Areas outside the box will not be scanned for marking

5.

(a)

In an experiment to illustrate electromagnetic induction, a permanent magnet is moved towards a coil, as shown in Figure 1, causing an emf to be induced across the coil. Figure 1
coil S permanent magnet to voltage sensor

Using Faradays law, explain why a larger emf would be induced in this experiment if a stronger magnet were moved at the same speed. You may be awarded additional marks to those shown in brackets for the quality of written communication in your answer. ..................................................................................................................................... ..................................................................................................................................... ..................................................................................................................................... ..................................................................................................................................... ..................................................................................................................................... .....................................................................................................................................
(3)

Turn over
0 5

Areas outside the box will not be scanned for marking

(b)

A conductor of length l is moved at a constant speed v so that it passes perpendicularly through a uniform magnetic field of flux density B, as shown in Figure 2. Figure 2 uniform magnetic field (perpendicular to the plane of the diagram) over this region

(i)

Give an expression for the area of the magnetic field swept out by the conductor in time t. ........................................................................................................................... ...........................................................................................................................

(ii)

Show that the induced emf, , across the ends of the conductor is given by
= Blv.

........................................................................................................................... ........................................................................................................................... ...........................................................................................................................


(3)

Turn over
0 6

Areas outside the box will not be scanned for marking

(c)

A simple electrical generator can be made from a copper disc, which is rotated at right angles to a uniform magnetic field, directed into the plane of the diagram (Figure 3). An emf is developed across terminals P (connected to the axle) and Q (connected to a contact on the edge of the disc). Figure 3

uniform magnetic field (acting into the plane of the diagram) over this region

axel M copper disc

contact

The radius of the disc is 64 mm and it is rotated at 16 revolutions per second in a uniform magnetic field of flux density 28 mT. (i) Calculate the angular speed of the disc. ........................................................................................................................... ........................................................................................................................... ...........................................................................................................................

(ii)

Calculate the linear speed of the mid-point M of a radius of the disc. ........................................................................................................................... ........................................................................................................................... ...........................................................................................................................

(iii) Hence, or otherwise, calculate the emf induced across terminals P and Q. ........................................................................................................................... ........................................................................................................................... ...........................................................................................................................
(5) (Total 11 marks)

Turn over
0 7

Areas outside the box will not be scanned for marking

6.

(a)

(i)

Give an equation showing how the principle of conservation of momentum applies to the colliding snooker balls shown in the diagram.
u1 m1 m2 u2

before collision
v1 m1 m2 v2

after collision ...........................................................................................................................

(ii)

State the condition under which the principle of conservation of momentum applies. ........................................................................................................................... ...........................................................................................................................
(3)

(b)

A trolley, A, of mass 0.25 kg and a second trolley, B, of mass 0.50 kg are held in contact on a smooth horizontal surface. A compressed spring inside one of the trolleys is released and they then move apart. The speed of A is 2.2 m s1. (i) Calculate the speed of B. ........................................................................................................................... ........................................................................................................................... ...........................................................................................................................

(ii)

Calculate a minimum value for the energy stored in the spring when compressed. ........................................................................................................................... ........................................................................................................................... ...........................................................................................................................
(4)

Turn over
0 8

Areas outside the box will not be scanned for marking

(c)

The rotor blades of a helicopter sweep out a cross-sectional area, A. The motion of the blades helps the helicopter to hover by giving a downward velocity, , to a cylinder of air, density . The cylinder of air has the same cross-sectional area as that swept out by the rotor blades. Explaining your reasoning, (i) derive an expression for the mass of air flowing downwards per second, and ........................................................................................................................... ........................................................................................................................... ...........................................................................................................................

(ii)

derive an expression for the momentum given per second to this air. ........................................................................................................................... ...........................................................................................................................

(iii) Hence show that the motion of the air results in an upward force, F, on the helicopter given by

F = A2.
........................................................................................................................... ........................................................................................................................... ...........................................................................................................................
(5)

(d)

A loaded helicopter has a mass of 2500 kg. The area swept out by its rotor blades is 180 m2. If the downward flow of air supports 50% of the weight of the helicopter, what speed must be given to the air by the motion of the rotor blades when the helicopter is hovering? Take the density of air to be 1.3 kg m3. ..................................................................................................................................... ..................................................................................................................................... .....................................................................................................................................
(3) (Total 15 marks)

Turn over
0 9

Physics A PHYA4 - AQA GCE Mark Scheme 2010 January series

Instructions to Examiners
1. Give due credit for alternative treatments which are correct. Give marks for what is correct in accordance with the mark scheme; do not deduct marks because the attempt falls short of some ideal answer. Where marks are to be deducted for particular errors, specific instructions are given in the marking scheme. Do not deduct marks for poor written communication. Refer the scripts to the Awards meeting if poor presentation forbids a proper assessment. In each paper, candidates are assessed on their quality of written communication (QWC) in designated questions (or part-questions) that require explanations or descriptions. The criteria for the award of marks on each such question are set out in the mark scheme in three bands in the following format. The descriptor for each band sets out the expected level of the quality of written communication of physics for each band. Such quality covers the scope (eg relevance, correctness), sequence and presentation of the answer. Amplification of the level of physics expected in a good answer is set out in the last row of the table. To arrive at the mark for a candidate, their work should first be assessed holistically (ie in terms of scope, sequence and presentation) to determine which band is appropriate then in terms of the degree to which the candidates work meets the expected level for the band. QWC descriptor mark range see specific mark scheme Good Excellent see specific mark scheme Modest Adequate see specific mark scheme Poor Limited The description and/or explanation expected in a good answer should include a coherent account of the following points: see specific mark scheme Answers given as bullet points should be considered in the above terms. Such answers without an overview paragraph in the answer would be unlikely to score in the top band. 3. An arithmetical error in an answer will cause the candidate to lose one mark and should be annotated AE if possible. The candidates incorrect value should be carried through all subsequent calculations for the question and, if there are no subsequent errors, the candidate can score all remaining marks. The use of significant figures is tested once on each paper in a designated question or partquestion. The numerical answer on the designated question should be given to the same number of significant figures as there are in the data given in the question or to one more than this number. All other numerical answers should not be considered in terms of significant figures. Numerical answers presented in non-standard form are undesirable but should not be penalised. Arithmetical errors by candidates resulting from use of non-standard form in a candidates working should be penalised as in point 3 above. Incorrect numerical prefixes and the use of a given diameter in a geometrical formula as the radius should be treated as arithmetical errors. Knowledge of units is tested on designated questions or parts of questions in each a paper. On each such question or part-question, unless otherwise stated in the mark scheme, the mark scheme will show a mark to be awarded for the numerical value of the answer and a further mark for the correct unit. No penalties are imposed for incorrect or omitted units at intermediate stages in a calculation or at the final stage of a non-designated unit question. All other procedures including recording of marks and dealing with missing parts of answers will be clarified in the standardising procedures.

2.

4.

5.

6.

7.

Physics A PHYA4 - AQA GCE Mark Scheme 2010 January series

PHYA4/2: Fields and Further Mechanics

Question 1 (a) (i) (ii)

V 19 = () V gives V = 190 (1) J kg1 (1) g = 10 x


W(= mV) = 9.0 190 = 1710J [or mgh = 9.0 19 10 = 1710J] (1) 3 on mountain, required energy would be less because gravitational field strength is less (1)

(iii)

(b)

GMm 1 1 (or F or correct use of F = ) (1) 2 2 r2 r r 19 g = 2 = 4.75(Nkg1) (1) 2

Total

Question 2 (a) Q (= CV = 330 9.0) = 2970 (C) (1) E (= QV) = _ 2.97 103 9.0 = 1.34 102J (1) [or E (= CV2) = 300 106 9.02 (1) = 1.34 102J (1)] time constant (= RC) = 470 103 330 106 = 155 s (1) 2

(b)

(c)

Q = Q0 e t / RC = 2970 e60/155 = 2020 (C) (allow C.E. for time constant from (b)) Q V = = 2020 = 6.11 V (1) C 330 (allow C.E. for Q) [or V = V0et/RC (1) = 9.0 e60/155 (1) = 6.11 V (1)]

Total

Physics A PHYA4 - AQA GCE Mark Scheme 2010 January series

Question 3
(a) (i) r = 0.012 (m) (use of v = 2fr gives) v = 250 0.012 (1) = 3.8 m s1 (1) (3.77 m s1) (ii)
2 2 correct use of a = v or a = 3.8 (1) r 0.012

= 1.2 103 m s2 (1) [or correct use of = 2r] (allow C.E. for value of v from (i) (b) panel resonates (1) (because) motor frequency = natural frequency of panel (1)

2 Total 7

Question 4
(a) forced vibrations or resonance (1)

(b)

reference to natural frequency (or frequencies) of structure (1) driving force is at same frequency as natural frequency of structure (1) resonance (1) large amplitude vibrations produced or large energy transfer to structure(1) could cause damage to structure [or bridge to fail] (1) stiffen the structure (by reinforcement) (1) install dampers or shock absorbers (1) [or other acceptable measure e.g. redesign to change natural frequency or increase mass of bridge or restrict number of pedestrians]

(c)

Total

Physics A PHYA4 - AQA GCE Mark Scheme 2010 January series

Question 5
(a) greater flux (linkage) or more flux lines (at same distance) [or stronger magnet produces flux lines closer together] (1) greater rate of change of flux (linkage) [or more flux lines cut per unit time] (1) emf rate of change of flux (linkage) (1) [or using = 3

, where = A B, v and t are the same (1) t

B is larger since magnet is stronger (1) and A are constant, is larger (1)]
(b) area swept out, A = lvt (1) 3

(= BA) = Blv t (1)

Blvt = ( ) gives result (1) = t t


(c) (i) w(=2f) = 2 l6 (1) = 101 rads1 (1) (ii) v(=rw) = 32 103 101 = 3.2(3)ms1 (1) (allow C.E. for value of w from (i)) (iii) (= Blv) = 28 103 64 103 3.23 (1) = 5.7(9) 103V (1) 5 (allow C.E. for values of v from (ii)) (solutions using = Bfnr2 to give 5.7(6) 103 V acceptable) 5

Total

Physics A PHYA4 - AQA GCE Mark Scheme 2010 January series

Question 6
(a) (i) equation showing momentum before = momentum after (1) correct use of sign (1) 3

(ii) (b) (i)

no external forces (on any system of colliding bodies) (1) (by conservation of momentum m11 + m22 = 0) 0.25 2.2 = ()0.502 (1) 4

2 = ()1.1(0)ms1 (1)
(ii) allow e.c.f from (i) min. stored energy = total k.e. = 1.12 (1) = (c) (i) 0.91J (1) mass of air per second = A (1) correct justification, incl ref to time (1) momentum per second (= M = 2 A) = 2 A (1) force = rate of change of momentum (hence given result) (1) upward force on helicopter equals (from Newton third law) downward force on air (1) 3 5

1 1 0.25 2.22 + 0.5 2 2

(ii) (iii)

(d)

mg (for 50% support) (1) 2 2500 9.81 2 180 1.3 = 2 1 gives = 7.2ms (1) (or 7.3, g taken as 10) if not 50% of weight, max 1/3 provided all correct otherwise (gives 10.2)

2 A =

Total

General Certificate of Education 2010 Advanced Examination


version 1.3

!"#
PHYA4/1

PHYSICS A Unit 4: Fields and Further Mechanics Section A


SPECIMEN PAPER
For this paper you must have: an objective test answer sheet black ball-point pen a calculator a question paper/answer book for Section B (enclosed)

Instructions Use a black ball-point pen. Do not use pencil. Answer all questions in this section. For each question there are four responses. When you have selected the response which you think is the most appropriate answer to a question, mark this response on your answer sheet. Mark all responses as instructed on your answer sheet. If you wish to change your answer to a question, follow the instructions on your answer sheet. Do all rough work in this book not on the answer sheet. Information The maximum mark for this paper is 25. All questions in Section A carry equal marks. No deductions will be made for incorrect answers. A Data and Formulae Booklet is provided as a loose insert. The question paper/answer book for Section B is enclosed within this question paper.

For the two physical quantities, impulse and force, which one of the following is correct? A B C D Impulse is a scalar and force is a scalar. Impulse is a scalar and force is a vector. Impulse is a vector and force is a scalar. impulse is a vector and force is a vector.

A particle of mass m strikes a rigid wall perpendicularly from the left with velocity v. m v rigid wall

If the collision is perfectly elastic, the change in momentum of the particle which occurs as a result of the collision is A B C D 3 F/N 10 2mv to the right. 2mv to the left. mv to the left. zero.

0 0 2.5 5.0 t/s

A force, F, varies with time, t, as shown by the graph and is applied to a body initially at rest on a smooth surface. What is the momentum of the body after 5.0 s? A B C D zero. 12.5 N s. 25 N s. 50 N s.

The rate of change of momentum of a body falling freely under gravity is equal to its A B C D weight. power. kinetic energy. potential energy.

What is the value of the angular velocity of a point on the surface of the Earth? A B C D 1.2 10-5 rad s-1 7.3 10-5 rad s-1 2.6 10-1 rad s-1 4.6 102 rad s-1

6 0.8 m P

A model car moves in a circular path of radius 0.8 m at an angular speed of is its displacement from point P, 6 s after passing P? A B C D zero 1.6 m 0.4 !m 1.6 !m

! rad s-1. What 2

A particle of mass m moves horizontally at constant speed v along the arc of a circle from P1 to P2 under the action of a force. What is the work done on the particle by the force during this displacement? P2 v

v P1 A B C D 8 zero
!mv 2 2

mv2 2 mv
2

A body moves with simple harmonic motion of amplitude 0.50 m and period 4! seconds. What is the speed of the body when the displacement of the body from the equilibrium position is 0.30 m? A B C D 0.10 m s-1 0.15 m s-1 0.20 m s-1 0.40 m s-1

The time period of a simple pendulum is doubled when the length of the pendulum is increased by 3.0 m. What is the original length of the pendulum? A B C D 1.0 m 1.5 m 3.0 m 6.0 m

10

Which one of the following statements is not true for a body vibrating in simple harmonic motion when damping is present? A B C D The damping force is always in the opposite direction to the velocity. The damping force is always in the opposite direction to the displacement. The presence of damping gradually reduces the maximum potential energy of the system. The presence of damping gradually reduces the maximum kinetic energy of the system.

11

The Earth has density " and radius R. The gravitational field strength at the surface is g. What is the gravitational field strength at the surface of a planet of density 2" and radius 2R? A B C D g 2g 4g 16g

12

Which one of the following graphs correctly shows the relationship between the gravitational force, F, between two masses and their separation, r?

13

Near the surface of a planet the gravitational field strength is uniform and for two points, 10 m apart vertically, the gravitational potential difference is 3 J kg-1. How much work must be done in raising a mass of 4 kg vertically through 5 m? A B C D 3J 6J 12 J 15 J

14

Two isolated point charges are separated by 0.04 m and attract each other with a force of 20 "N. If the distance between them is increased by 0.04 m, what is the new force of attraction? A B C D 5 "N 10 "N 20 "N 40 "N
& electrostatic force # ! ! for the forces acting between them? % gravitational force "

15

Two protons, each of mass m and charge e, are a distance d apart. Which one of the following expressions correctly gives the ratio $ $
4!# 0 e 2 Gm 2

Ge 2 4!# 0 m 2

e2m2 4!# 0 G e2 4!# 0 Gm 2

D 16

An electron travelling at constant speed enters a uniform electric field at right angles to the field. While the electron is in the filed it accelerates in a direction which is A B C D in the same direction as the electric field in the opposite direction to the electric field in the same direction as the motion of the electron in the opposite direction to the motion of the electron

17

Which one of the following statements about electric potential and electric field strength is correct? A B C D electric potential is zero whenever the electric field strength is zero electric field strength is a scalar quantity electric potential is a vector quantity electric potential due to a point charge varies as (l/r) where r is the distance from the point charge

18

A 1000 "F capacitor and a 10 "F capacitor are charged so that the potential difference across each of them is the same. The charge stored in the 1000 "F capacitor is Q1 and the charge stored in the 10 "F capacitor is Q2. What is the ratio A B C D 100 10 1
1 100
Q1 ? Q2

19

In experiments to pass a very high current through a gas, a bank of capacitors of total capacitance 50 "F is charged to 30kV. If the bank of capacitors could be discharged completely in 5.0 ms, what would be the mean power delivered? A B C D 22 kW 110 kW 4.5 MW 9.0 MW

20

The graph shows how the charge stored by a capacitor varies with the potential difference across it as it is charged from a 6 V battery.

Which one of the following statements is not correct? A B C D The capacitance of the capacitor is 5.0 "F. When the potential difference is 2 V the charge stored is 10 "C. When the potential difference is 2 V the energy stored is 10 "J. When the potential difference is 6 V the energy stored is 180 "J.

21

The magnetic flux, $, through a coil varies with time, t, as shown by the first graph. Which one of the following graphs, A to D, best represents how the magnitude, , of the induced emf varies in this same period of time?

22

Protons, each of mass m and charge e, follow a circular path when travelling perpendicular to a magnetic field of uniform flux density B. What is the time taken for one complete orbit? A B C D
2!eB m m 2!eB
eB 2!m

2!m eB

23

The diagram shows a square coil with its plane parallel to a uniform magnetic field. Which one of the following would induce an emf in the coil? A B C D movement of the coil slightly to the left movement of the coil slightly downwards rotation of the coil about an axis through XY rotation of the coil about an axis perpendicular to the plane of the coil through Z

24

The primary winding of a perfectly efficient transformer has 200 turns and the secondary has 1000 turns. When a sinusoidal pd of rms value 10 V is applied to the input, there is a primary current of rms value 0.10 A rms. Which line in the following table, A to D, gives correct rms output values obtainable from the secondary when the primary is supplied in this way? rms output emf/V A B C D 50 50 10 10 rms output current/A 0.10 0.02 0.10 0.02

25

Why, when transporting electricity on the National Grid, are high voltages and low currents used? A B C D The energy lost by radiation from electromagnetic waves is reduced. The electrons move more rapidly. The heat losses are reduced. The resistance of the power lines is reduced.

10

Surname Centre Number Candidate Signature

Other Names Candidate Number

Leave blank

General Certificate of Education 2010 Advanced Examination


version 1.1

!"#
PHYA4/2
For Examiners Use Number Mark Number 5 6 Mark 1 2 3 4 Total (Column 1) Total (Column 2) TOTAL Examiners Initials

PHYSICS A Unit 4 Fields and Further Mechanics Section B


SPECIMEN PAPER

Time allowed: 1 hour


Instructions Use blue or black ink or ball-point pen. Fill in the boxes at the top of this page. Answer all questions. A Data and Formula Booklet is provided as a loose insert. Information The maximum mark for this paper is 50. The marks for the questions are shown in brackets. You are reminded of the need for good English and clear presentation in your answers. You will be assessed on your quality of written communication where indicated in the question.

A golf club undergoes an inelastic collision with a stationary golf ball and gives it an initial velocity of 60 m s-1. The ball is in contact with the club for 15 ms and the mass of the ball is 4.5 10-2 kg. (a) Explain what is meant by an inelastic collision. ............................................................................................................................................. ............................................................................................................................................. (1 mark) (b) Calculate (i) the change in momentum of the ball, ................................................................................................................................... ................................................................................................................................... (ii) the average force the club exerts on the ball. ................................................................................................................................... ................................................................................................................................... ................................................................................................................................... (4 marks) Total 5 marks

(a) A spring, which hangs from a fixed support, extends by 40 mm when a mass of 0.25 kg is suspended from it. (i) Calculate the spring constant of the spring. ................................................................................................................................... ................................................................................................................................... (ii) An additional mass of 0.44 kg is then placed on the spring and the system is set into vertical oscillation. Show that the oscillation frequency is 1.5 Hz. ................................................................................................................................... ................................................................................................................................... ................................................................................................................................... ................................................................................................................................... (4 marks) (b) With both masses still in place, the spring is now suspended from a horizontal support rod that can be made to oscillate vertically, as shown in Figure 1, with amplitude 30 mm at several different frequencies. Figure 1

support rod

vertical oscillations

spring

masses

The response of the masses suspended from the spring to the vertical oscillations of the support rod varies with frequency. Describe and explain, as fully as you can, the motion of the masses when the support rod oscillates at a frequency of (i) 0.2 Hz, (ii) 1.5 Hz and (iii) 10 Hz. The quality of your written answer will be assessed in this question. ............................................................................................................................................. ............................................................................................................................................. ............................................................................................................................................. ............................................................................................................................................. ............................................................................................................................................. ............................................................................................................................................. ............................................................................................................................................. ............................................................................................................................................. ............................................................................................................................................. ............................................................................................................................................. ............................................................................................................................................. ............................................................................................................................................. ............................................................................................................................................. ............................................................................................................................................. ............................................................................................................................................. ............................................................................................................................................. (6 marks) Total 10 marks

Communications satellites are usually placed in a geo-synchronous orbit. (a) State two features of a geo-synchronous orbit. ............................................................................................................................................. ............................................................................................................................................. ............................................................................................................................................. ............................................................................................................................................. (2 marks) (b) The mass of the Earth 6.00 1024 kg and its mean radius is 6.40 106 m. (i) Show that the radius of a geo-synchronous orbit must be 4.23 107 m, ................................................................................................................................... ................................................................................................................................... ................................................................................................................................... ................................................................................................................................... ................................................................................................................................... ................................................................................................................................... (ii) Calculate the increase in potential energy of a satellite of 750 kg when it is raised from the Earths surface into a geo-synchronous orbit. ................................................................................................................................... ................................................................................................................................... ................................................................................................................................... ................................................................................................................................... (6 marks)

(c)

Satellites in orbits nearer the Earth than geo-synchronous satellites may be used in the future to track road vehicles. (i) State and explain one reason why geo-synchronous satellites would not be suitable for such a purpose. ................................................................................................................................... ................................................................................................................................... ................................................................................................................................... (ii) Give two points you would make in arguing for or against tracking road vehicles. Explain your answers. ................................................................................................................................... ................................................................................................................................... ................................................................................................................................... ................................................................................................................................... ................................................................................................................................... ................................................................................................................................... ................................................................................................................................... ................................................................................................................................... ................................................................................................................................... ................................................................................................................................... (4 marks) Total 12 marks

A small charged sphere of mass 2.1 10-4 kg, suspended from a thread of insulating material, was placed between two vertical parallel plates 60 mm apart. When a potential difference of 4200 V was applied to the plates, the sphere moved until the thread made an angle of 6.0 to the vertical, as shown in Figure 2. Figure 2

(a)

Show that the electrostatic force F on the sphere is given by F = mg tan 6.0, where m is the mass of the sphere. ............................................................................................................................................. ............................................................................................................................................. ............................................................................................................................................. ............................................................................................................................................. ............................................................................................................................................. (3 marks)

(b)

Calculate the charge on the sphere. ............................................................................................................................................. ............................................................................................................................................. ............................................................................................................................................. ............................................................................................................................................. ............................................................................................................................................. (3 marks) Total 6 marks

The flash tube in a camera produces a flash of light when a 180 !F capacitor is discharged across the tube. Figure 3 A S 100 V electronic charging unit + 180 B camera flash tube

(a)

The capacitor is charged to a pd of 100 V from an electronic charging unit in the camera, as shown in Figure 3. Calculate, (i) the energy stored in the capacitor, ................................................................................................................................... ................................................................................................................................... ................................................................................................................................... (ii) the work done by the battery. ................................................................................................................................... ................................................................................................................................... (2 marks)

(b)

When a photograph is taken, switch S in Figure 3 is automatically moved from A to B and the capacitor is discharged across the flash tube. The discharge circuit has a resistance of 1.5 ". Emission of light from the flash tube ceases when the pd falls below 30 V. (i) Calculate the duration of the light flash. ................................................................................................................................... ................................................................................................................................... ................................................................................................................................... ................................................................................................................................... ...................................................................................................................................

(ii)

The capacitor in the circuit in Figure 3 is replaced by a capacitor of greater capacitance. Discuss the effect of this change on the photograph image of a moving object. ................................................................................................................................... ................................................................................................................................... ................................................................................................................................... ................................................................................................................................... ................................................................................................................................... (4 marks) Total 6 marks

(a) In an experiment to illustrate electromagnetic induction, a permanent magnet is moved towards a coil, as shown in Figure 4, causing an emf to be induced across the coil. Figure 4

Using Faradays law, explain why a larger emf would be induced in this experiment if a stronger magnet were moved at the same speed. ............................................................................................................................................. ............................................................................................................................................. ............................................................................................................................................. ............................................................................................................................................. ............................................................................................................................................. ............................................................................................................................................. (3 marks)

10

(b)

A conductor of length l is moved at constant speed v so that is passes perpendicularly through a uniform magnetic field of flux density B, as shown in Figure 5. Figure 5

Show that the induced emf, , across the ends of the conductor is given by

= Blv.
............................................................................................................................................. ............................................................................................................................................. ............................................................................................................................................. (3 marks)

11

(c)

A simple electrical generator can be made from a copper disc, which is rotated at right angles to a magnetic field, directed into the plane of the diagram (Figure 6). An emf is developed across the terminals P (connected to the axle) and Q (connected to a contact on the edge of the disc). Figure 6

The radius of the disc is 64 mm and it is rotated at 16 revolutions per second in a uniform magnetic field of flux density 28 mT. (i) Calculate the angular speed of the disc. ................................................................................................................................... ................................................................................................................................... ................................................................................................................................... (ii) Calculate the linear speed of mid-point M of a radius of the disc. ................................................................................................................................... ................................................................................................................................... ................................................................................................................................... (iii) Hence, or otherwise, calculate the emf induced across the terminals P and Q. ................................................................................................................................... ................................................................................................................................... ................................................................................................................................... (5 marks) Total 11 marks

12

PHYA4: Fields and Further Mechanics


Keys to Objective Test Questions 1 D 14 A Question 1 (a) kinetic energy is not conserved ! (or velocity of approach equals velocity of separation) 1 2 B 15 D 3 C 16 B 4 A 17 D 5 B 18 A 6 B 19 C 7 A 20 D 8 C 21 C 9 A 10 B 22 D 11 C 23 C 12 D 24 B 13 B 25 C

(b)

(i)

(use of p = mv gives) p

= 4.5 10-2 60 ! = 2.7 kg m s-1 !

(ii)

(use of F =

(mv) gives) F t

2.7 ! 15 10 3

= 180 N ! [or a =
vu 60 = = 400 (m s-1)] ! t 15 10 3

F = ma = 4.5 10-2 4000 = 180 N !


Total Question 2 5

(a)

(i)

mg = ke !
& 0.25 9.81 # -1 k= $ ! = 61(.3) N m ! 3 % 40 10 "

(ii)

& m# ! = 2! T = $ = 2! $ ! k % "
1 & 1# f $= ! = ! T 0 . 667 % "

0.69 ! 61.3

(= 0.667 s)

(= 1.5(0) Hz)

(b)

The marking scheme for this part of the question includes an overall assessment for the Quality of Written Communication (QWC). There are no discrete marks for the assessment of QWC but the candidates QWC in this answer will be one of the criteria used to assign a level and award the marks for this part of the question.
Level Descriptor an answer will be expected to meet most of the criteria in the level descriptor Mark range

Good 3

- answer supported by an appropriate range of relevant points good use of information or ideas about physics, going beyond those given in the question accurate and clear expression of ideas with only minor errors of spelling punctuation and grammar good use of information or idea about physics given in the question but limited beyond this the ideas are expressed with reasonable clarity but with a few errors of spelling, punctuation and grammar
5-6

- argument well structured with minimal repetition or irrelevant points Modest 2

- answer partially supported by relevant points 3-4

- the argument shows some attempt at structure Limited 1

- valid points but not clearly linked to an argument structure - limited use of information or ideas about physics - unstructured - errors in spelling, punctuation and grammar or lack of fluency
1-2

- incorrect, inappropriate or no response examples of the sort of information or idea that might be used to support an argument

(i)

forced vibrations (at 0.2 Hz) ! amplitude fairly large (" 30 mm) ! in phase with driver ! resonance (at 1.5 Hz) ! amplitude very large (> 30 mm) ! oscillations may appear violent ! phase difference at 90 ! forced vibrations (at 10 Hz) ! small amplitude ! out of phase with driver or phase lag of ! on driver !
Total 10

(ii)

(iii)

Question 3

(a)

period is 24 hours (or equal to period of Earths rotation) ! remains in fixed position relative to surface of Earth ! equatorial orbit ! same angular speed as Earth (or equatorial surface) !
max 2

(b)

(i)

GMm = m#2r ! r2
T= 2! ! "

& GMT 2 r$ $ = 4! 2 %

# & 6.7 10 11 6.0 10 24 (24 3600) 2 ! =$ ! $ 4! 2 " %


(gives r = 42.3 103 km)

# ! ! "

1/ 3

! 6

(ii)

& 1 1# $V = GM $ ! ! %R r"
1 1 & # 7 -1 = 6.67 10-11 6 1024 $ ! = 5.31 10 (J kg ) ! 6 7 6 . 4 10 4 . 23 10 % "

$EP = m$V (= 750 5.31 107) = 3.98 1010 J ! (allow ecf for value of $V) (c) (i) signal would be too weak at large distance ! (or large aerial needed to detect/transmit signal, or any other acceptable reason) the signal spreads out more the further it travels ! (ii)
for

road pricing would reduce congestion stolen vehicles can be tracked and recovered uninsured/unlicensed vehicles can be apprehended
4

against

road pricing would increase cost of motoring possibility of state surveillance/invasion of privacy

!! any 2 valid points (must be for both for or against) Total 12

Question 4

(a)

T cos 6 = mg ! T sin 6 = F ! hence F = mg tan 6 ! [or by use of triangle: sides correct ! 6 correct ! tan 6 = F/mg !]
3

(b)

(use of E =

4200 V = 7.0 104 V m-1 ! gives) E = d 60 10 3


4 F & mg tan 6 # 2.1 10 9.81 tan 6 gives) Q $ ! ! = E E 7.0 10 4 % "

(use of Q =

= 3.1 10-9 C ! (allow ecf for value of E from (i))


Total Question 5 6

(a)

(i) (ii)

E (= CV2 = 0.5 180 10-6 1002) = 0.90 J ! W (= QV = CV2 = 180 10-6 1002) = 1.8 J ! (V = V0e-t/RC) gives 30 = 100 e-t/RC ! t = (-RC ln (30/100) = - 1.5 180 10-6 - 1.204 s) = 3.3 10-4 s !

(b)

(i)

(ii)

image would be less sharp (or blurred) because the discharge would last longer and the image would be photographed as it is moving ! image would be brighter because the capacitor stores more energy and therefore produces more light !
Total

Question 6

(a)

greater flux (linkage) or more flux lines (at same distance) [or stronger magnet produces flux lines closer together] ! greater rate of change of flux (linkage) [or more flux lines cut per unit time] ! induced emf [or =] rate of change of flux (linkage) ! [or using = NA B ! $B is larger since magnet is stronger ! t
3

N, A and $t are the same at the same speed is larger !] (b) area swept out $A = lv$t ! $# (= B$A) = Blv$t !
3

Blvt # & $ = (N ) gives result ! ! = t " t %


(c) (i) # (= 2!f) = 2! 16 ! = 101 rad s-1 ! (ii) v (= r") = 32 10-3 101 = 3.2(3) m s-1 ! (allow ecf for value of " from (i)) (iii)

(= Blv)

= 28 10-3 64 10-3 3.23 ! = 5.7(9) 10-3 V !

(allow ecf for value of v from (ii)) [or accept solutions using = Bf!r2 to give 5.7(9) 10-3 V]
Total 11

Centre Number Surname Other Names Candidate Signature

Candidate Number

General Certificate of Education Advanced Level Examination January 2010

Physics A
Unit 4 Fields and Further Mechanics Section A Thursday 28 January 2010 1.30 pm to 3.15 pm

PHYA4/1

In addition to this paper you will require: G an objective test answer sheet G a black ink or black ball-point pen G a calculator G a question paper/answer book for Section B (enclosed).

Time allowed G The total time for both sections of this paper is 1 hour 45 minutes. You are advised to spend approximately 45 minutes on this section. Instructions G Use black ink or black ball-point pen. Do not use pencil. G Answer all questions in this section. G For each question there are four responses. When you have selected the response which you think is the most appropriate answer to a question, mark this response on your answer sheet. G Mark all responses as instructed on your answer sheet. If you wish to change your answer to a question, follow the instructions on your answer sheet. G Do all rough work in this book not on the answer sheet. Information The maximum mark for this section is 25. G Section A and Section B of this paper together carry 20% of the total marks for Physics Advanced. G All questions in Section A carry equal marks. No deductions will be made for incorrect answers. G A Data and Formulae Booklet is provided as a loose insert. G The question paper/answer book for Section B is enclosed within this question paper.
G

WMP/Jan10/PHYA4/1

PHYA4/1

2 Multiple choice questions Each of Questions 1 to 25 is followed by four responses, A, B, C and D. For each question select the best response and mark its letter on the answer sheet. You are advised to spend approximately 45 minutes on this section.

1 The graph shows the variation with time, t, of the force, F, acting on a body. F

X 0 0 What physical quantity does the area X represent? A B C D the displacement of the body the acceleration of the body the change in momentum of the body the change in kinetic energy of the body t

2 Water of density 1000 kg m3 flows out of a garden hose of cross-sectional area 7.2 104 m2 at a rate of 2.0 104 m3 per second. How much momentum is carried by the water leaving the hose per second? A B C D 5.6 105 N s 5.6 102 N s 0.20 N s 0.72 N s

3 Which row, A to D, in the table correctly shows the quantities conserved in an inelastic collision? mass A B C D conserved not conserved conserved conserved momentum not conserved conserved conserved conserved kinetic energy conserved conserved conserved not conserved total energy conserved not conserved conserved conserved

WMP/Jan10/PHYA4/1

3 4 What is the angular speed of a point on the Earths equator? A B C D 7.3 105 rad s1 4.2 103 rad s1 2.6 101 rad s1 15 rad s1

5 Which one of the following does not involve a centripetal force? A B C D an electron in orbit around a nucleus a car going round a bend an particle in a magnetic field, travelling at right angles to the field an particle in a electric field, travelling at right angles to the field

6 Which one of the following gives the phase difference between the particle velocity and the particle displacement in simple harmonic motion? rad 4 rad 2 3 rad 4 2 rad

A B C D

7 A mass M hangs in equilibrium on a spring. M is made to oscillate about the equilibrium position by pulling it down 10 cm and releasing it. The time for M to travel back to the equilibrium position for the first time is 0.50 s. Which row, A to D, in the table is correct for these oscillations?

amplitude / cm A B C D 10 10 20 20

period / s 1.0 2.0 2.0 1.0

Turn over
WMP/Jan10/PHYA4/1

4 8 Which one of the following statements concerning forced vibrations and resonance is correct? A B C D An oscillating body that is not resonating will return to its natural frequency when the forcing vibration is removed. At resonance, the displacement of the oscillating body is 180 out of phase with the forcing vibration. A pendulum with a dense bob is more heavily damped than one with a less dense bob of the same size. Resonance can only occur in mechanical systems.

9 Two identical spheres exert a gravitational force F on each other. What is the gravitational force between two spheres, each twice the mass of one of the original spheres, when the separation of their centres is twice the original separation? A B C D F 2F 4F 8F

10 A planet of mass M and radius R rotates so rapidly that loose material at the equator only just remains on the surface. What is the period of rotation of the planet? G is the universal gravitational constant. 2 2 2 2 R GM R2 GM GM R3 R3 GM

WMP/Jan10/PHYA4/1

5 11 The radius of a certain planet is x times the radius of the Earth and its surface gravitational field strength is y times that of the Earth. Which one of the following gives the ratio A B C D xy x2y xy2 x2y2

mass of the planet ? mass of the Earth

12 Which one of the following could be a unit of gravitational potential? A B C D N J N kg1 J kg1

13 +Q 2Q

d The diagram shows two particles at a distance d apart. One particle has charge +Q and the other 2Q. The two particles exert an electrostatic force of attraction, F, on each other. Each particle is then given an additional charge +Q and their separation is increased to a distance 2d. Which one of the following gives the force that now acts between the two particles? F 4 F a repulsive force of 4 F an attractive force of 2 an attractive force of a repulsive force of F 2

A B C D

Turn over
WMP/Jan10/PHYA4/1

6 14 Which one of the following statements about a charged particle in an electric field is correct? A B C D No work is done when a charged particle moves along a field line. No force acts on a charged particle when it moves along a field line. No work is done when a charged particle moves along a line of constant potential. No force acts on a charged particle when it moves along a line of constant potential.

15 Two parallel metal plates separated by a distance d have a potential difference V across them. What is the magnitude of the electrostatic force acting on a charge Q placed midway between the plates? d 2 Q A B 2VQ d VQ d VQ 2d Qd V d 2

16 Which one of the following statements about electric field strength and electric potential is incorrect? A B C D Electric potential is a scalar quantity. Electric field strength is a vector quantity. Electric potential is zero whenever the electric field strength is zero. The potential gradient is proportional to the electric field strength.

WMP/Jan10/PHYA4/1

7 17 particle P Gold nucleus

An particle travels towards a gold nucleus and at P reverses its direction. Which one of the following statements is incorrect? A B C D The electric potential energy of the particle is a maximum at P. The kinetic energy of the particle is a minimum at P. The total energy of the particle is zero. The total energy of the particle has a constant positive value.

18 The graph shows how the potential difference across a capacitor varies with the charge stored by it.

potential difference

0 0

charge

Which one of the following statements is correct? A B C D The gradient of the line equals the capacitance of the capacitor. The gradient of the line equals the energy stored by the capacitor. The reciprocal of the gradient equals the energy stored by the capacitor. The reciprocal of the gradient equals the capacitance of the capacitor.

19 An initially uncharged capacitor of capacitance 10 F is charged by a constant current of 200 A. After what time will the potential difference across the capacitor be 2000 V? A B C D 50 s 100 s 200 s 400 s

Turn over
WMP/Jan10/PHYA4/1

8 20 A 1000 F capacitor, X, and a 100 F capacitor, Y, are charged to the same potential difference. Which row, A to D, in the table gives correct ratios of charge stored and energy stored by the capacitors? charge stored by X charge stored by Y A B C D 1 1 10 10 energy stored by X energy stored by Y 1 10 1 10

21 cross-sectional area 1.0 10 6 m2 current 8.0 A 0.40 m A current of 8.0 A is passed through a conductor of length 0.40 m and cross-sectional area 1.0 106 m2. The conductor contains 8.0 1028 free electrons per m3. When the conductor is at right angles to a magnetic field of flux density 0.20 T, it experiences a magnetic force. What is the average magnetic force that acts on one of the free electrons in the wire? A B C D 8.0 1030 N 5.0 1029 N 8.0 1024 N 2.0 1023 N 8.0 1028 free electrons per m3

WMP/Jan10/PHYA4/1

9 22 An electron moves due North in a horizontal plane with uniform speed. It enters a uniform magnetic field directed due South in the same plane. Which one of the following statements concerning the motion of the electron in the magnetic field is correct? A B C D It accelerated due West. It slows down to zero speed and then accelerates due South. It continues to move North with its original speed. It is accelerated due North.

23 Particles of mass m, each carrying charge Q and travelling with speed v, enter a magnetic field of flux density B at right angles. Which one of the following changes would produce an increase in the radius of the path of the particles? A B C D an increase in Q an increase in m a decrease in v an increase in B

24 The magnetic flux through a coil of N turns is increased uniformly from zero to a maximum value in a time t. An emf, E, is induced across the coil. What is the maximum value of the magnetic flux through the coil? Et N N Et EtN E Nt

25 An aircraft, of wing span 60 m, flies horizontally at a speed of 150 m s1, If the vertical component of the Earths magnetic field in the region of the plane is 1.0 105 T, what emf is induced across the wing tips of the plane? A B C D 0.09 V 0.90 V 9.0 V 90 V

END OF QUESTIONS

WMP/Jan10/PHYA4/1

10 There are no questions printed on this page

WMP/Jan10/PHYA4/1

11 There are no questions printed on this page

WMP/Jan10/PHYA4/1

12 There are no questions printed on this page

Copyright 2008 AQA and its licensors. All rights reserved.

WMP/Jan10/PHYA4/1

Centre Number Surname Other Names Candidate Signature

Candidate Number

For Examiners Use

Examiners Initials

Question

Mark

General Certificate of Education Advanced Level Examination January 2010

1 2 3

Physics A
Unit 4 Fields and Further Mechanics Section B Thursday 28 January 2010
For this paper you must have: G a calculator G a ruler G a Data and Formulae Booklet.

PHYA4/2

4 TOTAL

1.30 pm to 3.15 pm

Time allowed G The total time for both sections of this paper is 1 hour 45 minutes. You are advised to spend approximately one hour on this section. Instructions G Use black ink or black ball-point pen. G Fill in the boxes at the top of this page. G Answer all questions. G You must answer the questions in the spaces provided. Answers written in margins or on blank pages will not be marked. G Do all rough work in this book. Cross through any work you do not want to be marked. Information G The marks for questions are shown in brackets. G The maximum mark for this section is 50. G You are expected to use a calculator where appropriate. G A Data and Formulae Booklet is provided as a loose insert. G You will be marked on your ability to: use good English organise information clearly use specialist vocabulary where appropriate.

(JAN10PHYA4201)

WMP/Jan10/PHYA4/2

PHYA4/2

2 Answer all questions. You are advised to spend approximately one hour on this section.

Areas outside the box will not be scanned for marking

(a) Describe the energy changes that take place as the bob of a simple pendulum makes one complete oscillation, starting at its maximum displacement. ............................................................................................................................................. ............................................................................................................................................. ............................................................................................................................................. ............................................................................................................................................. (2 marks)

(b)

Figure 1

Figure 1 shows a young girl swinging on a garden swing. You may assume that the swing behaves as a simple pendulum. Ignore the mass of chains supporting the seat throughout this question, and assume that the effect of air resistance is negligible. 15 complete oscillations of the swing took 42s. 1 (b) (i) Calculate the distance from the top of the chains to the centre of mass of the girl and seat. Express your answer to an appropriate number of significant figures.

answer = .......................... m (4 marks)

(02)

WMP/Jan10/PHYA4/2

3 1 (b) (ii) To set her swinging, the girl and seat were displaced from equilibrium and released from rest. This initial displacement of the girl raised the centre of mass of the girl and seat 250 mm above its lowest position. If the mass of the girl was 18 kg, what was her kinetic energy as she first passed through this lowest point?

Areas outside the box will not be scanned for marking

answer = ............................ J (2 marks) 1 (b) (iii) Calculate the maximum speed of the girl during the first oscillation.

answer = ..................... m s1 (1 mark) 1 (c) Figure 2

kinetic energy

0 0 T/2 T time

On Figure 2 draw a graph to show how the kinetic energy of the girl varied with time during the first complete oscillation, starting at the time of her release from maximum displacement. On the horizontal axis of the graph, T represents the period of the swing. You do not need to show any values on the vertical axis. (3 marks)

12
Turn over (03)

WMP/Jan10/PHYA4/2

4 2 (a) A capacitor, initially charged to a pd of 6.0 V, was discharged through a 100 k resistor. A datalogger was used to record the pd across the capacitor at frequent intervals. The graph shows how the pd varied with time during the first 40 s of discharge.

Areas outside the box will not be scanned for marking

4 pd/V

0 0 10 20 time/s 30 40

(a)

(i) Calculate the initial discharge current.

answer = ........................... A (1 mark) 2 (a) (ii) Use the graph to determine the time constant of the circuit, giving an appropriate unit.

answer = ............................... (4 marks)

(04)

WMP/Jan10/PHYA4/2

5 2 (a) (iii) Hence calculate the capacitance of the capacitor.

Areas outside the box will not be scanned for marking

answer = .................................. F (1 mark) 2 (a) (iv) Show that the capacitor lost 90% of the energy it stored originally after about 25 s.

(3 marks) 2 (b) In order to produce a time delay, an intruder alarm contains a capacitor identical to the capacitor used in the experiment in part (a). This capacitor is charged from a 12 V supply and then discharges through a 100 k resistor, similar to the one used in the experiment. (b) (i) State and explain the effect of this higher initial pd on the energy stored by this capacitor initially. ................................................................................................................................... ................................................................................................................................... ................................................................................................................................... (2 marks) 2 (b) (ii) State and explain the effect of this higher initial pd on the time taken for this capacitor to lose 90% of its original energy. ................................................................................................................................... ................................................................................................................................... ................................................................................................................................... (1 mark)

12
Turn over (05)

WMP/Jan10/PHYA4/2

6 3 (a) (i) State the relationship between the gravitational potential energy, Ep, and the gravitational potential, V, for a body of mass m placed in a gravitational field. ................................................................................................................................... ................................................................................................................................... (1 mark) 3 (a) (ii) What is the effect, if any, on the values of Ep and V if the mass m is doubled? value of Ep ................................................................................................................ value of V ................................................................................................................. (2 marks) 3 (b) Figure 3 A B E

Areas outside the box will not be scanned for marking

Figure 3 shows two of the orbits, A and B, that could be occupied by a satellite in circular orbit around the Earth, E. The gravitational potential due to the Earth of each of these orbits is: orbit A orbit B 3 (b) 12.0 MJ kg1 36.0 MJ kg1.

(i) Calculate the radius, from the centre of the Earth, of orbit A.

answer = .......................... m (2 marks)

(06)

WMP/Jan10/PHYA4/2

Areas outside the box will not be scanned for marking

(b)

(ii) Show that the radius of orbit B is approximately 1.1 104 km.

(1 mark) 3 (b) (iii) Calculate the centripetal acceleration of a satellite in orbit B.

answer = ..................... m s2 (2 marks) 3 (b) (iv) Show that the gravitational potential energy of a 330 kg satellite decreases by about 8 GJ when it moves from orbit A to orbit B.

(1 mark) 3 (c) Explain why it is not possible to use the equation Ep = mgh when determining the change in the gravitational potential energy of a satellite as it moves between these orbits. ............................................................................................................................................. ............................................................................................................................................. ............................................................................................................................................. ............................................................................................................................................. (1 mark)

10
Turn over (07)

WMP/Jan10/PHYA4/2

8 4 (a) (i) Outline the essential features of a step-down transformer when in operation. ................................................................................................................................... ................................................................................................................................... ................................................................................................................................... ................................................................................................................................... ................................................................................................................................... (2 marks) 4 (a) (ii) Describe two causes of the energy losses in a transformer and discuss how these energy losses may be reduced by suitable design and choice of materials. The quality of your written communication will be assessed in this question. ................................................................................................................................... ................................................................................................................................... ................................................................................................................................... ................................................................................................................................... ................................................................................................................................... ................................................................................................................................... ................................................................................................................................... ................................................................................................................................... ................................................................................................................................... ................................................................................................................................... ................................................................................................................................... ................................................................................................................................... ................................................................................................................................... ................................................................................................................................... ................................................................................................................................... ...................................................................................................................................

Areas outside the box will not be scanned for marking

(08)

WMP/Jan10/PHYA4/2

Areas outside the box will not be scanned for marking

................................................................................................................................... ................................................................................................................................... ................................................................................................................................... ................................................................................................................................... ................................................................................................................................... ................................................................................................................................... ................................................................................................................................... ................................................................................................................................... (6 marks) 4 (b) Electronic equipment, such as a TV set, may usually be left in standby mode so that it is available for instant use when needed. Equipment left in standby mode continues to consume a small amount of power. The internal circuits operate at low voltage, supplied from a transformer. The transformer is disconnected from the mains supply only when the power switch on the equipment is turned off. This arrangement is outlined in Figure 4. Figure 4 power switch mains supply at 230V transformer to internal circuits at 9.0V

When in standby mode, the transformer supplies an output current of 300 mA at 9.0 V to the internal circuits of the TV set. 4 (b) (i) Calculate the power wasted in the internal circuits when the TV set is left in standby mode.

answer = .......................... W (1 mark) Question 4 continues on the next page Turn over (09)

WMP/Jan10/PHYA4/2

10 4 (b) (ii) If the efficiency of the transformer is 0.90, show that the current supplied by the 230 V mains supply under these conditions is 13 mA.

Areas outside the box will not be scanned for marking

(2 marks) 4 (b) (iii) The TV set is left in standby mode for 80% of the time. Calculate the amount of energy, in J, that is wasted in one year through the use of the standby mode.
7 1 year = 3.15 10 s

answer = ............................ J (1 mark) 4 (b) (iv) Show that the cost of this wasted energy will be about 4, if electrical energy is charged at 20 p per kWh.

(2 marks) 4 (c) The power consumption of an inactive desktop computer is typically double that of a TV set in standby mode. This waste of energy may be avoided by switching off the computer every time it is not in use. Discuss one advantage and one disadvantage of doing this. ............................................................................................................................................. ............................................................................................................................................. ............................................................................................................................................. ............................................................................................................................................. ............................................................................................................................................. ............................................................................................................................................. (2 marks) END OF QUESTIONS

16

(10)

WMP/Jan10/PHYA4/2

11 There are no questions printed on this page

DO NOT WRITE ON THIS PAGE ANSWER IN THE SPACES PROVIDED

(11)

WMP/Jan10/PHYA4/2

12 There are no questions printed on this page

DO NOT WRITE ON THIS PAGE ANSWER IN THE SPACES PROVIDED

Copyright 2010 AQA and its licensors. All rights reserved.

(12)

WMP/Jan10/PHYA4/2

Physics A PHYA4 - AQA GCE Mark Scheme 2010 January series

GCE Physics, Specification A, PHYA4, Fields and Further Mechanics Section A This component is an objective test for which the following list indicates the correct answers used in marking the candidates responses. Keys to Objective Test Questions 1 C 14 C Section B Question 1 (a) (grav) potential energy ! kinetic energy ! (grav) potential energy ! kinetic energy ! gravitational potential energy ! energy lost to surroundings in overcoming air resistance ! (b) (i) period T = ! & = 2.8 s !
$% "#

2 B 15 B

3 D 16 C

4 A 17 C

5 D 18 D

6 B 19 B

7 B 20 D

8 A 21 D

9 A

10 D 22 C

11 B 23 B

12 D 24 A

13 A 25 A

use of T = 2! ' gives length l = !*


)

+,) "-

,& =

#.., / 0..$ "-,

giving distance from pt of support to c of m, l = 1.9 (m) or 1.95 (m) ! answer must be to 2 or 3 sf only ! (b) (ii) Ek = mg"h stated or used ! gives Ek of girl at lowest point = 18 9.81 0.25 = 44 (J) ! (b) (iii) mv2 = 44.1 gives max speed of girl v = ' [alternatively: and (c)
# / "".$ $.

= 2.2 (m s-1) ! 1

A2 = (3.9 0.25) 0.25 gives A = 0.955 (m) vmax = 2! f A = (2!/2.8) 0.955 = 2.1 (m s-1) !]

graph drawn on Figure 2 which: shows Ek = 0 at t = 0, T/2 and T ! has 2 maxima of similar size (some attenuation allowed) at T/4 and 3T/4 ! is of the correct general shape ! Total 12 3

Physics A PHYA4 - AQA GCE Mark Scheme 2010 January series

Question 2 (a) (a) (i) (ii) initial discharge current !*


1 2

3.4 $.4 / $45

& = 6.0 10-5 (A) !

time constant is time for V to fall to (1/e) [or 0.368] of initial value ! pd falls to (6.0/e) = 2.21 V when t =time constant ! reading from graph gives time constant = 22 ( 1) ! unit: s ! ("F not acceptable) [alternatively accept solutions based on use of V = V0e-t/RC eg 1.5 = 6.0 e-30/RC ! gives RC =
64 78 93.4/$.%:

!= 22 ! s!]

(a)

(iii)

capacitance of capacitor C = !

;<=> ?@8A;B8; C

## $.4 / $45

&

= 2.2 10-4 (F) = 220 (F) ! (a) (iv) energy V2 (or energy = CV2) !
D, DE

= 0.10 gives =

1, 1E

(0.10)1/2 ! (= 0.316)

V2 = 0.316 6.0 = 1.90 (V) ! reading from graph gives V2 = 1.90 V when t = 25 s ! [alternatively accept reverse argument: ie when t = 25 s, V2 = 1.9 V from graph ! final energy stored = 2.2 10-4 1.92 = 3.97 10 (J) and initial energy stored = 3.96 10 (J) ! which is 10 greater, so 90% of initial energy has been lost !] [alternatively, using exponential decay equation: use of V = V0e-t/R with t = 25 s and RC = 22 s gives V = 1.93 V ! energy V2 (or energy = CV2) gives
D, DE
-4 -3

*!
DE

$.06 # 3.4

& = 0.103 !
D, DE

fraction of stored energy that is lost = (b) (i) initial energy stored is 4 greater ! because energy V2 (and V is doubled) ! (b) (ii)

D, FDE

=1

= 0.90 !] 2 1 Total 12

time to lose 90% of energy is unchanged because time constant is unchanged (or depends only on R and C) !

Physics A PHYA4 - AQA GCE Mark Scheme 2010 January series

Question 3 (a) (a) (i) (ii) relationship between them is Ep = mV (allow "Ep = m"V) [or V is energy per unit mass (or per kg)] ! value of Ep is doubled ! value of V is unchanged ! (b) (i) use of V =
GH I

1 2

gives rA =

3.3J / $4KEE / %.0. / $4,L $#.4 / $4M

= 3.3(2) 107 (m) ! (b) (ii) since V ()


$ I

!NO

IP IQ
4

1Q 1P

63.4 $#.4

* 3& rB =

6.6# / $4R S 6

(which is # 1.1 10 km) (b) (iii) centripetal acceleration gB =


GH IQ,

3.3J / $4KEE / %.0. / $4,L 9$.$$ / $4R :,

[allow use of 1.1 107 m from (b) (ii)] = 3.2 (m s-2) ! [alternatively, since gB = () = 3.2 (m s-2) !] (b) (iv) use of "Ep = m"V gives "Ep = 330 (12.0 (36.0)) 106 ! (which is 7.9 109 J or # 8 GJ) (c) g is not constant over the distance involved (or g decreases as height increases or work done per metre decreases as height increases or field is radial and/or not uniform) ! Total 1 1
1Q 1T

2 , gB =
63.4 / $4M $.$$ / $4R

10

Physics A PHYA4 - AQA GCE Mark Scheme 2010 January series

Question 4 (a) (i) primary coil with more turns than secondary coil ! (wound around) a core or input is ac ! (a) (ii) QWC the mark scheme for this part of the question includes an overall assessment for the Quality of Written Communication descriptor Two causes of energy losses are clearly identified, correct measures to indicate how these two losses may be reduced are stated and a detailed physical explanation of why these measures are effective is given. eg any two from the following four 1 When a transformer is in operation, there are ac currents in the primary and secondary coils. The coils have some resistance and the currents cause heating of the coils, causing some energy to be lost. This loss may be reduced by using low resistance wire for the coils. This is most important for the high current winding (the secondary coil of a step-down transformer). Thick copper wire is used for this winding, because thick wire of low resistivity has a low resistance. The ac current in the primary coil magnetises, demagnetises and remagnetises the core continuously in opposite directions. Energy is required both to magnetise and to demagnetise the core and this energy is wasted because it simply heats the core. The energy wasted may be reduced by choosing a material for the core which is easily magnetised and demagnetised, ie a magnetically soft material such as iron, or a special alloy, rather than steel. The magnetic flux passing through the core is changing continuously. The metallic core is being cut by this flux and the continuous change of flux induces emfs in the core. In a continuous core these induced emfs cause currents known as eddy currents, which heat the core and cause energy to be wasted. The eddy current effect may be reduced by laminating the core instead of having a continuous solid core; the laminations are separated by very thin layers of insulator. Currents cannot flow in a conductor which is discontinuous (or which has a very high resistance). If a transformer is to be efficient, as much as possible of the magnetic flux created by the primary current must pass through the secondary coil. This will not happen if these coils are widely separated from each other on the core. Magnetic losses may be reduced by adopting a design which has the two coils close together, eg by better core design, such as winding them on top of each other around the same part of a common core which also surrounds them. 3-4 mark range 2

good excellent 3

5-6

modest adequate poor limited incorrect, inappropriate or no response

Up to two sources of energy losses are stated and there is an indication of how these may be minimised by suitable features or materials. There is no clear appreciation of an understanding of the physical principles to explain why these measures are effective. Up to two sources of energy losses are given, but the answer shows no clear understanding of the measures required to minimise them.

12

There is no answer or the answer presented is irrelevant or incorrect.

Answers which address only one acceptable energy loss should be marked using the same principles, but to max 3.

Physics A PHYA4 - AQA GCE Mark Scheme 2010 January series

(b) (b)

(i) (ii)

power wasted internally (= I V) = 0.30 9.0 = 2.7 (W) ! input power !*


#.J 4.04

& = 3.0 (W) !


2

mains current !* (b) (b) (iii) (iv)

6.4

#64

& ! (= 1.30 10-2 A)


1 2

energy wasted per year (= P t) = 3.0 0.80 3.15 107 = 7.5(6) 107 (J) ! energy wasted =
J.%3 / $4R 6.3 / $4M

= 21.0 (kWh) !

cost of wasted energy = 21.0 20 = 420p (4.20) ! (c) answers should refer to: an advantage of switching off ! cost saving, saving essential fuel resources, reduced global warming etc 2

a disadvantage of switching off ! inconvenience of waiting, time taken for computer to reboot etc risk of computer failure increased by repeated switching on and off energy required to reboot may exceed energy saved by switching off Total

16

GCE Physics A PHYA4 - AQA GCE Report on the Examination 2010 January series

GCE Physics, Specification A, PHYA4, Fields and Further Mechanics General Comments This first January sitting for the unit 4 test of the Physics A specification was taken by more than 9000 candidates. Under this new specification, Section A (on which candidates are advised to spend around 45 minutes) consists of 25 objective test questions worth 1 mark each. Section B (on which candidates are advised to spend around 1 hour) is worth 50 marks and on this occasion contained four questions. Section A followed in the tradition of previous objective tests, although the number of questions has been increased from the 15 used latterly in the previous unit 4 tests. The mean pre-test facility and point biserial values for this test were closely similar to those for recent objective tests on the legacy Physics A specification. The mean values of facility and discrimination index from the candidates responses on Section A were in line with expectations. In Section B, the examiners were pleased to see some competent work from many of the candidates. Good work was well rewarded and the average mark for each section of the test was highly satisfactory. There was evidence that some candidates found the overall test demanding and were short of time towards the end of Section B. Section A Keys to Objective Test Questions 1 C 14 C 2 B 15 B 3 D 16 C 4 A 17 C 5 D 18 D 6 B 19 B 7 B 20 D 8 A 21 D 9 A 10 D 22 C 11 B 23 B 12 D 24 A 13 A 25 A

The facility of a question is a measure of all candidates attempting a question who choose the correct option. The mean facility of this paper was 64%. The facility for individual questions ranged from 87% for question 4 to 35% for question 21. For the purpose of monitoring standards over time, objective tests contain a proportion of questions that are re-banked after satisfactory use in an earlier examination. This test contained eight such questions, with a mean facility of 50% when last used. The eighteen new questions had all been pre-tested and had a mean pre-test facility of 54%. Candidates invariably produce higher facilities for the questions in a real examination than in the pre-testing situation. The mean facility of all of the re-banked questions also all improved, by an average of 10%. The point biserial index of a question is a measure of how well the question discriminates between the most able and the least able candidates. The mean point biserial for this paper was 0.39. The new questions had a mean pre-test point biserial of 0.37, whilst the value for the re-banked questions was 0.41. Therefore there was no discernable improvement in the discrimination of the questions; in fact most of the re-banked questions gave inferior discrimination this time to that recorded when last used. Thirteen of the questions (questions 1, 3, 4, 9, 12, 13, 14, 18, 19, 20, 24 and 25) proved to be easy, with facilities over 65%, whilst no question was found to be difficult. Question 1, on factual knowledge of the impulse momentum relationship, was an easy starter with a facility of 85%.

GCE Physics A PHYA4 - AQA GCE Report on the Examination 2010 January series

Question 2 required candidates to determine the momentum of the water flowing out of a garden hose in one second. This called for mathematical application as well as knowledge and it was therefore much more demanding. 41% of the candidates selected the correct answer, and the question was not a strong discriminator. The most popular incorrect distractor, chosen by 28%, was C (0.20). This numerical value could be found by multiplying the density of water by the flow rate, ignoring the cross-sectional area value given in the question. Like the first question, Question 3 was a straightforward test of candidates knowledge. It required candidates to decide whether or not mass, momentum, kinetic energy and total energy would be conserved in an inelastic collision. 85% of the candidates appreciated that everything except kinetic energy would be conserved. Incorrect responses were fairly evenly spread around the other three distractors. Question 4 proved to be the easiest question, with a facility of 87%. Application of = 2/T with T equal to the period of Earths rotation readily gave the correct answer. Question 5 asked candidates to identify a situation in which centripetal force would not be involved, so they ought to have known to look for the answer that did not involve circular motion. 61% realised that this was the particle in an electric field, where the trajectory would be parabolic rather than circular. However, 30% of the candidates chose distractor C, where the particle was in a magnetic field. Question 6, about phase differences in shm, had a facility of 72% but did not discriminate very well. 17% chose distractor A (/4 instead of /2); this may have been caused by a misunderstanding of the radian to degree conversion. Question 7 was concerned with the amplitude and period of a mass-spring system. The facility was 63%, but one in five of the candidates selected distractor A where the amplitude was correct but the period was 1.0s instead of 2.0s. Answers in Section B also showed that there was widespread misunderstanding of what is meant by the time taken for one oscillation. Question 8, on forced vibrations, had a facility of 59% and did not discriminate very well. Distractor B, where a phase relation was involved, attracted 23% of the candidates. This again may be an indication of a misunderstanding of phase angles, because the angle in the situation described is 90, not 180. Questions 9 and 10 were about gravitational forces. Application of the inverse square law was completed successfully by 70% of the candidates in the former question. In Question 10, candidates had to appreciate that the condition described would be met when the centripetal force acting on material is just equal to its weight, so 2R = GM/R2. Only 48% of them were successful, but the question discriminated very well. The correct algebraic rearrangement of g = GM/R2 would deliver a correct answer in Question 11, achieved by 62% of the candidates. The unit of gravitational potential was known correctly by 71% of them in Question 12. However, one in five selected distractor C N kg1 which is the unit of gravitational field strength. Coulombs law had to be applied in Question 13. 66% of the candidates realised that doubling the separation would have the effect of reducing the force by a factor of four, whilst the changes to the charges would mean that they would become +2Q and Q, so that the force would remain one of attraction. Distractor C was selected by 22% of candidates; this could be because they thought that F 1/r instead of F 1/r2.

GCE Physics A PHYA4 - AQA GCE Report on the Examination 2010 January series

Question 14, about a charged particle moving in an electric field, had a facility of 66% and was a good discriminator. Incorrect responses were almost equally distributed between the incorrect distractors. Question 15 required candidates to apply F = EQ in a charged parallel plate situation. 58% of them appreciated that the separation of the plates was d and that the field strength E would be V/d. However, 29% chose distractor A, for which the field strength must have been interpreted as V (d/2). Question 16, also on electrostatics, was concerned with field strength and potential. This was the worst discriminator in the test, and only 43% of candidates selected the correct response. A principal reason for as many as 24% of them choosing distractor A (potential is a scalar) must be that they had failed to notice that the question asked for the incorrect statement. The statement that is incorrect was also to be chosen in Question 17, about the energy of an particle during a head-on encounter with a gold nucleus. The facility of this question was 62%, the most common incorrect choice being distractor D (19%). Question 18 was a test of C = Q/V in a graphical application. Because Q is on the x-axis and V on the yaxis, the gradient is 1/C (answer D). The majority of candidates recognised this, making the 72%. It may not be surprising that distractor A was the most popular incorrect response, chosen by 15%, since this suggests that the gradient would be C. This question was another good discriminator. 78% of candidates were not troubled by the constant charging current of the capacitor in Question 19, because they chose the correct answer. This question required Q = I t to be combined with C = Q/V. Question 20 was also a test of C = Q/V, but in combination with E = CV2. Candidates found this question also to be relatively easy, for the facility was 72% but it was the most discriminating question in the test. Question 21 was the most demanding question, with a facility of 35%. It required a fundamental understanding of charge flow in conductors and the magnetic force equations. The required average force per electron could be found by dividing the force on the whole section of conductor (F = B I L) by the number of electrons in it (volume 8.0 1028). This was another question that discriminated well. The magnetic force on a moving electron was also tested in Question 22, but this time qualitatively instead of quantitatively. Since this electron was moving anti-parallel to the magnetic field through which it was travelling, it would experience no magnetic force. Therefore, its motion would not be affected by the B field. 45% of candidates appreciated this (answer C), but 21% thought it would grind to a halt and set off in the opposite direction (distractor B) whilst 18% thought it would accelerate in its original direction (distractor D). Confusion with the effects of electric fields is evident in these incorrect responses. Question 23 looked at factors that might increase the radius of curvature of charged particles following a circular arc in a B field. The facility of this question was 63%, and incorrect responses were fairly equally spread. Question 24 moved on to electromagnetic induction and tested E = N /t for a uniform rate of change of magnetic flux. 72% of the responses were correct. Like questions 22 and 23, this question was a very good discriminator. The uniform rate of change of flux experienced by an aircraft wing in steady horizontal flight, leading to an emf across the wing tips, was considered in Question 25. This had a facility of 71%. No doubt arithmetical errors were the cause of 13% of the candidates choosing distractor B, and 11% choosing distractor C.

Section B

GCE Physics A PHYA4 - AQA GCE Report on the Examination 2010 January series

The questions provided good coverage of the specification and gave candidates ample opportunities to show what they had learned. It was pleasing to see many good answers to questions which required fairly demanding mathematics, such as the solution of exponential equations by logarithms in question 2. The specification for this test includes a requirement for some of the content to be synoptic. A topic which comes into this category is cost of energy calculations (question 4 (b) (iv)) and this caused much difficulty for many of the candidates. When marking all parts of the paper, there were examples of candidates who had not read the questions sufficiently carefully to be able to answer them properly. This was particularly evident in descriptive parts. In the answers to question 1 (a), for example, many answers were encountered which did not even contain the word energy. The most important advice to candidates for this paper, as always, is therefore to read the questions thoroughly before attempting to answer them. Several of the calculations were answered competently, but less able candidates often presented their working in such a disorganised manner that examiners had great difficulty in interpreting their thinking. Candidates need to be informed that working is expected to be shown in calculations, and that specific marking points are often available for working. This enables examiners to award marks for partially correct calculations, where the final answer may well be wrong as a consequence of an arithmetic slip. Question 4 (a) (ii), on energy losses in transformers, was used to assess candidates quality of written communication by a method new to A2 level. Most answers were only partly satisfactory for one reason or another, one of the main ones being a lack of familiarity with and/or understanding of the subject matter. Although this topic is new to this specification at this level, it did once occupy a prominent place in most GCSE specifications. Question 1 In part (a), the award of the full two marks was comparatively rare. Most answers were incomplete because candidates had not addressed the need to describe the energy changes of the bob over one complete oscillation, starting at its maximum displacement. A large proportion of candidates confined their attention to the first half of the oscillation, which limited them to half marks. Another error was a reluctance to refer to the potential energy as gravitational. Some candidates missed the point of the question completely, and wrote about velocity and acceleration in shm. Calculation of the period of the swing in part (b) (i) was straightforward, and proved to be rewarding for most candidates. Those who confused period with frequency gained little credit, except for the mark for giving a final answer to an appropriate number of significant figures. Using the given data, the answer for the length was 1.948 m, when calculated to four significant figures. Final answers of 2.0 m (rather than 1.9 m) were therefore regarded as incorrect. The solution to part (b) (ii), where the maximum Ek of the girl was needed, came readily from Ek gained = gravitational Ep lost. Equating this result to m v2 then led to a neat solution to part (b) (iii), to find the maximum speed of the girl. Many candidates attempted much more tortuous routes to parts (ii) and/or (iii), using vmax = 2 f A. The principal downfall of this method (quite apart from its relative difficulty) was the adoption of 250 mm for the amplitude, A. Some successful solutions by the method were seen, however, where the correct value for A had been found by Pythagoras, or some equivalent calculation. Many reasonable graphs were drawn in part (c), where the Ek against t graph was required, starting at maximum displacement. The majority of answers recognised that Ek would be zero at t = 0, T/2 and T. On most answers there were also correct maxima, of similar amplitude, within one square of T/4 and 3T/4 on the graph. The most demanding aspect was the shape of the graph; half wave rectified waveforms tended to dominate, whilst triangular waveforms were by no means uncommon. Correct (sin2) shapes

GCE Physics A PHYA4 - AQA GCE Report on the Examination 2010 January series

were comparatively rare, but credit was given for any shape which showed appropriately curved characteristics. Question 2 Most candidates were able to calculate the initial discharge current successfully in part (a) (i). The common approaches to finding the time constant in part (a) (ii) were reading from the graph at the point where the pd had fallen to 6.0/e, or solving the exponential equation V = V0et/RC for corresponding V and t values. It was expected that candidates would know that a time constant is measured in s; the unit F was not accepted. The principal difficulty experienced by some candidates in part (a) (iii) was not spotting that the capacitance value had to be expressed in F. Answers of 2.2 104 F were clearly wrong and caused this mark to be lost. A wide variety of approaches could be adopted when answering part (a) (iv). Most candidates attempted to answer the question in reverse, by showing that after 25 s the energy lost would be 90% of the original; this was acceptable. Some lost the third mark when using this method by failing to link the two energies (calculated correctly) to the 90% value for the loss. A neat and concise solution was seen in a few cases, where candidates reasoned that, since E V2, the percentage loss of energy would be [1 (V/V0)2] 100. In part (b) (i) one mark was awarded for a correct consequence of the increased charging pd, and one mark for an explanation. The fact that E V2 was quite well known, and it was expected that candidates would realise that doubling the pd would quadruple the energy stored: to state just that the energy stored would increase was too simplistic to deserve this mark. Candidates who resorted to E = QV almost invariably reached the wrong conclusion, because they thought that the energy stored would double. In part (b) (ii) the one available mark was given for a correct consequence together with an acceptable explanation. Relatively few candidates were able to state that the time taken for 90% of the energy to be lost would be unchanged because the time constant had not altered. Question 3 Many very good answers were seen in part (a) (i), expressed either fully in words or simply by quoting Ep = mV. The corresponding equation for an incremental change, Ep = mV, was also acceptable but mixed variations on this such as Ep = mV (which showed a lack of understanding) were not. The consequences of doubling m were generally well understood in part (a) (ii), where most candidates scored highly, but some inevitably thought that Ep would be unchanged whilst V would double. Candidates who were not fully conversant with the metric prefixes used with units had great difficulty in part (b), where it was necessary to know that 1 MJ =106 J, 1 GJ =109 J, and (even) 1 km = 103 m. Direct substitution into V = () GM/r (having correctly converted the value of V to J kg1) usually gave a successful answer for the radius of orbit A in part (b) (i). A similar approach was often adopted in part (b) (ii) to find the radius of orbit B, although the realisation that V 1/r facilitated a quicker solution. Some candidates noticed that VB = 3 VA and guessed that rB = rA/3, but this was not allowed when there was no physical reasoning to support the calculation. Part (b) (iii) caused much difficulty, because candidates did not always appreciate that the centripetal acceleration of a satellite in stable orbit is equal to the local value of g, which is equal to GM/r2. This value turns out to equal to V/r, which provided an alternative route to the answer. Many incredible values were seen, some of them greatly exceeding 9.81 m s2.

GCE Physics A PHYA4 - AQA GCE Report on the Examination 2010 January series

Part (c) was generally well understood, with some very good and detailed answers from the candidates. Alternative answers were accepted: either that g is not constant over such large distances, or that the field of the Earth is radial rather than uniform. Question 4 In part (a) (i), the requirement that NP > NS was regarded as fundamental for the first mark; the second mark was awarded for either a core linking the coils or an ac supply. Some candidates missed the point of the question completely by writing about how transformers change voltages rather than concentrating on their essential features. Most attempts to answer part (a) (ii) fell well short of examiners expectations. In many cases the principal cause of this was a lack of detailed knowledge or confused understanding. When this kind of question is employed to assess the quality of candidates written communication, it will be expected that a good answer will be structured: well organised, and coherent. This was another general error, because candidates often presented answers that rambled on without general direction. Successful answers were primarily expected to address two (and only two) causes of energy loss; four causes are commonly identified by the standard sources that deal with this topic, so asking for two was not unduly demanding. For each cause that was identified, there was a requirement to discuss how the loss could be reduced by suitable features and materials. Answers that could be placed in the good to excellent category (five or six marks) should have backed up this factual knowledge with some physical reasoning. Very few answers addressed these requirements sufficiently successfully to deserve the award of full marks. It seemed that most of the candidates had heard about eddy currents, but not all of them knew exactly what they are or where they occur. Many answers showed considerable confusion; eddy currents caused by the coils can be stopped by using a soft iron core, heat losses from the currents can be reduced by using smaller currents, and energy losses from re-magnetising the core can be cut down by laminating it were typical of the confused responses seen. A proportion of the candidates evidently mixed up the principles of energy loss reduction in transformers with the principles involved in reducing power losses from transmission cables, because there were frequent references to using higher voltages in the transformers in order to reduce the currents causing the heating. The calculations in parts (b) (i) and (ii) were usually correct, with P = I V and the transformer efficiency equation being successfully applied. Almost all of the answers to part (b) (iii) were incorrect, because candidates did not realise that when on standby the transformer, as well as the load, continues to waste energy. Consequently, the power wasted on standby was 3.0 W, not 2.7 W. This error did not prevent candidates from accessing both available marks in part (b) (iv), provided they correctly applied the physical principles there. In this part, relatively few completely correct answers were seen, largely because the candidates were unable to convert an energy value from J to kWh. In part (c) it was usual to award both marks; most candidates knew that it takes an appreciable time for a computer to boot up and this would therefore be a disadvantage of switching off. For the advantage, a more specific point than saves energy was being looked for, because this response does little more than re-state the question. Mark Ranges and Award of Grades Grade boundaries and cumulative percentage grades are available on the Results statistics page of the AQA Website.

Surname Centre Number Candidate Signature

Other Names Candidate Number

Leave blank

General Certificate of Education 2010 Advanced Examination


version 1.2

!"#
PHYA5
For Examiners Use Number Mark Number 3 4 Mark 1 2

PHYSICS A Unit 5 Nuclear and Thermal Physics Section A


SPECIMEN PAPER

Time allowed: 55 minutes


Instructions Use blue or black ink or ball-point pen. Fill in the boxes at the top of this page. Answer all questions. A Data and Formula Booklet is provided as a loose insert. Information The maximum mark for this paper is 40. The marks for the questions are shown in brackets. You are reminded of the need for good English and clear presentation in your answers. You will be assessed on your quality of written communication where indicated in the question.

Total (Column 1) Total (Column 2) TOTAL Examiners Initials

Section A The maximum mark for this section is 40 marks. You are advised to spend approximately 55 minutes on this section. 1 (a) Sketch, using the axes provided, a graph of neutron number, N, against proton number, Z, for stable nuclei over the range Z = 0 to Z = 80. Show suitable numerical values on the N axis.

(2 marks) (b) On the graph indicate, for each of the following, a possible position of a nuclide that might decay by (i) (ii) ! emission, labelling the position with W, "- emission, labelling the position with X, (3 marks)

(iii) "+ emission, labelling the position with Y.

(c)

Used fuel rods from a nuclear reactor emit "- particles from radioactive isotopes that were not present before the fuel rod was inserted in the reactor. Explain why "- emitting isotopes are produced when the fuel roads are in the reactor. ............................................................................................................................................. ............................................................................................................................................. ............................................................................................................................................. ............................................................................................................................................. (3 marks) A nuclear power station is a reliable source of electricity that does not produce greenhouse gases but it does produce radioactive waste. Discuss the relative importance of these features in deciding whether or not new nuclear power stations are needed. The quality of your written answer will be assessed in this question. ............................................................................................................................................. ............................................................................................................................................. ............................................................................................................................................. ............................................................................................................................................. ............................................................................................................................................. ............................................................................................................................................. ............................................................................................................................................. ............................................................................................................................................. ............................................................................................................................................. ............................................................................................................................................. (6 marks) Total 14 marks

(d)

Some liquids in open bottles deteriorate exposure to air. Figure 1 shows one device used to reduce this deterioration. It consists of a rubber valve that is inserted into the neck of the bottle together with a pump that is used to remove some of the air in the bottle through this rubber valve. On an up-stroke of the pump, air enters the pump chamber from the bottle. On the down-stroke, the rubber valve closes and the air in the chamber is expelled to the atmosphere through another valve (not shown) in the handle. Figure 1

(a)

There is 3.5 10-4 m3 of air space in the bottle and the volume of the pump chamber changes from zero at the beginning of the up-stroke to 6.5 10-4 m3 at the end of the up-stroke. The initial pressure of the air in the bottle is that of the atmosphere with a value of 99 kPa. Assuming the process is at constant temperature, calculate the pressure in the bottle after one up-stroke of the pump. ............................................................................................................................................. ............................................................................................................................................. ............................................................................................................................................. ............................................................................................................................................. (3 marks)

(b)

Calculate the number of molecules of air originally in the air space in the bottle at a temperature of 18 C. ............................................................................................................................................. ............................................................................................................................................. ............................................................................................................................................. ............................................................................................................................................. (3 marks)

(c)

Explain how the kinetic theory of an ideal gas predicts the existence of a gas pressure inside the bottle. Go on to explain why this pressure decreases when some of the air is removed from the bottle. ............................................................................................................................................. ............................................................................................................................................. ............................................................................................................................................. ............................................................................................................................................. ............................................................................................................................................. ............................................................................................................................................. ............................................................................................................................................. ............................................................................................................................................. ............................................................................................................................................. (5 marks) Total 11 marks

In an experiment to measure the temperature of the flame of a Bunsen burner, a lump of copper of mass 0.12 kg is heated in the flame for several minutes. The copper is then transferred quickly to a beaker, of negligible heat capacity, containing 0.45 kg of water, and the temperature rise of the water measured. Specific heat capacity of water = 4200 J kg-1 K-1 Specific heat capacity of copper = 390 J kg-1 K-1 (a) (i) The temperature of the water rises from 15C to 35C. Calculate the thermal energy gained by the water.

thermal energy gained = .................................... (ii) Calculate the temperature reached by the copper in the flame. Assume no heat is lost when the copper is transferred.

temperature = .................................... (4 marks) (b) When the lump of copper entered the water, some of the water was turned to steam. (i) The specific latent heat of vaporisation of steam is 2.25 MJ kg-1. What further measurement would need to be made to calculate the energy used to produce this steam? ................................................................................................................................... ................................................................................................................................... (ii) Without further calculation, describe how this further measurement should be used to obtain a more accurate value of the flame temperature. ................................................................................................................................... ................................................................................................................................... ................................................................................................................................... ................................................................................................................................... (3 marks) Total 7 marks

Potassium-42 decays with a half-life of 12 hours. When potassium-42 decays, it emits "- particles and gamma rays. One freshly prepared source has an activity of 3.0 107 Bq. (a) To determine the radiation dose absorbed by the scientist working with the source, the number of gamma rays photons incident on each cm2 of the body has to be known. One in every five of the decaying nuclei produces a gamma ray photon. A scientist is initially working 1.50 m from the fresh source with no shielding. Show that at this time approximately 21 gamma photons per second are incident on each cm2 of the scientists body.

(3 marks) (b) The scientist returns 6 hours later and works at the same distance from the source. (i) Calculate the new number of gamma ray photons incident per second on each cm2 of the scientists body.

number of gamma photons per second per cm2 = ................................. (ii) Explain why it is not necessary to consider the beta particle emissions when determining the radiation dose the scientist receives. ................................................................................................................................... ................................................................................................................................... ................................................................................................................................... (5 marks) Total 8 marks

PHYA5: Nuclear and Thermal Physics


Question 1 (a) graph passes through N = 100 to 130 when Z = 80 ! and N = 20 when Z = 20 ! (b) (i) (ii) (iii) (c) W X Y at Z > 60 just below line ! just above line ! just below line ! 3 2

fission nuclei (or fragments) are neutron-rich and therefore unstable (or radioactive) ! neutron-proton ratio is much higher than for a stable nucleus (of the same charge (or mass)) ! !- particle emitted when a neutron changes to a proton (in a neutron-rich nucleus) ! 3

(d)

The marking scheme for this part of the question includes an overall assessment for the Quality of Written Communication (QWC). There are no discrete marks for the assessment of written communication but the quality of written communication will be one of the criteria used to assign the answer to one of three levels. Level Good 3 Descriptor an answer will be expected to meet most of the criteria in the level descriptor - answer supported by an appropriate range of relevant points good use of information or ideas about physics going beyond those given in the question - argument well structured with minimal repetition or irrelevant points accurate and clear expression of ideas with only minor errors of spelling, punctuation and grammar - answer partially supported by relevant points good use of information or ideas about physics given in the question but limited beyond this - the argument shows some attempt at structure the ideas are expressed with reasonable clarity but with a few errors of spelling, punctuation and grammar valid points but clearly linked to an argument structure limited use of information or ideas about physics unstructured errors in spelling, punctuation and grammar or lack of fluency Mark range

5-6

Modest 2

3-4

Limited 1

1-2

- incorrect, inappropriate or no response examples of the sort of information or ideas that might be used to support an argument: reduction of greenhouse gas emissions is (thought to be) necessary to stop global warming ! long term storage of radioactive waste is essential because the radiation from it damages (or kills) living cells ! radioactive isotopes with very long half lives are in the used fuel rods ! nuclear power is reliable because it does not use oil or gas from other countries ! radioactive waste needs to be stored in secure and safe conditions for many years ! conclusion either nuclear power is needed; reduction of greenhouse gases is a greater problem than the storage of radioactive waste because 1 global warming would cause the ice caps to melt/sea levels to rise ! 2 safe storage of radioactive waste can be done ! or nuclear power is not needed; storage of radioactive waste is a greater problem than reduction of greenhouse gases because 1 2 radioactive waste has to be stored for thousands of years ! greenhouse gases can be reduced using renewable energy sources ! Total

14

Question 2 (a) use of pV = constant or p1V1 = p2V2 ! p = 99 3.50/4.15 ! = 83.5 kPa ! (b) no. of moles = 99 [ 10] 3.5 10-4/8.31 291 ! = 1.4(3) 10-2 moles ! no. of molecules (= 1.4(3) 10-2 6.02 10-23) = 8.6 (1) 10-21 ! (c) molecules/particles have momentum ! momentum change at wall ! momentum change at wall/collision at wall leads to force ! [allow impulse arguments] less air so fewer molecules ! so change in momentum per second/rate of change is less [or per unit per time] ! pressure is proportional to number of molecules (per unit volume) ! Total 11 3 3

max 5

Question 3

(a)

(i)

thermal energy gained by water = 0.45 4200 (35 15) = 3.78 104 J ! (thermal energy loss by copper = thermal energy gained by water gives) 0.12 390 "T = 3.78 104 ! "T = 3

(ii)

3.78 10 4 = 808 K 0.12 390

flame temperature (= 808 + 35C) = 843C or 1116 K ! (b) (i) (ii) measure the total mass of the water, beaker and iron lump (to find the mass of water lost) ! mass of water lost due to conversion to steam, m = mass measured in (b) (i) - initial mass of water, beaker and iron ! add the thermal energy due to steam produced, mL, to the thermal energy gained by the water ! calculated flame temperature would be greater !
Total Question 4 7 4

(a)

number of gamma photons per second =

3.0 10 7 (= 6.0 106) ! 5


3

area of sphere of radius 1.50 m (= 4 #r2 = 4# 1.52) = 28.3 m2 ! number of gamma photons per sec per cm2 =
(b) (i)

6.0 10 6 ! (= 21(.2)) 28.3 10 4

decay constant = (

0.693 ln 2 ) = 1.60 10-5 s-1 ! = 12 3600 t1/ 2


5

new no. of gamma photons per sec per cm2 = 21(.2) e-(1.6 10 = 15(.0) !

6.0 3600)

(or 6 hours is 0.5 half-lives ! source activity decreases to 2-0.5 of initial activity in this time ! new no. of gamma photons per sec per cm2 = 21(.2) 2-6/12 ! = 15(.0) !) (ii) any two of the following points !! beta particle range in air is less than 1.5 m beta particle absorbed by air beta particles lose energy in air more rapidly than gamma photons beta particles ionise air much more than gamma photons
Total

Surname Centre Number Candidate Signature

Other Names Candidate Number

Leave blank

General Certificate of Education 2010 Advanced Examination


version 1.1

!"#
PHA5A
For Examiners Use Number Mark Number 5 Mark 1 2 3 4 Total (Column 1) Total (Column 2) TOTAL Examiners Initials

PHYSICS A Unit 5A Astrophysics Section B


SPECIMEN PAPER

Time allowed: 50 minutes


Instructions Use blue or black ink or ball-point pen. Fill in the boxes at the top of this page. Answer all questions. A Data and Formula Booklet is provided as a loose insert. Information The maximum mark for this paper is 35. The marks for the questions are shown in brackets. You are reminded of the need for good English and clear presentation in your answers. You will be assessed on your quality of written communication where indicated in the question.

Section B The maximum mark for this section is 35 marks. You are advised to spend approximately 50 minutes on this section. 1 A converging lens can be used to produce both a magnified real image and a magnified virtual image of an object. (a) Draw ray diagrams to show how each image are formed. Label the principal foci of the lens in each case.

(4 marks) (b) Calculate the object distance required to produce a magnified image 0.25 m from a lens of focal length 0.10 m where the image is virtual.

Object distance = .................................... (2 marks) Total 6 marks

Figure 1 shows the black body radiation curves for three stars, labelled P, Q and R. Figure 1

(a)

(i) State and explain, without calculation, which one of these three stars is the hottest. ................................................................................................................................... ................................................................................................................................... ................................................................................................................................... ................................................................................................................................... ................................................................................................................................... (ii) Calculate the black body temperature of the hottest star.

Temperature =..................................... (3 marks)

(b)

More detailed analysis of the hottest stars spectrum revealed the presence of Hydrogen Balmer absorption lines. (i) For which two spectral classes are these lines the prominent feature? ................................................................................................................................... (ii) Describe how these absorption lines are produced in the spectrum of a star. ................................................................................................................................... ................................................................................................................................... ................................................................................................................................... ................................................................................................................................... ................................................................................................................................... ................................................................................................................................... ................................................................................................................................... ................................................................................................................................... (4 marks) Total 7 marks

Modern astronomy relies on the analysis of radiation from many different parts of the electromagnetic spectrum. Compare the main features of telescopes used to detect radio waves with those of optical reflecting telescopes. Explain the differences in their resolving and collecting powers. ...................................................................................................................................................... ...................................................................................................................................................... ...................................................................................................................................................... ...................................................................................................................................................... ...................................................................................................................................................... ...................................................................................................................................................... ...................................................................................................................................................... ...................................................................................................................................................... ...................................................................................................................................................... ...................................................................................................................................................... ...................................................................................................................................................... ...................................................................................................................................................... ...................................................................................................................................................... ...................................................................................................................................................... ...................................................................................................................................................... ...................................................................................................................................................... (6 marks) Total 6 marks

(a) Define the absolute magnitude of a star. ............................................................................................................................................. ............................................................................................................................................. (1 mark) (b) Figure 2 shows the axes of a Hertzsprung-Russell (H-R) diagram. Figure 2

(i) (ii)

On each axis indicate a suitable range of values. Label with an S the current position of the Sun on the H-R diagram.

(iii) Label the positions of the following stars on the H-R diagram: (1) (2) (3) (4) star W, which is significantly hotter and brighter than the Sun. star X, which is significantly cooler and larger than the Sun. star Y, which is the same size as the Sun, but significantly cooler. star Z, which is much smaller than the Sun, and has molecular bands as an . important feature in its spectrum. (7 marks) Total 8 marks

The red shift of a galaxys spectrum can be used to determine its velocity, relative to the Earth. (a) The wavelength of the hydrogen alpha line in the spectrum of the galaxy NGXC 1357 is 660.86 nm. The wavelength of the same line from a laboratory based source is 656.28 nm. Calculate the velocity of galaxy NGC 1357.

Velocity = .................................... (2 marks) (b) Use the value obtained in (a) to complete the table. Plot a graph of the data in the table and use the graph to determine a value for the Hubble constant. galaxy NGC 1357 NGC 1832 NGC 5548 NGC 7469 2000 5270 4470 velocity/km s-1 distance/Mpc 28 31 67 65

Hubble constant..................................... (3 marks)

(c)

Analysis of light from supernovae suggests that the expansion of the Universe is accelerating. (i) Explain how the light from supernovae can be used to determine the distance to galaxies. ................................................................................................................................... ................................................................................................................................... ................................................................................................................................... ................................................................................................................................... ................................................................................................................................... ................................................................................................................................... (ii) What is the name given to the energy believed to be responsible for this accelerating expansion? ................................................................................................................................... (3 marks) Total 8 marks

PHA5A: Astrophysics
Question 1 (a) for both diagrams: rays from top of object passes straight through centre of lens ! principal foci correctly labelled ! ray parallel to principal axis passes through focal point to form virtual image ! ray parallel to principal axis passes through focal point to form real image !

(b)

= 1/f 1/u u

= 1/u + 1/v = 1/0.1 + 1/0.25 ! = 1/14 = 0.17 m = 0.07 m ! Total 6 2

Question 2 (a) (i) P, it has the lowest peak wavelength ! max ! and ! max T = constant, so lowest ! max means highest T ! (ii) (b) (i) (ii) use of ! max T = 0.0029 and ! max = 300 10-9 m ! gives T = 9 700 K ! A and B ! light from the star passes through the atmosphere of the star ! which contains hydrogen with electrons in the n = 2 state ! electrons in the n = 2 state absorb certain energies and therefore frequencies of light ! the light is reemitted in all directions and therefore the intensity of the light of these frequencies in the direction of the observer is reduced, resulting in absorption lines in the spectrum ! Total 7 max 4 max 3

Question 3 3 marks for any of the following 3 features compared with optical reflecting telescopes, radio telescopes: are much longer have a much lower resolving power are not as affected by the atmosphere and so their positioning is less critical have only one reflecting surface rather than two have a similar structure in that a concave reflecting surface reflects the em radiation to a detector at the focal point 3

explanations of resolving power radio telescopes have a lower resolving power: because the ratio of wavelength to telescope diameter is larger ! because radio wavelengths are very much larger than optical wavelengths (even though the diameters of radio telescopes are larger) ! explanations of collecting power: collecting power depends on the area of the objective which is much larger for radio telescopes (depends on the square of the diameter) ! Total Question 4 (a) (b) (i) (ii) (iii) brightness of star from a distance of 10 pc ! temperature from 30000 K to 2500 K ! absolute magnitude from +15 to "10 ! S at 6000, 5 ! 1 6 3

W above and to left of S ! X above and to right of S ! Y below and to right of S ! Z below and to right of S !
30000 2500

Total

Question 5 (a) # !/! = - v/c (660.86 656.28)/656.28 = (-)v/(3 108) ! v = (-) 2094 km s-1 !
6000

5000

4000

velocity / kms-1

2
3000

2000

1000

0 0 10 20 30 40 50 60 70 80

distance / Mpc

(b)

graph points !, line through the origin ! H = v/d = slope = 70 ( 4) km s-1 Mpc-1 !

(c)

(i)

supernovae act as standard candles ! known amount of light emitted (absolute magnitude known), measured amount detected at Earth (apparent magnitude measured) ! inverse square law can be used to determine distance ! max 3

(ii)

dark energy ! Total 8

Centre Number Surname Other Names Candidate Signature

Candidate Number

For Examiners Use

Examiners Initials

Question General Certificate of Education Advanced Level Examination Specimen paper 2 2010

Mark

1 2 3

Physics A
Unit 5 Section A Nuclear and Thermal Physics Specimen Paper 2
For this paper you must have: a calculator a ruler a Data and Formulae Booklet

PHYA5

4 TOTAL

Time allowed The total time for both sections of this paper is 1 hour 45 minutes You are advised to spend approximately 55 minutes on this section. Instructions Use black ink or black ball-point pen. Fill in the boxes at the top of this page. Answer all questions. You must answer the questions in the spaces provided. Answers written in margins or on blank pages will not be marked. Do all rough work in this book. Cross through any work you do not want to be marked. Information The marks for questions are shown in brackets. The maximum mark for this section is 40 You are expected to use a calculator where appropriate. A Data and Formulae Booklet is provided as a loose insert. You will be marked for your ability to: - use good English - organise information clearly - use specialist vocabulary where appropriate.

Areas outside the box will not be scanned for marking

1.

In an experiment to measure the temperature of the flame of a Bunsen burner, a lump of copper of mass 0.12 kg is heated in the flame for several minutes. The copper is then transferred quickly to a beaker, of negligible heat capacity, containing 0.45 kg of water, and the temperature rise of the water measured. Specific heat capacity of water = 4200 J kg1 K1 Specific heat capacity of copper = 390 J kg1 K1

(a)

(i)

The temperature of the water rises from 15C to 35C. Calculate the thermal energy gained by the water.

thermal energy gained = ....................................

(ii)

Calculate the temperature reached by the copper in the flame. Assume no heat is lost when the copper is transferred.

temperature = ....................................
(4)

(b)

When the lump of copper entered the water, some of the water was turned to steam. (i) The specific latent heat of vaporisation of steam is 2.25 MJ kg1. What further measurement would need to be made to calculate the energy used to produce this steam? ........................................................................................................................... ........................................................................................................................... (ii) Without further calculation, describe how this further measurement should be used to obtain a more accurate value of the flame temperature. ........................................................................................................................... ........................................................................................................................... ........................................................................................................................... ...........................................................................................................................
(3) (Total 7 marks)

Turn over
0 1

Areas outside the box will not be scanned for marking

2.

Nuclear radii can be determined by observing the diffraction of high energy electrons, as shown in the diagram.

electron beam thin sample detector

(a)

On the axes below, sketch a graph of the results expected from such an electron diffraction experiment.

intensity of diffracted electrons

diffraction angle()
(2)

(b)

State why high energy electrons are used in determining nuclear size. ..................................................................................................................................... ..................................................................................................................................... .....................................................................................................................................
(1)

Turn over
0 2

Areas outside the box will not be scanned for marking

(c)

Electron diffraction experiments have been performed on a range of different nuclei to give information about nuclear density and average separation of particles in the nucleus. Give the main conclusion in each case. nuclear density ............................................................................................................ ..................................................................................................................................... average separation of particles ................................................................................... .....................................................................................................................................
(2)

(d)

On the axes below, sketch the relationship between the radius of a nucleus and its nucleon number.

radius of nucleus

nucleon number
(1)

(e)

Given that the radius of the


16 8O

12 6C

nucleus is 3.04 1015m, calculate the radius of the

nucleus.

..................................................................................................................................... ..................................................................................................................................... ..................................................................................................................................... ..................................................................................................................................... .....................................................................................................................................


(3) (Total 9 marks)

Turn over
0 3

Areas outside the box will not be scanned for marking

3.

(a)

The equation of state of an ideal gas is pV = nRT For each of these symbols, state the physical quantity and the SI unit. symbol p V n R T
(4)

physical quantity

unit

(b)

An ideal gas of volume 1.0 104 m3 is trapped by a piston in a cylinder as shown in the diagram. There is negligible friction between the piston and the cylinder. Initially, the temperature of the gas is 20C and the external atmospheric pressure acting on the piston is 100 kPa.

cylinder

piston

heater

trapped gas
The gas expands slowly when heat is supplied by an electric heater inside the cylinder.

(i)

Calculate the work done by the gas when its volume slowly increases by 5.0 105 m3, at a constant pressure, while being heated. ........................................................................................................................... ...........................................................................................................................

Turn over
0 4

Areas outside the box will not be scanned for marking

(ii)

What is the temperature of the gas, in C, following its expansion? ........................................................................................................................... ........................................................................................................................... ........................................................................................................................... ........................................................................................................................... ...........................................................................................................................

(iii)

Describe two changes that occur in the motion of a typical molecule of the gas during the expansion. ........................................................................................................................... ...........................................................................................................................
(8) (Total 12 marks)

Turn over
0 5

Areas outside the box will not be scanned for marking

4.

(a)

(i)

Complete the equation below which represents the induced fission of a nucleus of uranium 235 92 U.
235 92 U

1 0

98 38

Sr +

54Xe

1 0

(ii)

The graph shows the binding energy per nucleon plotted against nucleon number A. Mark on the graph the position of each of the three nuclei in the equation.

binding eneregy per nucleon/Me V 8.0 7.0 6.0 5.0 4.0 3.0 2.0 1.0 0.0

50

100

150

200 250 nucleon number A

(iii)

Hence determine the energy released in the fission process represented by the equation. ........................................................................................................................... ........................................................................................................................... ........................................................................................................................... ........................................................................................................................... ........................................................................................................................... ...........................................................................................................................
(6)

Turn over
0 6

Areas outside the box will not be scanned for marking

(b)

(i)

Use your answer to part (a)(iii) to estimate the energy released when 1.0 kg of uranium, containing 3% by mass of 235 92 U, undergoes fission. ........................................................................................................................... ........................................................................................................................... ........................................................................................................................... ........................................................................................................................... ........................................................................................................................... ...........................................................................................................................

(ii)

Oil releases approximately 50 MJ of heat per kg when it is burned in air. State and explain one advantage and one disadvantage of using nuclear fuel to produce electricity. advantage .......................................................................................................... ........................................................................................................................... ........................................................................................................................... ........................................................................................................................... disadvantage ..................................................................................................... ........................................................................................................................... ........................................................................................................................... ...........................................................................................................................
(6) (Total 12 marks)

Turn over
0 7

Physics A PHYA5 - AQA GCE Mark Scheme 2010

Instructions to Examiners
1. Give due credit for alternative treatments which are correct. Give marks for what is correct in accordance with the mark scheme; do not deduct marks because the attempt falls short of some ideal answer. Where marks are to be deducted for particular errors, specific instructions are given in the marking scheme. Do not deduct marks for poor written communication. Refer the scripts to the Awards meeting if poor presentation forbids a proper assessment. In each paper, candidates are assessed on their quality of written communication (QWC) in designated questions (or part-questions) that require explanations or descriptions. The criteria for the award of marks on each such question are set out in the mark scheme in three bands in the following format. The descriptor for each band sets out the expected level of the quality of written communication of physics for each band. Such quality covers the scope (eg relevance, correctness), sequence and presentation of the answer. Amplification of the level of physics expected in a good answer is set out in the last row of the table. To arrive at the mark for a candidate, their work should first be assessed holistically (ie in terms of scope, sequence and presentation) to determine which band is appropriate then in terms of the degree to which the candidates work meets the expected level for the band. QWC descriptor mark range see specific mark scheme Good Excellent see specific mark scheme Modest Adequate see specific mark scheme Poor Limited The description and/or explanation expected in a good answer should include a coherent account of the following points: see specific mark scheme Answers given as bullet points should be considered in the above terms. Such answers without an overview paragraph in the answer would be unlikely to score in the top band. 3. An arithmetical error in an answer will cause the candidate to lose one mark and should be annotated AE if possible. The candidates incorrect value should be carried through all subsequent calculations for the question and, if there are no subsequent errors, the candidate can score all remaining marks. The use of significant figures is tested once on each paper in a designated question or partquestion. The numerical answer on the designated question should be given to the same number of significant figures as there are in the data given in the question or to one more than this number. All other numerical answers should not be considered in terms of significant figures. Numerical answers presented in non-standard form are undesirable but should not be penalised. Arithmetical errors by candidates resulting from use of non-standard form in a candidates working should be penalised as in point 3 above. Incorrect numerical prefixes and the use of a given diameter in a geometrical formula as the radius should be treated as arithmetical errors. Knowledge of units is tested on designated questions or parts of questions in each a paper. On each such question or part-question, unless otherwise stated in the mark scheme, the mark scheme will show a mark to be awarded for the numerical value of the answer and a further mark for the correct unit. No penalties are imposed for incorrect or omitted units at intermediate stages in a calculation or at the final stage of a non-designated unit question. All other procedures including recording of marks and dealing with missing parts of answers will be clarified in the standardising procedures.

2.

4.

5.

6.

7.

Physics A PHYA5 - AQA GCE Mark Scheme 2010

PHYA5: uclear and Thermal Physics

Question 1 (a) (i) (ii) thermal energy gained by water = 0.45 4200 (35 15) = 3.78 104 J (1) (thermal energy loss by copper = thermal energy gained by water gives) 0.12 390 T = 3.78 104 (1) T =
3.78 10 4 = 808 K 0.12 390

flame temperature (= 808 + 35C) = 843C or 1116 K (1)

(b)

(i)

measure the total mass of the water, beaker and iron lump (to find the mass of water lost) (1) mass of water lost due to conversion to steam, m = mass measured in (b) (i) - initial mass of water, beaker and iron (1) add the thermal energy due to steam produced, mL, to the thermal energy gained by the water (1) calculated flame temperature would be greater (1) Total 7

Question 2 (a)
intensity of scattered electrons

2
diffraction angle

graph shows a minimum (1) which does not touch the axis (1) (b) the (de Broglie) wavelength of high energy electrons is comparable to nuclear radii [or not subject to the strong nuclear force] (1) nuclear density is constant (1) separation of neighbouring nucleons is constant [or nucleons are close-packed] (1) 1

(c)

Physics A PHYA5 - AQA GCE Mark Scheme 2010

(d)

A
correct curve (1) (e)
1

R = r0A 3 (1)
A0 R0 = Rc A c 3 16 3 (1) 15 = 3.04 10 12
1 1

R0 = 3.35 1015 m (1) Total 9

Question 3 (a) pressure volume (*) (1)(not allow kPa) number of moles molar gas constant temperature Pa or N m2 (*) m3 (*) mol (or none) (1) J K1 mol1 (1) (mol1 implies molar) K (1) 4

Physics A PHYA5 - AQA GCE Mark Scheme 2010

(b)

(i)

W(= pV) = 1.0 105 5 105 (1) = 5.0 J (1)

(ii)

V = constant (or use of pV = nRT) (1) T 1.0 10 4 1.5 10 4 (1) = 293 T2 gives T2 = 440K (1) = 440 273 = 167 C (1)
any two from the following:
speed (or kinetic energy) increases (1) collision rate (with walls) decreases (or time between collisions increases) (1) distance between collisions increases (mean free path inverse prop. to molecular density) (1)

(iii)

Total

12

Physics A PHYA5 - AQA GCE Mark Scheme 2010

Question 4
(a) (i)
235 92

U+

1 0

98 38

Sr +

135 54

1 Xe + 3 0 n(+Q) (1)

(ii)

three correct positions to within 2 on x-axis (1) (1) (one mark if two correct) estimate of energy released: binding energy of U-235 nucleus = (235 7.5) = 1763(15)(MeV) (1) binding energy of Sr-98 = (98 8.6) = 843( 15)(MeV) (1) binding energy of Xe-135 = (135 8.4) = 1134( 15)(MeV) (1) binding energy released = 1134 + 843 1763 = 214MeV (1) (40MeV) 235g of U-235 releases 6 1023 214 1.6 1013J = 2.1 1013(J) (1) 1.0 kg of uranium containing 3% U-235 contains 30g of U-235 (1) 13 energy from 1.0kg of uranium = 2.1 10 30 235 12 25 = 2.6 10 J [[1.6 10 MeV]] (1) advantage: less mass of fuel used (1) because more energy per kilogram (1) [alternative: less harm to environment (1) because does not generate greenhouse gases (1) or any statement (1) argued (1)] disadvantage: hazardous waste (1) because fission products are radioactive (1) [alternative: long term responsibility (1) because waste needs to be stored for many years (1) or any statement (1) argued (1)]

(iii)

(b)

(i)

(ii)

Total

12

Centre Number Surname Other Names Candidate Signature

Candidate Number

For Examiners Use

Examiners Initials

Question General Certificate of Education Advanced Level Examination Specimen paper 2 2010

Mark

1 2 3

Physics A
Unit 5A Section B Specimen Paper 2
For this paper you must have: a calculator a ruler a Data and Formulae Booklet

PHYA5A
Astrophysics

4 TOTAL

Time allowed The total time for both sections of this paper is 1 hour 45 minutes You are advised to spend approximately 50 minutes on this section. Instructions Use black ink or black ball-point pen. Fill in the boxes at the top of this page. Answer all questions. You must answer the questions in the spaces provided. Answers written in margins or on blank pages will not be marked. Do all rough work in this book. Cross through any work you do not want to be marked. Information The marks for questions are shown in brackets. The maximum mark for this section is 35 You are expected to use a calculator where appropriate. A Data and Formulae Booklet is provided as a loose insert. You will be marked for your ability to: - use good English - organise information clearly - use specialist vocabulary where appropriate.

Areas outside the box will not be scanned for marking

1.

(a)

Describe the main features of black holes and quasars. You may be awarded marks for the quality of written communication in your answer. ................................................................................................................................ ................................................................................................................................ ................................................................................................................................ ................................................................................................................................ ................................................................................................................................ ................................................................................................................................ ................................................................................................................................ ................................................................................................................................ ................................................................................................................................ ................................................................................................................................ ................................................................................................................................ ................................................................................................................................
(3)

(b)

There is some evidence to suggest that there is a black hole of 3 109 solar masses at the centre of the galaxy M87. Calculate the radius of the event horizon for this black hole. ................................................................................................................................ ................................................................................................................................ ................................................................................................................................ ................................................................................................................................ ................................................................................................................................ ................................................................................................................................
(2) (Total 5 marks)

Turn over
0 1

Areas outside the box will not be scanned for marking

2.

(a)

Using the axes below, sketch the emissive power of a black body, E, as a function of wavelength for each of three temperatures, 1000 K, 1200 K and 1600 K. Indicate the temperature on each curve.
E /Wm 2 nm1

/nm
(3)

(b)

(i)

Wiens displacement law may be written as max T = constant. State what max represents. ........................................................................................................................... ...........................................................................................................................

(ii)

Obtain the value of the constant from the Data Sheet and calculate max for a temperature of 1600 K. ........................................................................................................................... ...........................................................................................................................

(iii) Assuming that max for the Sun lies within the range of the visible spectrum, use Wiens law to estimate the temperature of the Sun. ........................................................................................................................... ........................................................................................................................... ...........................................................................................................................
(5)

Turn over
0 2

Areas outside the box will not be scanned for marking

(c)

Stefans law for a black body may be written as E = T4 where E, the energy radiated per second per square metre of the surface area, has units W m2. (i) Hence state how a value of E may be obtained from one of the curves drawn in part (a) for a given temperature. ........................................................................................................................... ...........................................................................................................................

(ii)

For a black body of surface area A, give Stefans law in the form which gives total radiative power of the body. ...........................................................................................................................
(2)

(d)

(i)

The diagram represents the Earth in orbit around the Sun. Given that the Earth receives 1400 W m2 of energy from the Sun and that the Sun emits energy equally in all directions, estimate the total output power, in W, of the Sun. mean radius, R, of the Earths orbit around the Sun = 1.5 1011 m

R Sun

Earth

........................................................................................................................... ........................................................................................................................... ........................................................................................................................... ........................................................................................................................... ...........................................................................................................................

(ii)

Hence, use Stefans law to deduce the temperature of the Sun. radius of the Sun = 7.0 108 m ........................................................................................................................... ........................................................................................................................... ...........................................................................................................................
(4) (Total 14 marks)

Turn over
0 3

Areas outside the box will not be scanned for marking

3.

(a)

An astronomical telescope consists of two thin converging lenses, as shown. The focal lengths of the objective lens and the eyepiece lens are f and f, respectively. The telescope is used to view the Moon and the separation of the lenses is such that the telescope is in normal adjustment. objective eyepiece

(i)

Draw rays to show how a magnified image is formed by the telescope. Your ray diagram should show the paths through the telescope of two parallel non-axial rays from a point on the Moon to the observers eye. Indicate the position of the principal focus for each lens.

(ii)

An observers unaided eye has a resolving power of 120 seconds of arc. If the angular magnification of a telescope is 24, determine the angular separation of two points on the Moon, which the same observer can just resolve with the aid of the telescope. The angular magnification of the instrument is given by M=
angle subtended at the eye by the image angle subtended at the eye by the object

............................................................................................................................ ............................................................................................................................ ............................................................................................................................


(7)

Turn over
0 4

Areas outside the box will not be scanned for marking

(b)

A parallel beam of monochromatic light, incident on a converging lens and parallel to the axis, fills the lens aperture completely. After transmission through the lens the beam falls on a plane surface placed parallel to the plane of the aperture and approximately 1 m away from it. The plane surface is in the focal plane of the lens. (i) Sketch a graph, using the axes, to show how the intensity of the light varies with the radial distance from the central axis of the lens, for small radial distances.

intensity

central axis

distance

(ii)

With the aid of another diagram, describe Rayleighs criterion for the resolution of two point sources viewed through the lens.

............................................................................................................................ ............................................................................................................................ ............................................................................................................................


(5)

Turn over
0 5

Areas outside the box will not be scanned for marking

(c)

(i)

The objective lens of the telescope described in part (a) has a diameter of 15 cm. The telescope is used to view the star Mizar, which is a double star with an angular separation of 7.0 l05 rad. Calculate the approximate value of the resolving power of the telescope for light of wavelength 6.0 107 m. Hence determine whether the two stars could be resolved by the telescope. ............................................................................................................................ ............................................................................................................................ ............................................................................................................................ ............................................................................................................................

(ii)

If a double star, similar to that described in part (c)(i), cannot be resolved by the telescope, discuss whether or not increasing the angular magnification of this telescope would allow resolution. ............................................................................................................................ ............................................................................................................................ ............................................................................................................................ ............................................................................................................................
(4) (Total 16 marks)

Turn over
0 6

Physics A PHYA5A - AQA GCE Mark Scheme 2010

Instructions to Examiners
1. Give due credit for alternative treatments which are correct. Give marks for what is correct in accordance with the mark scheme; do not deduct marks because the attempt falls short of some ideal answer. Where marks are to be deducted for particular errors, specific instructions are given in the marking scheme. Do not deduct marks for poor written communication. Refer the scripts to the Awards meeting if poor presentation forbids a proper assessment. In each paper, candidates are assessed on their quality of written communication (QWC) in designated questions (or part-questions) that require explanations or descriptions. The criteria for the award of marks on each such question are set out in the mark scheme in three bands in the following format. The descriptor for each band sets out the expected level of the quality of written communication of physics for each band. Such quality covers the scope (eg relevance, correctness), sequence and presentation of the answer. Amplification of the level of physics expected in a good answer is set out in the last row of the table. To arrive at the mark for a candidate, their work should first be assessed holistically (ie in terms of scope, sequence and presentation) to determine which band is appropriate then in terms of the degree to which the candidates work meets the expected level for the band. QWC descriptor mark range see specific mark scheme Good Excellent see specific mark scheme Modest Adequate see specific mark scheme Poor Limited The description and/or explanation expected in a good answer should include a coherent account of the following points: see specific mark scheme Answers given as bullet points should be considered in the above terms. Such answers without an overview paragraph in the answer would be unlikely to score in the top band. 3. An arithmetical error in an answer will cause the candidate to lose one mark and should be annotated AE if possible. The candidates incorrect value should be carried through all subsequent calculations for the question and, if there are no subsequent errors, the candidate can score all remaining marks. The use of significant figures is tested once on each paper in a designated question or partquestion. The numerical answer on the designated question should be given to the same number of significant figures as there are in the data given in the question or to one more than this number. All other numerical answers should not be considered in terms of significant figures. Numerical answers presented in non-standard form are undesirable but should not be penalised. Arithmetical errors by candidates resulting from use of non-standard form in a candidates working should be penalised as in point 3 above. Incorrect numerical prefixes and the use of a given diameter in a geometrical formula as the radius should be treated as arithmetical errors. Knowledge of units is tested on designated questions or parts of questions in each a paper. On each such question or part-question, unless otherwise stated in the mark scheme, the mark scheme will show a mark to be awarded for the numerical value of the answer and a further mark for the correct unit. No penalties are imposed for incorrect or omitted units at intermediate stages in a calculation or at the final stage of a non-designated unit question. All other procedures including recording of marks and dealing with missing parts of answers will be clarified in the standardising procedures.

2.

4.

5.

6.

7.

Physics A PHYA5A - AQA GCE Mark Scheme 2010

PHYA5A: Astrophysics

Question 1 (a) black hole: large gravitational field or very dense (1) escape velocity greater than c (1) quasar: large red shift or very far away (1) very powerful sources (1)
11 10 9 2 10 30 (1) R = 2GM = 2 6.67 10 3 8 2 c (3 10 ) 2

Max 3

(b)

= 8.9 10 m (1) Total 5

12

Physics A PHYA5A - AQA GCE Mark Scheme 2010

Question 2 (a)
E 1600K

1200K

1000K

wavelength

peaks displaced towards increasing wavelength (1) steeper slope on left hand side (1) correct temperatures (1) (b) (i) (ii)

max is wavelength at which maximum intensity occurs (1)


constant = 2.9 103mK (1) for T = 1600K, gives max = 1800 nm (1) 5

(iii)

max 550 nm (1)


T=

2.9 10 3 = 5272K (1) (accept 5300K) 550 10 9


2

(c)

(i) (ii)

E is the area under one of the curves in the graph (1) P = AT4 (1) (total power output from Sun = E) power arriving at Earth = E (1) 4R 2 2 = 1400 (W m ) (1) E = 1400 4 (1.5 1011)2 = 4.0 1026 (W) (1) Max 4

(d)

(i)

(ii)

(P = AT4) gives T4 = (= 1.12 1015) T = 5800 K (1)

4.0 10 26 (1) 4 (7 10 8 ) 2 5.7 10 8

Total

14

Physics A PHYA5A - AQA GCE Mark Scheme 2010

Question 3
(a) (i)

objective

eyepiece

correct rays through objective (1) first image at F0 (1) F0 and Fe coincide (1) correct construction line(s) (1) rays emerge parallel (1)

(ii)

' 120 M = gives 24 = (1) a


= 5 sec (of arc) (1)

(b)

(i)

intensity

5
central maximum (1) decreasing maxima (1) approximate equal spacing between minima either side of central axis (1) (ii) two intensity curves [or two Airy discs] (1) central maximum of first coincides with first minimum of second curve (1)

Physics A PHYA5A - AQA GCE Mark Scheme 2010

(c) (i)

6.0 10 7 = gives = = 4.0 106 rad (1) d 15 10 2


angular separation of Mizar 7.0 105 rad, can be resolved (1)

(ii)

resolving power controlled by diffraction effects [ or by

4
]

(1)
hence increasing angular magnification does not increase resolution (1)

Total

16

Centre Number Surname Other Names Candidate Signature

Candidate Number

For Examiners Use

Examiners Initials

Question General Certificate of Education Advanced Level Examination Specimen paper 3 2010

Mark

1 2 3

Physics A
Unit 5 Section A Nuclear and Thermal Physics Specimen Paper 3
For this paper you must have: a calculator a ruler a Data and Formulae Booklet

PHYA5

4 TOTAL

Time allowed The total time for both sections of this paper is 1 hour 45 minutes You are advised to spend approximately 55 minutes on this section. Instructions Use black ink or black ball-point pen. Fill in the boxes at the top of this page. Answer all questions. You must answer the questions in the spaces provided. Answers written in margins or on blank pages will not be marked. Do all rough work in this book. Cross through any work you do not want to be marked. Information The marks for questions are shown in brackets. The maximum mark for this section is 40 You are expected to use a calculator where appropriate. A Data and Formulae Booklet is provided as a loose insert. You will be marked for your ability to: - use good English - organise information clearly - use specialist vocabulary where appropriate.

Areas outside the box will not be scanned for marking

1.

(a)

(i)

State what is meant by thermal equilibrium. ........................................................................................................................... ........................................................................................................................... ...........................................................................................................................

(ii)

Explain thermal equilibrium by reference to the behaviour of the molecules when a sample of hot gas is mixed with a sample of cooler gas and thermal equilibrium is reached. ........................................................................................................................... ........................................................................................................................... ...........................................................................................................................
(3)

(b)

A sealed container holds a mixture of nitrogen molecules and helium molecules at a temperature of 290 K. The total pressure exerted by the gas on the container is 120 kPa. molar mass of helium molar gas constant R the Avogadro constant = = = 4.00 103 kg mol1 8.31 J K1 mol1 6.02 1023 mol1

(i)

Calculate the root mean square speed of the helium molecules. ........................................................................................................................... ........................................................................................................................... ........................................................................................................................... ...........................................................................................................................

(ii)

Calculate the average kinetic energy of a nitrogen molecule. ........................................................................................................................... ........................................................................................................................... ...........................................................................................................................

Turn over
0 1

Areas outside the box will not be scanned for marking

(iii)

If there are twice as many helium molecules as nitrogen molecules in the container, calculate the pressure exerted on the container by the helium molecules. ........................................................................................................................... ........................................................................................................................... ...........................................................................................................................
(6) (Total 9 marks)

Turn over
0 2

Areas outside the box will not be scanned for marking

2.

An electrical heater is used to heat a 1.0 kg block of metal, which is well lagged. The table shows how the temperature of the block increased with time. temp/C time/s (a) 20.1 0 23.0 60 26.9 120 30.0 180 33.1 240 36.9 300

Plot a graph of temperature against time on the grid provided.

temp /C

time/s
(3)

Turn over
0 3

Areas outside the box will not be scanned for marking

(b)

Determine the gradient of the graph. ..................................................................................................................................... .....................................................................................................................................


(2)

(c)

The heater provides thermal energy at the rate of 48 W. Use your value for the gradient of the graph to determine a value for the specific heat capacity of the metal in the block. ..................................................................................................................................... ..................................................................................................................................... .....................................................................................................................................
(2)

(d)

The heater in part (c) is placed in some crushed ice that has been placed in a funnel as shown.

heater

crushed ice

The heater is switched on for 200 s and 32 g of ice are found to have melted during this time. Use this information to calculate a value for the specific latent heat of fusion for water, stating one assumption made. ..................................................................................................................................... ..................................................................................................................................... ..................................................................................................................................... ..................................................................................................................................... .....................................................................................................................................
(3) (Total 10 marks)

Turn over
0 4

Areas outside the box will not be scanned for marking

3.

The radioactive isotope of sodium has a half life of 2.6 years. A particular sample of this isotope has an initial activity of 5.5 105 Bq (disintegrations per second). (a) Explain what is meant by the random nature of radioactive decay. You may be awarded marks for the quality of written communication provided in your answer. .................................................................................................................................... .................................................................................................................................... .................................................................................................................................... .................................................................................................................................... ....................................................................................................................................
(2)

(b)

Use the axes to sketch a graph of the activity of the sample of sodium over a period of 6 years.

activity/10 5 Bq 7 6 5 4 3 2 1 0 0 1 2 3 4 5 6 time/year
(2)

Turn over
0 5

Areas outside the box will not be scanned for marking

(c)

Calculate (i) the decay constant, in s1, of 1 year = 3.15 107 s ........................................................................................................................... ........................................................................................................................... ........................................................................................................................... ...........................................................................................................................
22 11 Na ,

(ii)

the number of atoms of

22 11 Na

in the sample initially,

........................................................................................................................... ........................................................................................................................... ........................................................................................................................... ...........................................................................................................................

(iii)

the time taken, in s, for the activity of the sample to fall from 1.0 105 Bq to 0.75 105 Bq. ........................................................................................................................... ........................................................................................................................... ........................................................................................................................... ...........................................................................................................................
(6) (Total 10 marks)

Turn over
0 6

Areas outside the box will not be scanned for marking

4.

(a)

(i)

Complete the equation below to represent the emission of an particle by a 238 92 U isotope.
238 92 U

238 92 U

(ii)

Calculate the energy released when this

isotope nucleus emits an particle

........................................................................................................................... ........................................................................................................................... ........................................................................................................................... ........................................................................................................................... ...........................................................................................................................


(5)

(b)

238 92 206 82

U decays sequentially by emitting particles and particles, eventually forming Pb, a stable isotope of lead. There are eight particles in the sequence. Calculate the number of particles in the sequence. ........................................................................................................................... ........................................................................................................................... ...........................................................................................................................

(i)

(ii)

State the nuclear change that occurs during positron emission. Hence, explain why no positrons are emitted in this sequence. ........................................................................................................................... ........................................................................................................................... ........................................................................................................................... ........................................................................................................................... ........................................................................................................................... ...........................................................................................................................
(6) (Total 11 marks)

Turn over
0 7

Physics A PHYA5 - AQA GCE Mark Scheme 2010

Instructions to Examiners
1. Give due credit for alternative treatments which are correct. Give marks for what is correct in accordance with the mark scheme; do not deduct marks because the attempt falls short of some ideal answer. Where marks are to be deducted for particular errors, specific instructions are given in the marking scheme. Do not deduct marks for poor written communication. Refer the scripts to the Awards meeting if poor presentation forbids a proper assessment. In each paper, candidates are assessed on their quality of written communication (QWC) in designated questions (or part-questions) that require explanations or descriptions. The criteria for the award of marks on each such question are set out in the mark scheme in three bands in the following format. The descriptor for each band sets out the expected level of the quality of written communication of physics for each band. Such quality covers the scope (eg relevance, correctness), sequence and presentation of the answer. Amplification of the level of physics expected in a good answer is set out in the last row of the table. To arrive at the mark for a candidate, their work should first be assessed holistically (ie in terms of scope, sequence and presentation) to determine which band is appropriate then in terms of the degree to which the candidates work meets the expected level for the band. QWC descriptor mark range see specific mark scheme Good Excellent see specific mark scheme Modest Adequate see specific mark scheme Poor Limited The description and/or explanation expected in a good answer should include a coherent account of the following points: see specific mark scheme Answers given as bullet points should be considered in the above terms. Such answers without an overview paragraph in the answer would be unlikely to score in the top band. 3. An arithmetical error in an answer will cause the candidate to lose one mark and should be annotated AE if possible. The candidates incorrect value should be carried through all subsequent calculations for the question and, if there are no subsequent errors, the candidate can score all remaining marks. The use of significant figures is tested once on each paper in a designated question or partquestion. The numerical answer on the designated question should be given to the same number of significant figures as there are in the data given in the question or to one more than this number. All other numerical answers should not be considered in terms of significant figures. Numerical answers presented in non-standard form are undesirable but should not be penalised. Arithmetical errors by candidates resulting from use of non-standard form in a candidates working should be penalised as in point 3 above. Incorrect numerical prefixes and the use of a given diameter in a geometrical formula as the radius should be treated as arithmetical errors. Knowledge of units is tested on designated questions or parts of questions in each a paper. On each such question or part-question, unless otherwise stated in the mark scheme, the mark scheme will show a mark to be awarded for the numerical value of the answer and a further mark for the correct unit. No penalties are imposed for incorrect or omitted units at intermediate stages in a calculation or at the final stage of a non-designated unit question. All other procedures including recording of marks and dealing with missing parts of answers will be clarified in the standardising procedures.

2.

4.

5.

6.

7.

Physics A PHYA5 - AQA GCE Mark Scheme 2010

PHYA5: uclear and Thermal Physics

Question 1 (a) (i) no net flow of (thermal) energy (between two or more bodies) (1) bodies at same temperature (1) 3 (ii) (kinetic) energy is exchanged in molecular collisions (1) until average kinetic energy of all molecules is the same (1) max 3 (b) (i) cr.m.s.
= 3RT M =

3 8.31 290 (1) 4.00 10 3

= 1340m s1 (1) (ii) average k.e. of nitrogen molecules = average k.e. of helium molecules (1) 1 4.00 10 3 = (1340)2 = 5.97 1021 J (1) 23 2 6.02 10 alternative schemes for (ii): average k.e. = =

3 kT (1) 2
6

3 1.38 1023 290 2 = 6.00 1021 J (1)

or

3 RT (1) 2 A 8.31 290 3 = 6.02 10 23 2 = 6.00 1021 J (1)


average k.e. =

(iii)

nkT or equivalent [or, at same temperature, V p no. of molecules] (1)


use of p = pHe =

2 120 = 80 kPa (1) 3


Total 9

Physics A PHYA5 - AQA GCE Mark Scheme 2010

Question 2
(a) adequate scale (1) points plotted correctly (1) best fit line (at least a point to right and left of line) (1) use of triangle for at least half line (1) 11.6 = 0.056 0.004 (C/s) (1) gradient = 208 (P = Q = mcT ) gives 48 = c (1.0) 0.056 (1) t t c = 860 60 J kg1 K1 (or J kg1 C1) (1) (use of Eth = ml gives) 48 200 = 32 103 l (1) l = 3.0 105 J kg1 (1) sensible assumption, e.g. no heat lost to surroundings or temperature does not change or heat is transferred to ice (1)

(b)

(c)

(d)

Total

10

Physics A PHYA5 - AQA GCE Mark Scheme 2010

Question 3
(a) (use of isotope instead of nucleus not accepted) there is equal probability of any nucleus decaying, it cannot be known which particular nucleus will decay next, it cannot be known at what time a particular nucleus will decay, the rate of decay is unaffected by the surrounding conditions, it is only possible to estimate the proportion of nuclei decaying in the next time interval QWC awarded by levels continuous curve starting at 5.5 105 Bq plus correct 1st half-life (2.6 yrs, 2.75 105 Bq (1) correct 2nd half-life (5.2 years, 1.4 105 Bq) (1) (allow C.E. for incorrect 1st half-life) (i) (use of T1/2 =

(b)

(c)

ln 2

gives) =

ln 2 (1) 2.6 3.15 10 7

= 8.5 10 (s1) (1) (8.46 109 (s1))

(ii)

(use of

5.5 10 d (1) = gives) = 8.5 10 9 dt = 6.5 1013 (atoms) (1) (allow C.E, for value of from (i))
5

(iii)

(use of N = N0et and A N gives) 5 5 ln( A0 / A ln(1.0 10 / 0.75 10 (1) t = = 8.5 10 9


= 3.4 107 (s) (1) (allow C.E. for value of from (i))

Total

10

Physics A PHYA5 - AQA GCE Mark Scheme 2010

Question 4
(a) (i) (ii)
238 92

4 2

(1) +

234 90

Th (1)

m = 238.05076 4.00260 234.04357 = 0.00459(u) (1) Q = 931 0.00459 (MeV) (1) = 4.3MeV (1) overall change in proton number (= 92 82) = 10 change in proton number due to particles (= 8 2) = 16 (1) therefore Z = 6 for the particles corresponding to the six particles (1) proton changes to a neutron plus a positron [or p n + +(+ ve + Q)] (1) Pb-206 has a lower neutron to proton ratio than U-238 (1) alpha emission raises the neutron to proton ratio slightly (1) emission lowers the ratio (more) (1) + emission increases neutron to proton ratio (1) positron emission competes with emission but is energetically less favourable (1)

(b)

(i)

(ii)

Total

11

Centre Number Surname Other Names Candidate Signature

Candidate Number

For Examiners Use

Examiners Initials

Question General Certificate of Education Advanced Level Examination Specimen paper 3 2010

Mark

1 2 3

Physics A
Unit 5A Section B Specimen Paper 3
For this paper you must have: a calculator a ruler a Data and Formulae Booklet

PHYA5A
Astrophysics

4 TOTAL

Time allowed The total time for both sections of this paper is 1 hour 45 minutes You are advised to spend approximately 50 minutes on this section. Instructions Use black ink or black ball-point pen. Fill in the boxes at the top of this page. Answer all questions. You must answer the questions in the spaces provided. Answers written in margins or on blank pages will not be marked. Do all rough work in this book. Cross through any work you do not want to be marked. Information The marks for questions are shown in brackets. The maximum mark for this section is 35 You are expected to use a calculator where appropriate. A Data and Formulae Booklet is provided as a loose insert. You will be marked for your ability to: - use good English - organise information clearly - use specialist vocabulary where appropriate.

Areas outside the box will not be scanned for marking

1.

(a)

(i)

Draw the diffraction pattern produced when light from a star passes through a circular aperture.

(ii)

Explain what is meant by the Rayleigh criterion for the resolution of two stars. Draw a diagram to help if you wish. ........................................................................................................................... ........................................................................................................................... ...........................................................................................................................

(3)

(b)

The star Arich in the constellation Virgo is two stars separated by an angle of 1.1 105 radians when viewed from Earth. Calculate the minimum diameter of a telescope objective which would just allow the two stars to be resolved. Assume the light from the star has a wavelength of 5.7 107 m. ..................................................................................................................................... ..................................................................................................................................... ..................................................................................................................................... ..................................................................................................................................... .....................................................................................................................................
(2) (Total 5 marks)

Turn over
0 1

Areas outside the box will not be scanned for marking

2.

(a)

The original dish design of the Lovell Radio Telescope at Jodrell Bank used a 50 mm open wire mesh. Estimate the minimum wavelength detectable using this design. .................................................................................................................................... ....................................................................................................................................
(1)

(b)

Before completion, the mesh was replaced by a solid metal surface of diameter 76 m capable of detecting radio signals as small as 60 mm wavelength. Calculate the resolving power of the telescope when detecting radiation of this wavelength. .................................................................................................................................... .................................................................................................................................... .................................................................................................................................... ....................................................................................................................................
(2)

(c)

The Jodrell Bank Observatory also has a 13 m diameter radio telescope. State two advantages the telescope described in part (b) has over this smaller telescope when detecting radio waves of the same wavelength. Support each answer with a calculation. advantage 1: .................................................................................................................................... .................................................................................................................................... .................................................................................................................................... .................................................................................................................................... advantage 2: .................................................................................................................................... .................................................................................................................................... .................................................................................................................................... ....................................................................................................................................
(4) (Total 7 marks)

Turn over
0 2

Areas outside the box will not be scanned for marking

3.

The table gives information on two stars. star Proxima Centauri Antares apparent magnitude 11 1.0 spectral class M M distance/pc 1.30 160

(a)

(i)

Explain what is meant by apparent magnitude .......................................................................................... ........................................................................................................................... absolute magnitude .......................................................................................... ...........................................................................................................................

(ii)

The two stars named in the table are viewed through a telescope. State and explain one difference in the appearance of the two stars. ........................................................................................................................... ........................................................................................................................... ...........................................................................................................................
(4)

(b)

(i)

Draw a Hertzsprung-Russell diagram for main sequence stars, Giant Stars and White Dwarfs.

absolute magnitude 10 5 0 5 10 15 0 B A F G K M spectral class

Turn over
0 3

Areas outside the box will not be scanned for marking

(ii)

With reference to the table, calculate the absolute magnitude of Proxima Centauri. ........................................................................................................................... ........................................................................................................................... ...........................................................................................................................

(iii)

Given that the absolute magnitude of Antares is -5, mark and label with an X its approximate position on the Hertzsprung-Russell diagram.
(6)

(c)

(i)

Giving your reason, what can you say about the surface temperature of the two stars? ........................................................................................................................... ...........................................................................................................................

(ii)

Hence, deduce which star has the larger diameter. Explain how you arrive at your answer. ........................................................................................................................... ........................................................................................................................... ...........................................................................................................................
(4) (Total 14 marks)

Turn over
0 4

Areas outside the box will not be scanned for marking

4.

Quasars are star-like objects whose spectra have very large red shifts. (a) What property of quasars led to their discovery? ..................................................................................................................................... .....................................................................................................................................
(1)

(b)

3C 48 is a quasar which has one of the largest red shifts ever measured. A particular spectral line has a value of 279.8 nm when measured using a laboratory source. The equivalent line in the spectrum of this quasar is 382.5 nm. (i) Calculate the speed of this quasar relative to the Earth, ignoring relativistic effects. ........................................................................................................................... ...........................................................................................................................

(ii)

Show that the distance to the quasar is approximately 2 109 pc. Assume the Hubble constant is 65 km s1 Mpc1. ........................................................................................................................... ...........................................................................................................................
(3)

Turn over
0 5

Areas outside the box will not be scanned for marking

(c)

To have the same apparent magnitude as Quasar 3C 48, the Sun would have to be placed approximately 2 103 pc from the Earth. (i) Assuming the distance to the quasar is 2 109 pc, use the inverse square law to estimate the ratio of the power output of Quasar 3C 48 to that of the Sun. ........................................................................................................................... ........................................................................................................................... ........................................................................................................................... ...........................................................................................................................

(ii)

A controversy exists concerning the nature of quasars. List the properties which give rise to this controversy. ........................................................................................................................... ........................................................................................................................... ........................................................................................................................... ........................................................................................................................... ........................................................................................................................... ........................................................................................................................... ........................................................................................................................... ...........................................................................................................................
(4) (Total 8 marks)

Turn over
0 6

Physics A PHYA5A - AQA GCE Mark Scheme 2010

Instructions to Examiners
1. Give due credit for alternative treatments which are correct. Give marks for what is correct in accordance with the mark scheme; do not deduct marks because the attempt falls short of some ideal answer. Where marks are to be deducted for particular errors, specific instructions are given in the marking scheme. Do not deduct marks for poor written communication. Refer the scripts to the Awards meeting if poor presentation forbids a proper assessment. In each paper, candidates are assessed on their quality of written communication (QWC) in designated questions (or part-questions) that require explanations or descriptions. The criteria for the award of marks on each such question are set out in the mark scheme in three bands in the following format. The descriptor for each band sets out the expected level of the quality of written communication of physics for each band. Such quality covers the scope (eg relevance, correctness), sequence and presentation of the answer. Amplification of the level of physics expected in a good answer is set out in the last row of the table. To arrive at the mark for a candidate, their work should first be assessed holistically (ie in terms of scope, sequence and presentation) to determine which band is appropriate then in terms of the degree to which the candidates work meets the expected level for the band. QWC descriptor mark range see specific mark scheme Good Excellent see specific mark scheme Modest Adequate see specific mark scheme Poor Limited The description and/or explanation expected in a good answer should include a coherent account of the following points: see specific mark scheme Answers given as bullet points should be considered in the above terms. Such answers without an overview paragraph in the answer would be unlikely to score in the top band. 3. An arithmetical error in an answer will cause the candidate to lose one mark and should be annotated AE if possible. The candidates incorrect value should be carried through all subsequent calculations for the question and, if there are no subsequent errors, the candidate can score all remaining marks. The use of significant figures is tested once on each paper in a designated question or partquestion. The numerical answer on the designated question should be given to the same number of significant figures as there are in the data given in the question or to one more than this number. All other numerical answers should not be considered in terms of significant figures. Numerical answers presented in non-standard form are undesirable but should not be penalised. Arithmetical errors by candidates resulting from use of non-standard form in a candidates working should be penalised as in point 3 above. Incorrect numerical prefixes and the use of a given diameter in a geometrical formula as the radius should be treated as arithmetical errors. Knowledge of units is tested on designated questions or parts of questions in each a paper. On each such question or part-question, unless otherwise stated in the mark scheme, the mark scheme will show a mark to be awarded for the numerical value of the answer and a further mark for the correct unit. No penalties are imposed for incorrect or omitted units at intermediate stages in a calculation or at the final stage of a non-designated unit question. All other procedures including recording of marks and dealing with missing parts of answers will be clarified in the standardising procedures.

2.

4.

5.

6.

7.

Physics A PHYA5A - AQA GCE Mark Scheme 2010

PHYA5A: Astrophysics

Question 1 (a) (i) diagram to show: at least two bright rings (1) radius of central spot greater than thickness of secondary rings (1) two intensity or circular patterns (1) two objects can just be resolved when central maximum of one coincides with first minimum of other (1)

Max 3

(ii)

(b)

5.7 10 7 = D= (1) 1.110 5


= 5.2 102m (1) Total

Question 2 (a)

20

= 0.05 and = 1.0 m (1)

(b)

(1) 2

0.06 = = 7.9 104 rad (1) 76


(c) advantage 1: power D2 much more power detected by larger diameter telescope (1)
76 larger ratio of power detected = = 34 (1) 13
2

advantage 2: resolving power D larger diameter has greater resolving power (1) ratio of resolving power =

76 = 5.8 (1) 13

(inverse accepted if angle referred to) Total 7

Physics A PHYA5A - AQA GCE Mark Scheme 2010

Question 3
(a) (i) (ii)

apparent magnitude: brightness of star as seen from Earth (1) absolute magnitude: apparent magnitude at a distance of 10 pc (1)
one star is much brighter (1) has lower value of apparent magnitude (1) correct main sequence (1) correct Giants and White Dwarfs (1)

(b)

(i)

(ii)

d m M = 5 log (1) 10 1.3 gives 11 M = 5 log (1) 10 M = 11 + 4.43 = 15.4 (1)


correct position on diagram (spectral class M, abs magnitude 5) (1) same temperature [or temperature less than 3500K ] (1) same spectral class (1) Antares is brightest (and at same temperature) (1) so has largest surface area [diameter] (1)

(iii)

(c)

(i) (ii)

Total

14

Question 4
(a) (b) (quasars are) strong radio sources (1) (i) (use of

v 3.00 10 8 (382.5 279.8) = () gives) v = () c 279.8


3

(1)
= ()1.10 108 ms1 (1) (ii) (use of v = Hd gives) d =
1.10 10 8

(1) (= 1700 Mpc) 65 10 3 (allow C.E. for value of v from (b)(i))


Psun (1) (2 10 3 ) 2

(c) (i)

Pquasar
9 2

(2 10 ) Pquasar = 1012 (1) Psun

(ii)

very large power (1) large distance away or large redshift (1) small size/star like/rapid variation (1)

Total

Potrebbero piacerti anche